You are on page 1of 100

BLOCK B 2015

ACADEMICS COMMITTEE
TABLE OF CONTENTS

page
Elective Officials (cont’d)
1 A. Vacancies and Succession
6 B. Local Legislation
15 C. Disciplinary Action
21 1. The Ombudsman
23 2. The Courts
26 3. Removal
28 D. Recall

Human Resources and Development


32 A. Organizational Structure
32 B. Appointment, Vacancies, Compensation, Grievance
32 C. Resignation of Elective Local Officials
33 D. Administrative Discipline Against Appointive Local Officials
33 E. Sec. 91-97, LGC
38 F. Prohibited Business & Pecuniary Interests
38 G. Practice of Profession

The Local Boards, Development Councils, and Special Economic Zones


40 A. The Local School Boards
41 B. The Local Health Boards
41 C. The Local Development Councils
42 D. The Local Peace and Order Council
42 E. Autonomous Special Economic Zones

Other Provisions Applicable to Local Government Units


43 A. Settlement of Boundary Disputes
44 B. Local Initiative and Referendum

Local Taxation & Fiscal Matters


48 A. Local Government Taxation
51 B. Common Limitations on Taxing Powers of LGUs
53 C. Franchise Taxes
55 D. Amusement Taxes
56 E. Real Property Taxation
64 F. Violations of Tax Ordinance

Shares of LGUs in the Proceeds of National Taxes


65 A. Internal Revenue Allocation
67 B. Share of LGUs in National Wealth
67 C. Credit Financing
67 D. Local Fiscal Administration
69 E. Property and Supply Management in LGUs
The Local Government Units
70 A. The Barangay
71 1. Katarungang Pambarangay
75 2. Sangguniang Kabataan
77 B. The Municipality
81 C. The City
84 D. The Province

86 Appointive Local Officials Common to all Municipalities, Cities, Provinces

88 Leagues of LGUs and Elective Officials

Miscellaneous and Final Provisions


90 A. Provisions for Implementation
90 B. Transitory Provisions
91 C. The Autonomous Regions
94 D. The Metropolitan Manila Development Authority

Sources:

B2015 Class Case Digest


B2015 Class Notes
The Local Government Code, by Aquilino Pimentel
UP Law Political Law Bar Reviewer
2009 LocGov Reviewer
D2014 LocGov Reviewer

Supplementary Documents:

B2015 Weekly Digest Compilations


B2015 LocGov Midterms Reviewer
Compilation of Special Laws for the Midterm Exams
Compilation of Special Laws – Post Midterms Coverage

Prepared By:

B2015 Academics Committee – LocGov Reviewer Ops


TITLE TWO: ELECTIVE OFFICIALS
continued

In case of TEMPORARY vacancy – the LGC is silent on the mode of


VACANCIES AND SUCCESSION succession in the event of a temporary vacancy (except for local chief
executives)
- De Leon v. Esguerra says that the mode of succession
PERMANENT VACANCIES: provided for permanent vacancies may likewise be observed
LOCAL CHIEF EXECUTIVE in case of a temporary vacancy in the same office

Breakdown of Sec. 44, LGC:


Vacancy in Punong Barangay’s Office
(a) If permanent vacancy occurs in the office of governor or - The highest ranking Sanggunian member or in case of his
mayor: permanent inability, the 2nd highest ranking member and so
(1) Vice-governor or vice mayor shall become the on down the line assumes office
governor or mayor
IN SUMMARY:
 If permanent vacancy occurs in the offices of governor,
vice-governor, mayor or vice-mayor: VACANT POSITION SUCCESSOR
1) Highest ranking sanggunian member shall PERMANENT VACANCY
become the governor, vice-governor, mayor, or Governor Vice-governor
vice-mayor Mayor Vice-mayor
2) In case of the latter’s permanent inability, then Governor, vice-governor Highest ranking Sanggunian
the 2nd highest ranking member Mayor, vice-mayor member
 Subsequent vacancies in said office shall be filled Highest ranking Sangguniang
automatically by other sanggunian members Punong Barangay
Barangay member
according to their ranking PERMANENT INABILITY
Highest ranking Sangguniang Second highest ranking
(b) If permanent vacancy occurs in office of Punong Barangay member (as successor of the Sanggunian member
 Highest ranking sanggunian barangay member shall Governor, vice-governor, Mayor, (subsequent vacancies filled
become punong barangay or vice-mayor) according to rank)
 If latter has permanent inability, then 2nd highest
ranking member

(c) Tie between or among highest-ranking sanggunian


members shall be resolved by drawing lots PERMANENT VACANCIES:
SANGGUNIAN
(d) Successors shall serve only the unexpired term of their
predecessors Breakdown of Sec. 45, LGC: Permanent Vacancies in the Sanggunian:

 Permanent Vacancy = arises when elective local (a) Permanent vacancies in sanggunian where automatic
official fills a higher vacant office, refuses to assume succession in Sec. 44 do not apply shall be filled by
office, fails to qualify, dies, is removed from office, APPOINTMENT:
voluntarily resigns or is otherwise permanently (1) President, through Exec. Sec.
incapacitated to discharge the functions  In case of Sangguniang Panlalawigan and
Sangguniang Panlungsod of highly urbanized
 Ranking in sanggunian = determined on the basis of cities and independent component cities
proportion of votes obtained by each winning (2) Governor
candidate to total number of registered voters in each  In case of Sangguniang Panlungsod of
district in immediately preceding local election component cities and Sangguniang Bayan
(3) City/Municipal Mayor
Permanent Vacancy – arises when an elective official:  In case of Sangguniang Barangay
1. Fills a higher vacant office,  Upon recommendation of Sangguniang
2. Refuses to assume office, Barangay concerned
3. Fails to qualify, dies,
4. Is removed from office, (b) Only the nominee of political party under which the
5. Voluntarily resigns or sangguniang member concerned had been elected and
 Included there us the act of local officials who file COCs whose elevation to position next higher in rank created the
for offices other than the ones they are occupying or for last vacancy in the sanggunian shall be appointed
presidency or vice-presidency of the country  EXCEPT for Sangguniang Barangay
6. Is otherwise permanently incapacitated to discharge the  Appointee shall come from same political party as that
functions of the sangguniang member who caused the vacancy
 Shall serve the unexpired term
There is no vacancy whenever the office is occupied by a legally  In the appointment, the following are conditions sine
qualified incumbent. (Menzon v. Petilla) qua non:
- NOTE: Succession is automatic. 1. A nomination and
2. Certificate of membership of the appointee from
the highest official of the political party
concerned
 Without the nomination and certification, c. When permanent vacancy refers to position of
appointment is null and void youth representatives (go to (d))
 Shall also be a ground for administrative action d. When permanent vacancy concerns the position of
against official responsible barangay electives in the sanggunian of a
municipality, city, or province (go to (d))
(c) In case of permanent vacancy is caused by a sangguniang
member who does NOT belong to any political party: Sec. 45 (c) – Pimentel clarifies the phrase “local chief executive”
 Local chief executive shall appoint a qualified person - Under Sec. 45(a) there is an implicit policy to vest in the
 Upon recommendation of sanggunian concerned President, the governor, and the mayor in descending order
the exercise of executive power whether to appoint in order
(d) In case of vacancy in representation of the youth and the to fill vacancies in local councils
barangay in the sanggunian  Same policy in Sec. 63 on preventive suspension
 Vacancy shall be filled automatically by the official - Thus, “local chief executive”:
next in rank in the organization concerned  Refers to the governor with respect to vacancies in
Sangguniang Panlungsod of component cities and
Application of Sec. 45 – This deals with permanent vacancies in the Sangguniang Bayan, or to the mayor with respect to
various sanggunians under circumstances where Sec. 44 does not apply Sangguniang Barangay
 President cannot be referred to as local chief executive
Sec. 45(b) in Sec. 45(c) but this is a misnomer
- RATIO: to maintain the party representation in the - So Pimentel thinks that the more accurate phrase to use is
sanggunian as willed by the people in the elections “authorities concerned”
- Two certifications are required, without them the  Meaning that with respect to Sangguniang Panlalawigan
appointment is null and void: and Sagguniang Panlungsod of highly urbanized cities
1. Certificate of nomination to the nominee and independent component cities, President appoints
2. Certificate of membership of the nominee duly signed the vacancy
by the highest official of the political party
Vacancies in the Sangguniang Barangay – There is only one rule
NOTE: The 2-Certification requirement does NOT apply: - Any vacancy must be made by the mayor
a. When person who caused permanent vacancy  Upon recommendation of the sanggunian
does not belong to any party (go to (c)) - RATIO: Members of the sangguniang barangay are not
b. When permanent vacancy occurs in sangguniang allowed to have party affiliations
barangay

SUMMARY: Vacancy in the Sanggunian (from Farinas v. Barba)

IF PRIOR MEMBER NOT A MEMBER IF PRIOR MEMBER WAS A MEMBER


POSITION APPOINTING AUTHORITY
OF ANY POLITICAL PARTY OF A POLITICAL PARTY
Recommendation of the
Sangguniang Panlalawigan Nomination and Certification of the
Sangguniang Panlalawigan
The President political party of the member who
Sangguning Panlungsod
Through the Executive Secretary Recommendation of the caused the vacancy issued by the
(Highly urbanized and independent
Sangguniang Panlungsod highest official of the political party
component cities)
Sangguniang Panlungsod Recommendation of Nomination and Certification of the
(of component cities) Sangguniang Panlungsod political party of the member who
Governor
Recommednation of caused the vacancy issued by the
Sangguniang Bayan
Sangguniang Bayan highest official of the political party
Recommendation of Sangguniang Barangay
Sangguniang Barangay City or Municipal Mayor
(note that no party affiliations in Sangguniang Barangay)
TEMPORARY VACANCIES: Designation of OICs
- Designation by the local chief of officials other than their
Temporary Vacancy Occurs when local chief executive is unavailable vice-governors, vice-mayors, or the next highest ranking
due to: Sangguniang Barangay member as OICs whenever the former
1. Leave of absence are absent from their jurisdictions is PROHIBITED by the LGC
2. Travelling outside territorial jurisdiction  EXCEPTION: When the particular local chief is travelling
3. Suspension from office for 3 days or less, outside their territorial jurisdiction
but within the country
Breakdown of Sec. 46, LGC: Temporary Vacancy in the Office of the
Local Chief Executive Vacancy Caused by Travel
- When the local chief travels outside their territorial
(a) When governor, city/municipal mayor, or punong barangay jurisdiction but within the country for 3 days or less, he is
NOT considered physically absent
is temporarily incapacitated to perform his duties for
physical or legal reasons  Thus, NO vacancy (not even a temporary one)
- Thus, the local chief may designate OIC of his office
 Such as LOA, travel abroad, and suspension of office
(1) Vice-governor, city/municipal vice-mayor, or highest  Failure to appoint OIC authorizes the vice-governor or
ranking Sangguniang Barangay member shall vice-mayor, or the highest member of the Sanggunian to
automatically exercise powers and perform duties assume office on the 4th day of absence
and functions of the local chief executive
a) EXCEPT the power to appoint, suspend, or Travel Beyond Three Days – when travel exceeds 3 days, vice-governor,
dismiss employees vice-mayor, or the highest member of the Sanggunian is to assume
1) This can only be exercised if period of office
temporary incapacity EXCEEDS 30 working
days SUMMARY:

(b) Said temporary incapacity shall terminate upon submission OFFICE WHERE TEMPORARY WHO TEMPORARILY SUCCEEDS
to appropriate sanggunian of a written declaration by local VACANCY OCCURS INTO OFFICE
chief executive that he has reported back to office Vice-Governor
Governor
 When incapacity was due to legal causes, local chief (automatically)
executive shall submit necessary documents showing Vice Mayor
Mayor
that said legal causes no longer exist (automatically)
Highest ranking sanggunian
Punong barangay
(c) When incumbent local chief executive is traveling within the member (automatically)
country but outside his territorial jurisdiction for a period Person designated in writing by
not exceeding 3 consecutive days: Local chief executive is the local chief executive
(1) He may designate in writing the OIC of said office travelling within the country Vice-governor, vice-mayor, or
 Authorization to specify the powers and functions that BUT outside territorial highest ranking sanggunian
local official shall exercise in the absence of the local jurisdiction for a period not member on the 4th day of absence
chief executive exceeding 3 consecutive days if local executive fails or refuses to
 EXCEPT power to appoint, suspend, or dismiss designate successor
employees

(d) If local chief executive fails or refuses to issue authorization Breakdown of Sec. 47, LGC: Approval of Leaves of Absences
(1) Vice governor or city/municipal vice mayor, or highest
ranking Sangguniang Barangay member, shall have (a) LOAs of local elective officials shall be approved as follows:
the right to assume powers, duties, functions on the (1) LOA of governor and mayor of highly urbanized city or
FOURTH day of absence independent component city
(2) Subject to limitation in (c)  Approved by President or duly authorized
representative
(e) Local chief executive shall in no case authorize any local (2) LOA of vice governor or city/municipal vice mayor
official to assume powers, duties, and functions of the office  Approved by local chief executive concerned
other than the vice-governor, city/municipal vice-mayor, or  LOAs of members of sanggunian and its
highest ranking Sangguniang Barangay member, as the employees shall be approved by vice governor or
case may be city/municipal mayor
 EXCEPT as provided above (3) LOA of component city or municipal mayor
 Approved by governor
Temporary Vacancies in Local Executive Office (4) LOA of punong barangay
- Temporary vacancies will be filled by the vice-governor, vice-  Approved by city or municipal mayor
mayor or highest ranking Sangguniang Barangay member, as  LOA of Sangguaniang Barangay memebrs shall
the case may be be approved by punong barangay
- Automatic exercise of the powers of the temporarily vacant
office means that they need not be appointed by anyone (b) Whenever LOA application is not acted upon within 5 days
after receipt
Certain Powers Withheld  Application deemed approved
- Occupant CANNOT exercise the power of:
1. Appointment of employees NOTE: Take note of the officials authorized to approve LOA application
2. Suspension of employees and the 5-day period to act on the application.
3. Dismissal of employees
- UNLESS the period of temporary incapacity exceeds 30
working days
GAMBOA v. AGUIRRE Class Notes: Good law? The recommendations are not via DILG
July 20, 1999 Secretary anymore

Summary: The Governor of Negros Occidental went on an official trip


abroad. Before he left, he designated the Vice-Governor Gamboa as the FARINAS v. BARBA
Acting Governor. While being an Acting Governor. Gamboa sought to April 19, 1996
exercise the power of the Vice-Governor by presiding over the
Sanuguniang Panlalawigan. To this, the SP members Aguirre and Summary: To fill the vacancy caused by a member, who did not belong
Araneta objected. These SP members brought a petition for prohibition to any political party, in the Sangguniang Bayan (SB) of San Nicolas,
against the Vice-Governor, then the Acting Governor. The novel issue Ilocos Norte, Mayor Barba to Governor Fariñas the appointment of
presented in this case is whether a Vice Governor, while being the Palafox. The SB likewise recommended Palafox. On the other hand, the
Acting Governor, continue to preside over the sessions of SP. The SC Sangguniang Panlalawigan of Ilocos Norte recommended Nacino.
answered the question in the negative. Governor Fariñas appointed Nacino. Mayor Barba appointed Palafox.
Governor Fariñas filed a quo warranto petition. The RTC of Ilocos Norte
Doctrine: When the Vice-Governor exercises the "powers and duties" of dismissed and upheld the appointment by Mayor Barba. The SC also
the Office of the Governor, he does not assume the office of the dismissed, but on the ground that neither Palafox nor Nacino is entitled
Governor. He only "acts" as the Governor but does not "become" the to the position, since they were not properly appointed.
Governor. His assumption of the powers of the Governor does not
create a permanent vacancy in his position as the Vice-Governor. Doctrine:
Necessarily, he does not relinquish nor abandon his position and A. Where the Permanent Vacancy is Caused by a Sanggunian
title as Vice-Governor by merely becoming an Acting Governor, Member belonging to a Political Party
(not Governor) or by merely exercising the powers and duties of the 1. Sangguniang Panlalawigan and Sangguniang
higher officer. Panlungsod of highly urbanized cities and independent
component cities - The President, through the Executive
Being the Acting Governor, the Vice-Governor cannot continue to Secretary, upon the nomination and certification of the
simultaneously exercise the duties of the latter office, since the nature political party to which the member who caused the
of the duties of the provincial Governor call for a full-time occupant to vacancy belonged, as provided in Section 45(b).
discharge them. The creation of a temporary vacancy in the office of the
Governor creates a corresponding temporary vacancy in the office of 2. Sangguniang Panlungsod of component cities and
the Vice-Governor whenever the latter acts as Governor by virtue of Sangguniang Bayan - The Governor upon the
such temporary vacancy. This event constitutes an "inability" on the nomination and certification of the political party to
part of the regular presiding officer (Vice Governor) to preside during which the member who caused the vacancy belonged,
the SP sessions, which thus calls for the operation of the Article 49(b, as provided in Section 45(b).
LGC — “(i)n the event of the inability of the regular presiding officer to
preside at the sanggunian session, the members present and B. Where the Vacancy is Caused by a Sanggunian Member Not
constituting a quorum shall elect from among themselves a temporary Belonging to a Political Party
presiding officer." 1. Sangguniang Panlalawigan and Sangguniang
Panlungsod of highly urbanized and independent
It has been held that if a Mayor who is out of the contrary is considered component cities - The President, through the Executive
"effectively absent", the Vice-Mayor should discharge the duties of the Secretary, upon recommendation of the Sangguniang
mayor. The same is true for the governor and Vice-Governor. “Effective" Panlalawigan or Sangguniang Panlungsod as the case
absence is one that renders the officer concerned powerless, for the may be.
time being, to discharge the powers and prerogatives of his office.
2. Sangguniang Panlungsod of component cities and
Class Notes: Sir notes that the rule on the temporary vacancy of the vice Sangguniang Bayan - The Governor upon
governor finds no legal basis in law, just jurisprudence. recommendation of the Sangguniang Panlungsod or
- So cite Gamboa for the rule on temporary vacancy for this Sangguniang Bayan as the case may be.
specific vacancy
C. Where the Vacancy is Caused by a Member of the
Sangguniang Barangay - City or Municipal Mayor upon
DOCENA v. SANGGUNIANG PANLALAWIGAN OF EASTERN SAMAR recommendation of the Sangguniang Barangay.
June 25, 1991

Summary: The Secretary of Local Governments, Santos, appointed DAMASEN v. TUMAMO


Docena to succeed the late Capito as member of the Sangguniang February 17, 2010
Panlalawigan of Eastern Samar (SPES). Santos then appointed Alar to
the same position. Santos then recalled Alar’s appointment. SPES issued Summary: The Vice Mayor of San Isidro, Isabela died. As a result, a
a resolution recognizing Alar’s appointment. Santos recalled Docena’s permanent vacancy was created in the Office of the Vice Mayor. The
appointment. Docena filed a petition to the SC. highest-ranking member of the Sangguniang Bayan, pursuant to Sec. 44
of RA 7160, was elevated to the position of Vice Mayor. As a result, a
Doctrine: From the tenor of the appointment of Docena, it was intended permanent vacancy was created in the Sangguniang Bayan. The Mayor
to be permanent, to fill the permanent vacancy caused by Capito's death. recommended to the Governor the appointment of Oscar Tumamao, a
As such, it was to be valid for the unexpired portion of the term of the member of the Laban ng Demokratikong Pilipino (LDP), the same
deceased member. The said appointment had been accepted by Docena, political party to which the highest raking Sanggu. Memmber (now the
who already assumed office. Docena's appointment had already become Vice Mayor) belonged. Tumamao took this oath as a Sanggu Member
complete and enforceable at the time it was supposed to have been before the Mayor. Subsequently, petitioner Lucky Damasen became a
"superseded" by the appointment in favor of Alar. Docena had already member of LDP after taking his oath of affiliation before the LDP
acquired security of tenure in the position and could be removed Provincial Chairperson. The Governor appointed Damasen as Sanggu.
therefrom only for any of the causes, and conformably to the procedure, Member. Lucky Damasen filed a petition for quo warranto against
prescribed by the LGC. These requirements could not be circumvented Tumamao. The RTC ruled in favor of Lucky Damasen. The CA ruled in
by the simple process of recalling his appointment. favor of Tumamao. The SC, giving credence to the two letters- one,
revoking the nomination of Lucky Damasen and two, stating that Lucky
Damasen is not a bona fide member of the LDO – ruled in favor of a. If the highest member changes political party mid-term
Tumamao. Damasen is not entitled, not even qualified, to assume the (1) Same result as the case of Navarro
vacant position in the Sangguniang Bayan because he is not a bona fide (2) Ratio: party representation; the party they were
member of LDP. voted for by the people
b. If the party was dissolved mid-term
Doctrine: Sec. 45 of RA 7160 provides for conditions for the rule of (1) Maybe we can apply Sec. 45(c) on the rule
succession to apply: First, the appointee shall come from the same regarding no political affiliation
political party as that of the Sanggunian member who caused the
vacany. Second, the appointee must have a nomination and a Certificate
of Membership from the highest official of the political party. MIRANDA v. CARREON
Concerned. April 11, 2003

In Navarro v. CA, the SC explained the reason behind the rule of Summary: Vice Mayor Navarro, while serving as Acting Mayor,
succession under Sec. 45 (b) of RA 7160: the reason behind the right appointed the respondents in various positions in the city government.
given to a political party to nominate a replacement where a permanent After Mayor Miranda has served his suspension, he returned to office
vacancy occurs in the Sanggunian is to maintain the party and ordered an investigation be made on the appointments made by
representation as willed by the people in the election. Vice Mayor Navarro. The result of the investigation states that the
respondents ought to be removed for “wanting in their performance”.
Mayor Miranda removed the respondents from office. CSC issued a
VICTORIA v. COMELEC resolution reinstating the respondents. Later, Mayor Miranda was
January 10, 1994 disqualified to run for the next election. His son then substituted for
him. His son won and was proclaimed as mayor. He then filed a MRF on
Summary: Due to the suspension of Governor Romeo Salalima of the the CSC resolution. Subsequently, COMELEC disqualified him as Mayor.
Province of Albay, Vice-Governor Danilo Azana automatically assumed Vice Mayor Navarro replaced him as Mayor. Mayor Navarro filed a
the powers and functions of the governor, leaving vacant his post as “Motion to Withdraw the MFR”. The issue here is WON Mayor Navarro
vice-governor. Azana’s position was occupied by private respondent has the authority to withdraw actions filed by the previous Mayor. SC
Jesus James Calisin being the highest ranking members of the ruled that the successor has such authority by virtue of Section 17, Rule
Sangguniang Panlalawigan of Albay, based on COMELEC’s certification. 3 of the 1997 Rules of Civil Procedure.
Petitioner Juan Victoria filed a motion for reconsideration of the
COMELEC resolution but the same was denied. The Supreme Court Doctrine: "Sec. 17. Death or separation of a party who is a public officer.
upheld the validity of the COMELEC resolution and held that Calisin was — When a public officer is a party in an action in his official capacity
the highest ranking Sangguniang Panlalawigan member following the and during its pendency dies, resigns or otherwise ceases to hold office,
rule laid down in Sec. 44 of the Local Government Code. the action may be continued and maintained by or against his successor
if, within thirty (30) days after the successor takes office or such time as
Doctrine: Sec. 44 of the Local Government Code states that “for may be granted by the Court, it is satisfactorily shown by any party that
purposes of succession [in case of vacancies], ranking in the sanggunian there is substantial need for continuing or maintaining it and the
shall be determined on the basis of the proportion of votes obtained by successor adopts or continues or threatens to adopt or continue the
each winning candidate to the total number of registered voters in each action of his predecessor."
district in the immediately preceding local election”.
It is clear from the above Rule that when petitioner ceased to be mayor
of Santiago City, the action may be continued and maintained by his
NAVARRO v. CA successor, Mayor Amelita Navarro, if there is substantial need to do so.
March 28, 2001

Summary: Upon the death of the mayor, Vice Mayor Aquino succeeded
to the said office by operation of law. Following Sec. 44 and 45 LGC,
Sanggunian member Tamayo (highest number of votes among the
members) succeeded to the office of vice mayor. Due to the vacancy in
the Sangguniang Bayan, Governor appointed Navarro to the
Sanggunian. Navarro belonged to the same political party as Tamayo
(REFORMA-LM). Respondents filed this case assailing the appointment
of Navarro and argued that the appointee should come from Lakas
NUCD-KAMPI which is the political party of Vice Mayor Aquino since he
is the one who created the vacancy. The SC held that the appointee
should come from REFORMA-LM since the purpose and intent of the
law is to maintain party representation in the Sanggunian as willed by
the people. Since REFORMA-LM lost one member, another one from its
party should be appointed to the Sanggunian.

Doctrine: The reason behind the right given to a political party to


nominate a replacement where a permanent vacancy occurs in the
Sanggunian is to maintain the party representation as willed by the
people in the elections.

Last vacancy in the Sanggunian refers to that created by the elevation of


the member formerly occupying the next higher in rank which in turn
also had become vacant. The term “last vacancy” is used in Sec. 45(b) to
differentiate it from the other vacancy previously created.

Class Notes:
- Sir said that the CA decision was cute but it is still wrong
- Situations: What would be the effect if:
Temporary Presiding Officer - note that there is no automatic
LOCAL LEGISLATION succession; members in a quorum must elect among themselves a
temporary presiding officer

Breakdown of Sec. 48, LGC: Local Legislative Power INTERNAL RULES OF PROCEDURE

Local legislative power shall be exercised by: Breakdown of Sec. 50, LGC: Internal Rules of Procedure
1. Sangguniang Panlalawigan for Province
2. Sangguniang Panlungson for City (a) On the first regular session following election and within 90
3. Sangguniang Bayan for Municipality days thereafter, sanggunian shall adopt or update its
4. Sangguniang Barangay for Barangay existing rules of procedure

Four Local Sanggunians – Take note of the 4 local legislative bodies (b) Rules of procedure shall provide:
- Their powers are outlined in: (1) Organization of the sanggunian
a. Sec. 391 – Sangguniang Barangay  Election of its officers
b. Sec. 447 – Sangguniang Bayan  Creation of standing committed which shall
c. Sec. 458 – Sangguniang Panlungsod include but not limited to Committees on:
d. Sec. 468 – Sangguniang Panlalawigan a. Appropriations
b. Women and family
Subordinate Legislation c. Youth and sports development
- Local political subdivisions are able to legislate only by virtue d. Environmental protection
of a valid delegation of legislative power from Congress e. Cooperatives
 Except that power to create own sources of revenue  General jurisdiction of each committee
and to levy taxes are conferred by Constitution  Election of chairman and members of committee
- As delegates, they cannot go against national policy and (2) Order and calendar of business for each session
Congress (3) Legislative process
(4) Parliamentary procedures
Contempt Powers NOT vested in LGUs  Includes conduct of meetings during sessions
- Unlike Congress, LGUs do not have an inherent power to cite (5) Discipline of members for disorderly behavior and
anyone for contempt absences without justifiable cause for 4 consecutive
- No provision in the LGC sessions
 For which they may be:
Validity of Ordinance a. Censured,
- Ordinances enacted in the exercise of powers under the LGC b. Reprimanded, or
enjoy the presumption of constitutionality c. Excluded from session for not more than 60
- Validity of an ordinance: (Lagcao v. Labra) days, or
1. Must be within the powers of the LGU d. Expelled
2. Passed according to the procedure prescribed by law  Provided that penalty of suspension or expulsion
3. In accordance with certain substantive principles: shall require 2/3 vote of all members
a. Must not contravene the Constitution or any  Provided that member is automatically expelled
statute if he is convicted by final judgment of:
b. Must not be unfair or oppressive a. Imprisonment of at least 1 year or
c. Must not be partial or discriminatory b. Any offense involving moral turpitude
d. Must not prohibit but may regulate trade (6) Other rules as sanggunian may adopt
e. Must be general and consistent with public policy
f. Must not be unreasonable Sanggunian Rules of Procedure
- Sec. 50 deals with procedural matters concerned with the
PRESIDING OFFICER organization and conduct of the sessions of the sanggunians
- NOTE: LGC does not require the completion of the updating
Breakdown of Sec. 49, LGC: Presiding Officer or adoption of internal rules before the sanggunian could act
 It simply requires that the matter of adopting or
(a) The presiding officers: updating be taken up during the first session
(1) Vice governor for the Sangguniang Panlalawingan
(2) City vice mayor for Sangguniang Panlungsod Committees
(3) Municipal vice mayor for Sangguniang Bayan 1. Sanggunian is allowed to create standing committees
(4) Punong barangay for the Sangguniang Barangay 2. But LGC provides for the 5 MANDATORY committees:
a. Appropriations
Presiding officer shall vote only to break a tie b. Women and family
c. Youth and sports development
(b) In the event of inability of the regular presiding officer to d. Environmental protection
preside at a session: e. Cooperatives
 Members present and constituting a quorum shall
elect from among themselves a temporary presiding Disciplinary Action
officer - Sec. 50 recognizes 2 general causes for which a member may
 He shall certify within 10 days from passage of be subject to disciplinary action:
ordinances enacted and resolutions adopted by 1. Disorderly behavior
sanggunian in the session he temporarily presided 2. Four consecutive absences without justifiable cause
- Sub section (b)(5) provides for the sanctions
Presiding Officer – this is designed to distribute powers among the local  2/3 votes needed for suspension or expulsion
elective officials so that the sanggunian can properly check the local  Automatic expulsion if member is convicted:
chief executive a. Imprisonment of at least 1 year
b. Offense involving moral turpitude
FULL DISCLOSURE OF FINANCIAL AND BUSINESS INTERESTS THE SESSIONS

Breakdown of Sec. 51, LGC: Full Disclosure of Financial and Business Breakdown of Sec. 52, LGC: Sessions
Interests of Sanggunian Members
(a) On 1st day of session immediately following election of
(a) Every sanggunian member shall, upon assumption of office, members, sanggunian shall fix the day, time, and place of its
make full disclosure of his business and financial interests regular sessions
 Disclose any business, financial, or professional  How? By RESOLUTION
relationship or any relation by affinity or  Minimum number of regular sessions:
consanguinity within 4th civil degree which he may 1. Once a week for Sangguniang Panlalawigan,
have with any person, firm, or entity, affected by any Panlungsod, and Bayan
ordinance or resolution UNDER CONSIDERATION by 2. Twice a month for Sangguniang Barangay
the sanggunian
 Which relationship may result in conflict of (b) Special sessions
interest  When public interest so demands
 Interest shall include:  May be called by:
1. Ownership of stock, capital, investment 1. Local chief executive, or
2. Contracts or agreements 2. Majority of sanggunian members
 Conflict of Interest = one where it may be reasonably
deduced that a member of a sanggunian may NOT act (c) All sanggunian sessions shall be open to the public
in the public interest due to some private, pecuniary,  UNLESS :
or other personal considerations that may tend to 1. A closed door session is ordered
affect his judgment to the prejudice of the service or a. By an affirmative vote of majority of
public members present, there being a quorum
b. In the public interest
(b) Disclosure required shall be: 2. Or for reasons of security, decency or morality
1. In writing, and  NO 2 sessions (regular or special) may be held in ONE
2. Submitted to the secretary of sanggunian or of the day
committee of which he is a member
(d) In case of special sessions of sanggunian:
 Disclosure shall form part of the record of the a. Written notice to members shall be served personally
proceedings and shall be made in the ff manner:  At the member’s usual place of business
a. Disclosure shall be made before member  At least 24 hours before
participates in the deliberation on the ordinance b. No other matters may be considered at a special
or resolution under consideration session except those stated in the notice
(1) If he did not participate in deliberations,  UNLESS otherwise concurred by 2/3 vote of
disclosure shall be made before voting on sanggunian members present, there being a
the ordinance or resolution on 2nd and 3rd quorum
readings
b. Disclosure shall be made when a member takes a (e) Each sanggunian shall keep a journal and record of its
position or makes a privilege speech on a matter proceedings
that may affect business interest, financial  May be published upon resolution of sanggunian
connection, or professional relationship
The Sessions
Ratio: To avoid conflict between public duty and private interests which 1. Regular sessions
may prejudice public service a. Day, time, place fixed on first session after election
b. Minimum required number of sessions
Time of Disclosure (1) Once a week for sanggunians of the province, city,
1. Upon assumption of office, FULL disclosure of his OWN municipality
business or financial interests (2) Twice a month for the barangay
 This is an absolute requirement 2. Special sessions
2. Disclosure of interests of his 4th degree RELATIVES in a. May be called by the local chief executive or majority of
business, finance, or in the profession before he discharges sanggunian members
his duties as when he: b. Called in the name of public interest
a. Participates in the discussion of an ordinance or c. Written notices required
resolution that may affect the interest, or d. Only agenda in the notice is to be taken up
b. Votes on 2nd or 3rd reading of said ordinance, or (1) Unless 2/3 vote of members present, there being
c. Delivers a privilege speech, or quorum, should consider other items
d. Takes a position thereon
General Rule: Sessions are open to the public
Where Filed – in writing with the secretary of the sanggunian Exception: When closed door sessions are ordered
concerned or with the secretary of the committee to which he belong
QUORUM APPROVAL OF ORDINANCES

Breakdown of Sec. 53, LGC: Quorum Breakdown of Sec. 54, LGC: Approval of Ordinances

(a) Majority of members of sanggunian elected and qualified (a) Every ordinance enacted by the Sangguniang Panlalawigan,
shall constitute quorum to transact official business Panlungsod, or Bayan shall be presented to the provincial
 If question of quorum is raised during session, governor or city/municipal mayor
presiding officer shall immediately proceed to call the (1) If local chief executive APPROVES
roll and announce results  He shall affix his signature on each and every
page thereof
(b) Where there is no quorum: (2) Otherwise, he shall VETO it
1. Presiding officer may declare a recess until such time a  And return the same with objections to the
quorum is constituted, or sanggunian
2. Majority of members present may adjourn from day to  Sanggunian may then reconsider the same
day
a. And may compel immediate attendance of any Sanggunian may override the veto by 2/3 vote of all its
member absent without justifiable cause by members; ordinance or resolution will then be
designating a member to be assisted by member effective for all legal intents and purposes
of police force assigned in the territorial
jurisdiction to arrest the absent member and (b) Veto shall be communicated to sanggunian within:
present him in the session 1. 15 days in case of a province
2. 10 days in case of city or municipality
(c) If there is still no quorum despite enforcement of (b)
 No business shall be transacted Otherwise, ordinance deemed approved as if signed
 Presiding officer shall then declare session adjourned
for lack of quorum (c) Ordinances enacted by Sangguniang Barangay shall be
 Upon proper motion duly approved by members signed by the punong barangay
present  Upon approval by majority of all its members

Quorum = the fixed number of members present at a session which is NOTE: Sec. 54 deals only with the approval required of governor for
considered legally sufficient to transact the business of the sanggunian provincial ordinances/resolutions and mayor for city/municipality
- For sanggunian, quorum is majority of members (1/2 of ordinances
members plus 1) - It does not treat the power of approval or disapproval that
the Sangguniang Panlalawigan has over ordinances and
Special Quorum - There are sessions where a special quorum is resolutions of component cities or municipalities or the
required; like: power of the latter over barangay ordinances and resolutions
1. If session is intended to override veto of the local chief  This is provided for in Sec. 56 and 57
executive, quorum required must be not less than 2/3 of all
members since this is the vote needed to override veto Review:
2. Sanggunian may meet with less than a majority when Sec. 53 1. Requisites of a valid ordinance
states that a majority of all members of the Sanggunian who 2. Differences between an ordinance and resolution
have been elected and qualified shall constitute quorum
 This covers situations where less than all members of On Resolutions
the Sanggunian are elected and qualified - Note that resolutions are not mentioned in the 1st sentence as
 Ex: 10 sanggunian members, 5 have been elected and among the acts of sanggunian that need to be approved by
qualified while the other 5 are facing electoral protest the local chief
 3 of the 5 elected and qualified members can meet - Note further that resolutions are mentioned in the 3rd
and hold a valid session sentence which deals with the sanggunian’s power to
 Under these circumstances, 3 out 5 would override the veto
constitute a quorum - In any case, resolutions in practice are submitted for
signature also by the local chief
No Quorum = No Valid Session – any transaction or business conducted
is void ab initio NOT a ministerial act – see De los Reyes v. Sandiganbayan
- The approval of the sanggunian resolution by the local chief
Arresting Members to Secure Quorum is NOT a ministerial act
- Presiding officer of majority of the members present may - The grant of veto power confers authority beyond the simple
adjourn and compel attendance of absent members mechanical act of signing an ordinance or resolution as a
- Done by ordering their arrest by the police force requisite to its enforceability
 Not for failing
 But to compel attendance Manner of Approval
- Approval of an ordinance may be done by:
See the following cases: 1. Governor’s or mayor’s signing of all the pages of an
1. Zamora v. Caballero - Only qualification is material in the ordinance; or
counting of a quorum. The filing of a leave of absence does 2. By inaction of local chief to act on the ordinance:
not affect the member's material qualification or election, a. Within 15 days for a provincial ordinance
hence quorum shall be based on the total number of b. 10 days for a city or municipal ordinance
members without regard to the filing of a leave of absence. - In case of barangay ordinances:
2. Casino v. CA - It is legally permissible, as exceptions to the  Once approved by majority of all members of
general provisions covered by the city charters and the LGC, Sangguniang Barangay, approval of punong barangay is
that the vote requirement be specifically provided for instead not required
of the usual majority vote.  WHY? The punong barangay IS a member of the
Sangguniang Barangay
VETO POWER REVIEW OF ORDINANCES AND RESOLUTIONS

Breakdown of Sec. 55, LGC: Veto Power of the Local Chief Executive Breakdown of Sec. 56, LGC: Review of Component City and Municipal
Ordinances or Resolutions by the Sangguniang Panlalawigan
(a) Local chief executive may veto any ordinance of the
Sangguniang Panlalawigan, Panlungsod, or Bayan on the (a) WITHIN 3 DAYS from approval, Secretary of the
ground that: sangguniang panlungsod or bayan shall forward to
1. It is ultra vires, or sangguniang panlalawigan for review copies of:
2. Prejudicial to the public welfare 1. Ordinances and
2. Resolutions approving local development plans and
Stating his reasons therefor in writing public investment programs formulated by local
development councils
(b) Local chief executive, except the punong barangay:
 Shall have the power to veto: (b) WITHIN 30 days after receipt of copies of such ordinances
1. Any particular item/s of an appropriation and resolutions:
ordinance or 1. Sangguniang Panlalawigan shall:
2. Resolution adopting a local development plan a. Examine the documents or
and public investment program, or b. Transmit them to provincial attorney (or
3. An ordinance directing payment of money or provincial prosecutor if no attorney) for prompt
creating liability examination
 Veto shall NOT affect the item/s not objected to 2. Provincial attorney/prosecutor shall inform the
 Vetoed item/s shall not take effect sangguniang panlalawigan of his comments and
 UNLESS sanggunian overrides the veto recommendations
 Item/s in appropriations ordinance of the a. Within 10days from receipt
previous year corresponding to those vetoed, if b. In writing
any, shall be deemed reenacted c. Recommendations may be considered by the
sangguniang panlalawigan
(c) Local chief executive may veto ONLY ONCE
 Sanggunian may override the veto by 2/3 vote of all its (c) If sangguniang panlalawigan finds that such ordinance or
members resolution is BEYOND the power of the sanggunian:
 Ordinance effective without approval of the local 1. It shall declare such ordinance/resolution invalid
chief executive a. In whole or in part
2. Sangguniang panlalawigan shall enter its action in
NOTE: The punong barangay has no veto power over ordinances of the the minutes
Sangguniang Barangay (remember that he is a member of the a. It shall also advise the corresponding city or
Sanggunian himself) municipal authorities of the action it has taken

Grounds for Veto: only 2 grounds are specified (d) If no action has been taken by the Sangguniang
1. Ultra vires Panlalawigan within 30 days after submission:
2. Prejudicial to the public welfare 1. Ordinance/resolution shall be:
a. Presumed consistent with the law, and
Scope of Veto: b. Therefore valid
1. Entire ordinances
2. Particular items only in: Power of Review
a. Appropriation ordinances - Note that it is the sangguniang panlalawigan and not the
b. Ordinances adopting local government plans and public governor who has the power to review component city and
investment programs municipal:
c. Ordinances directing payment of money or creating 1. Ordinances of ALL KINDS, and
liability against the LGU 2. Resolutions dealing with local development plans and
public investment programs formulated by local
NOTE: Sec. 55 allows local chief executive to veto development councils
ordinances AND resolutions; and only ONCE
Take note of the periods
Effect of Veto 1. Submit the ordinances and resolutions within 3 days from
1. If entire ordinance vetoed approval
a. Unenforceable for all intents and purposes 2. Upon receipt, sangguniang panlalawigan must act on them
2. If particular items only within 30 days
a. Only vetoed items are ineffectual a. Approval may come in 2 ways:
b. Item in the appropriations ordinance of the previous (1) Formally, by passing a resolution of approval
year corresponding to those vetoed, if any, deemed (2) Informally, by its inaction within 30 days
reenacted b. Note also that the sangguniang panlalawigan must
communicate to the component city/municipalities its
Duty of Local Chief Executive After Veto: Remember that the local chief action within the same 30 day period
must inform the Sanggunian of the veto within:
1. 15 days for provincial ordinances Ground to Invalidate Ordinance or Resolution: Sec. 56 mentions the sole
2. 10 days for city or municipal ordinances ground: That the ordinance/resolution in whole or in part are found to
be outside the powers of the sanggunian
Overriding of Veto
- Sanggunian may override by a 2/3 vote of all members
- Effect of overriding: Ordinance effective as if approved and
signed by local chief
 Once overridden, it cannot be “re-vetoed”
Breakdown of Sec. 57, LGC: Review of Barangay Ordinances by the WHAT HAPPENS AFTER REJECTION
Sangguniang Panlungsod or Sangguniang Bayan It shall declare such
Sanggunian concerned shall
ordinance/resolution invalid In
return the same with its
(a) WITHIN 10 days after its enactment, sangguniang whole or in part
comments and recommendations
barangay shall furnish copies of ALL barangay ordinances
to the sangguniang barangay
to the sangguniang panlungsod or bayan for review Sangguniang panlalawigan shall
concerned For adjustment,
 As to whether the ordinance is consistent with law and enter its action in the minutes
amendment or modification
city or municipal ordinances
It shall also advise the
Effectivity of barangay ordinance
(b) If sangguniang panlungsod or bayan fails to take action corresponding city or municipal
suspended until such time as
within 30 days from receipt authorities of the action it has
revision is effected
 Barangay ordinance shall be deemed approved taken

(c) If the sangguniang panlungsod or bayan finds the barangay


ordinances inconsistent with law or city or municipal ENFORCEMENT AND EFFECTIVITY
ordinances: OF ORDINANCES AND RESOLUTIONS
a. Sanggunian concerned shall return the same with its
comments and recommendations to the sangguniang Breakdown of Sec. 58, LGC: Enforcement of Disapproved Ordinances
barangay concerned or Resolutions
 Within 30 days from receipt
b. For adjustment, amendment or modification Any attempt to enforce any ordinance or resolution approving local
 Effectivity of barangay ordinance suspended development plan and public investment program, AFTER disapproval
until such time as revision is effected - Sufficient ground for the suspension or dismissal of official
or employee concerned
Review of Barangay Ordinances – they are subject to review by the
sangguniang panlungsod or bayan of city/municipality to which it NOTE:
belongs - Sec. 58 does not distinguish between an elective or
appointive official so both are covered
Approval of Ordinances: the sanggunian concerned may approve the - This is an additional ground for disciplinary action against
barangay ordinances if these are consistent with law or with city or erring LGU officials or employees
municipal ordinances
- Approval may come in 2 ways: Breakdown of Sec. 59, LGC: Effectivity of Ordinances and Resolutions
1. Formally in the form of a city or municipal resolution
2. Informally, by inaction within 30 days from submission (a) Ordinance or resolution approving local development plan
by the sanggunian concerned and public investment program shall take effect
 AFTER 10 days from the date a copy thereof is posted
Disapproval of Ordinances in a bulletin board at the entrance of provincial
- Sanggunian concerned may disapprove on the ground of capitol or city, municipal, or barangay hall
inconsistency with law or with city/municipal ordinances  And in at least 2 other conspicuous places in the
- Sanggunian shall return the ordinances with its comments LGU concerned
and recommendations to the barangay for adjustment,  UNLESS otherwise stated in the ordinance or
amendment, or modification resolution
- Until revisions are made, effectivity SUSPENDED
(b) Secretary of sanggunian concerned shall cause the posting
REVIEW OF SEC. 56 AND 57: of an ordinance or resolution:
 In the bulletin board at entrance of provincial capitol,
Ordinances by Sanggunian of and city, municipal, or barangay hall
Ordinances by Sangguniang
Component Cities and  In at least 2 conspicuous places in the LGU
Barangay
Municipalities  NOT later than 5 days after approval
BY WHOM
Sangguniang Panlungsod or Text of ordinance or resolution:
Sangguning Panlalawigan
Sangguniang Bayan 1. Shall be:
WHEN TO SUBMIT FOR REVIEW a. Disseminated, and
Within 3 days from approval, b. Posted in Filipino or English
Within 10 days from approval,
forwarded by secretary of (1) AND in the language or dialect understood
forwarded by Sanggunian
sanggunian by the majority of the people in the LGU
WHAT 2. Secretary to the sanggunian shall record such fact in a
Ordinances of ALL kinds book kept for the purpose
Resolutions on local a. Stating dates of approval and positing
ALL barangay ordinances
development plans and public
investment programs (c) Gist of all ordinances with penal sanctions shall be
PERIOD OF REVIEW published in a newspaper of general circulation within the
Within 30 days from receipt Within 30 days from receipt province where the LGU body belongs
APPROVAL 1. In the absence of any newspaper of general circulation
Formally, by issuing a resolution Formally, by issuing a resolution within the province, posting shall be done in all
of approval of approval municipalities and cities of the province where
Informally, by inaction after 30 Informally by inaction after 30 sanggunian of origin is situated
days from receipt days from receipt
(d) In case of highly urbanized and independent component
GROUNDS FOR REJECTION
cities
If beyond the power conferred Whether consistent with the law
 Main features of ordinance of ordinance/resolution
upon the Sanggunian concerned and city/municipal ordinances
duly enacted or adopted shall be published once in a
local newspaper of general circulation within the city CASINO v. CA
 In absence thereof, publish in a newspaper of December 2, 1991
general circulation
 This is in ADDITION to being posted Summary: Robinson Casino is a cockpit operator of the Don Romulo
Coliseum. Resolution 49 was passed declaring the area where DR
Other LGC Provisions Coliseum was located as residential. The resolution also provided for a
1. Sec. 188 – Publication of tax ordinances and revenue clause that said that any amendment to the area classification of
measures Resolution 49 would have to be approved by ¾ of the Sangguniang
2. Sec. 511 – Posting and publication of ordinances with penal Panglungsod. Resolution 378 was subsequently passed, it provided that
sanctions the area where DR Coliseum was will be changed from residential to
recreational. Resolution 378 was passed with only a majority of the
SUMMARY OF RULES: Publication and Effectivity of Ordinances votes of Sangguniang Panglunsod. Gallera filed a petition seeking to
(from LGC provisions and IRR) declare Resolution 378 null and void. Casino objected saying that the ¾
vote requirement was ultra vires because it contravened LGC. SC sided
PUBLICATION EFFECTIVITY with Gallera.
GENERAL RULE
Posted in a bulletin board at the Doctrine: It is legally permissible, as exceptions to the general
entrance of the provincial provisions on measures covered by city charters and the Local
capitol or city, municipal, or Government Code, that the vote requirement in certain ordinances may
10 days after posting be specially provided for, as in the case of Section 6.44 of Resolution No.
barangay hall, as the case may
be, and in at least 2 other 49, instead of the usual majority Vote. Although the general law on the
conspicuous places matter requires a mere majority, the higher requisite vote in Resolution
FOR HIGHLY URBANIZED AND INDEPENDENTC OMPONENT CITIES No. 49 shall govern since municipal authorities are in a better position
Main features of the ordinances, to determine the evils sought to be prevented by the inclusion or
in addition to posting, shall be incorporation of particular provisions in enacting a particular statute
published once in a newspaper and, therefore, to pass the appropriate ordinance to attain the main
10 days after posting object of the law.
of local circulation; if none, in a
newspaper of general
circulation; Class Notes:
ORDINANCE OR - Remember, to pass an ordinance or resolution, Sec. 107(g) of
RESOLUTION ON LOCAL DEVELOPMENT PLAN AND the LGC IRR states that there must be a majority vote of those
PUBLIC INVESTMENT PROGRAMS present constituting a quorum
- Sir takes issue on the sanctioning of the ¾ requirement since
Follow that stated in the
it, in effect, gives rise to an irrepealable law
Follow the general rule ordinance or resolution; if none,
follow general rule;  Not really irrepealable, but it gives conditions before it
can be repealed
ALL ORDINANCES WITH PENAL SANCTIONS
Posted at conspicuous places in
the provincial capitol, or
MALONZO v. ZAMORA
city/municipal or barangay hall
July 27, 1999
for a minimum of 3 consecutive
weeks
Summary: Ordinance No. 0168 was passed by the
AND
Take effect on the day following SangguniangPanlungsod of Caloocan City in 1994. It authorized the
Publication in a newspaper of
its publication or at the end of the expropriation of Lot 26 of the Maysilo Estate owned by the CLT Realty
general circulation
period of posting, whichever is Development. It turned out that the lot was subject of a boundary
EXCEPT in the case of barangay
higher dispute between Caloocan and Malabon. In 1997, Ordinance No. 0246
ordinances
was enacted, increasing the appropriated amount. In 1998, Vice-Mayor
AND
UNLESS otherwise provided Malapitan requested the immediate repair of the offices of the
Gist of such ordinance shall be
therein councilors, as well as the hiring and retention of several personnel. In a
published in a newspaper of
memorandum endorsed by the Mayor, the City Treasurer manifested
general circulation within the
that since the expropriation of Lot 26 was discontinued, the
province
appropriation for this worth P50M can be reverted for use in a
If none, posting in all
supplemental budget. Pursuant to this certification, Ordinance No. 0254
municipalities and cities of the
was passed providing for payments for items in the supplemental
province
budget, the amounts of which shall be taken from the general fund from
TAX ORDINANCES
the reversion of the funds for expropriation. Eduardo Tidor by himself
Within 10 days after their and as taxpayer filed an administrative complaint in the Office of the
approval, published in full for 3 Beginning of the quarter, if President for Dishonesty, Misconduct in Office and Abuse of Authority
consecutive days in a newspaper otherwise, the same shall be against Mayor Malonzo, Vice-Mayor Malapitan, and members of the
of local circulation; if none. the considered as falling at the SangguniangPanlungsod of Caloocan City. One of the allegations was
same may be posted in at least 2 beginning of the next ensuing that Ordinance No. 0254 was passed without complying with Sections
conspicuous and publicly quarter 50 and 52 of the LGC requiring that on the first regular session
accessible places following the election of its members and within 90 days thereafter, the
Sanggunian concerned shall adopt or update its existing rules of
procedure. In their defense, Malonzo et. al. said that the matter of
adoption of the house rules was taken up and that the council arrived at
a decision to create an ad hoc committee to study the rules, and that
even if they failed to approve the new rules, the rules which were
applied in the previous year shall be deemed in force until new ones are
adopted. The OP ruled against Malonzo et. al. and ordered their
suspension. The SC reversed and said that there has been sufficient
compliance with the LGC.
Doctrine: There is nothing in the language of Section 50 and 52 of the A sanggunian is a collegial body. Legislation, which is the principal
LGC that restricts the matters to be taken up during the first regular function and duty of the sanggunian, requires the participation of all its
session merely to the adoption or updating of the house rules. members so that they may not only represent the interests of their
Moreover, adoption or updating of house rules would necessarily entail respective constituents but also help in the making of decisions by
work beyond the day of the first regular session. voting upon every question put upon the body.

MALONZO v. ZAMORA DE LOS REYES v. SANDIGANBAYAN


January 28, 2000 November 13, 1997

Summary: [Summary from the 1997 case]During the incumbency of Summary: Mayor de los Reyes was charged with falsification of a public
Mayor Asistio, an ordinance was passed authorizing the the City Mayor document, specifically a resolution. Prior to his arraignment, he filed a
to initiate expropriation proeceedings for Lot 26 of the Maysilo estate. motion for reinvestigation which was denied by the Sandiganbayan.
Said ordinance appropriated P35,997, 975.00 for this purpose. The SC sustained the Sandiganbayan. On the issue of the
However, as the Maysilo Estate straddled Caloocan and Malabon, the Sandiganbayan’s reliance upon the minutes taken during the
owner of such, CLT, filed an action for interpleader. In the meantime, Sanggunian’s session, the SC said that the Sandiganbayan was correct in
the sanggunian of Caloocan enacted another ordinance which increased relying upon such. The minutes show that the mayor approved the
the appropriated amount for the expropriation of the subject lot. After subject resolution without the subject matter thereof being taken up or
the re-election of Mayor Malonzo, the Vice Mayor requested for the passed upon by the Sanggunian during its legislative session.
repair and renovation of offices of the incoming councilors. This request
was subsequently endorsed to the Sanggunian Supplemental Budget Doctrine: [Implied] A resolution may not be passed without the subject
No. 01. The City Council acted favorably on the endorsement, passing matter treated therein being taken up and discussed or passed upon by
the assailed ordinance. An administrative complaint against Malonzo the Sangguniang Bayan during the legislative session
et. al. was subsequently filed. The OP found them guilty and were
suspended from office. The SC reversed. As regards the argument that The Court accords full recognition to the minutes as the official
Malonzo et. al. violated Secs. 50 and 52 of the LGC which pertained to repository of what actually transpires in every proceeding. It has
the adoption of house rules and the organization of the council, the SC happened that the minutes may be corrected to reflect the true account
said that nothing in the rules restrict the matters to be taken up during of a proceeding, thus giving the Court more reason to accord them great
the first regular session merely to the adoption or updating of the house weight for such subsequent corrections, if any, are made precisely to
rules. preserve the accuracy of the records.

[Summary from the 2000 case] The OSG filed a motion for
reconsideration. The SC denied, reiterating its earlier position in the HAGONOY MARKET VENDOR ASSOC. v. MUN. OF HAGONOY
1997 Malonzo case. February 6, 2002

Doctrine: The law does not require the completion of the updating or Summary: The Municipality of Hagonoy enacted an Ordinance
adoption of the internal rules of procedure before the Sanggunian could increasing the stall rentals if market vendors. One year after the
act on any other matter like the enactment of an ordinance. It simply effectivity of the ordinance, the Hagonoy Marjet Vendor Association
requires that the matter of adopting or updating the internal rules of filed an appeal assailing the constitutionality of the ordinance. The
procedure be taken up during the first day of session. Court dismissed the petition since the appeal was filed beyond the 30-
day period provided in the LGC. Posting was also validly made in lieu of
publication since there was no newspaper of local circulation in the
ZAMORA v. CABALLERO municipality.
January 14. 2003
Doctrine: The periods stated in Section 187 of the Local Government
Summary: The Sangguniang panlalawigan of compostela valley had its Code are mandatory. Ordinance No. 28 is a revenue measure adopted
regular session with only 7 members present. During the session, 2 by the municipality of Hagonoy to fix and collect public market stall
resolutions were passed and the resignation letter of one member was rentals. Being its lifeblood, collection of revenues by the government is
noted. Member Zamora filed a petition before the RTC to nullify the said of paramount importance. The funds for the operation of its agencies
acts. The SC nullified the acts primarily because there was no quorum and provision of basic services to its inhabitants are largely derived
during the session and that the number of votes required for the from its revenues and collections. Thus, it is essential that the validity
validity of the said acts were not met. of revenue measures is not left uncertain for a considerable length of
time.
Doctrine: The entire membership must be taken into account in
computing the quorum of the sangguniang panlalawigan, for while the Posting was validly made in lieu of publication as there was no
constitution merely states that “majority of each House shall constitute newspaper of local circulation in the municipality of Hagonoy.
a quorum,” Section 53 of the LGC is more exacting as it requires that the
“majority of all members of the sanggunian . . . elected and qualified”
shall constitute a quorum. CITY OF MANILA v. LAGUIO, JR.
April 12, 2005
“Quorum” is defined as that number of members of a body which, when
legally assembled in their proper places, will enable the body to Summary: Malate Tourist Development Corporation petitioned against
transact its proper business or that number which makes a lawful body the Ordinance issued by the City of Manila which tries to curtail
and gives it power to pass upon a law or ordinance or do any valid act. activities in Ermita Manila. , MTDC argued that
the Ordinance erroneously and improperly included in its enumeration
“Majority,” when required to constitute a quorum, means the number of prohibited establishments, motels and inns such as MTDC’s Victoria
greater than half or more than half of any total. Court and contested the constitutionality of such ordinance.
Respondent city of Manila protests the ruling of the RTC by Judge
The qualification in the LGC that the majority be based on those Laguio which states that it was unconstitutional. SC upheld RTC.
“elected and qualified” was meant to allow sanggunians to function
even when not all members thereof have been proclaimed. Doctrine: The tests of a valid ordinance are well established. A long line
of decisions has held that for an ordinance to be valid, it must not only
be within the corporate powers of the local government unit to enact purposes of the measure and the means employed for its
and must be passed according to the procedure prescribed by law, it accomplishment,
must also conform to the following substantive requirements:
(1) must not contravene the Constitution or any statute; Class Notes: Lesson from White Light and City of Manila
(2) must not be unfair or oppressive; - Local legislation may also be struck down on substantial
(3) must not be partial or discriminatory; grounds
(4) must not prohibit but may regulate trade; - So, to strike down a local legislation, one can cite:
(5) must be general and consistent with public policy; and 1. Its non-compliance with procedure and requirements of
(6) must not be unreasonable. law
2. On substantial grounds like constitutionality
To successfully invoke the exercise of police power as the rationale for
the enactment of the Ordinance, and to free it from the imputation of
constitutional infirmity, not only must it appear that the interests of the MODAY v. CA
public generally, as distinguished from those of a particular class, February 20, 1997
require an interference with private rights, but the means adopted must
be reasonably necessary for the accomplishment of the purpose and not Summary: The Sangguniang Bayan of the Municipality of Bunawan in
unduly oppressive upon individuals. Agusan del Sur passed a Resolution authorizing the Municipal Mayor to
Initiate a Petition for Expropriation. It was approved by the Mayor and
It is an ordinance which permanently restricts the use of property that transmitted to the Sangguniang Panlalawigan for approval. The
it can not be used for any reasonable purpose goes beyond regulation Sangguniang Panlalawigan disapproved the Resolution. The
and must be recognized as a taking of the property without just Municipality went ahead with the expropriation. The issue was brought
compensation. to the RTC, which upheld the municipality’s power to expropriate. The
CA affirmed. The SC also affirmed, ruling that the Sangguniang
There are two different types of taking that can be identified. A Panlalawigan’s power to declare a municipal resolution invalid is on the
“possessory” taking occurs when the government confiscates or sole ground that it is beyond the power of the Sangguniang Bayan or
physically occupies property. A “regulatory” taking occurs when the the Mayor to issue such resolution. The exercise of eminent domain is
government’s regulation leaves no reasonable economically viable use within the authority of the municipality.
of the property.
Doctrine: The law grants the Sangguniang Panlalawigan the power to
What is crucial in judicial consideration of regulatory takings is that declare a municipal resolution invalid on the sole ground that it is
government regulation is a taking if it leaves no reasonable beyond the power of the Sangguniang Bayan or the Mayor to issue. A
economically viable use of property in a manner that interferes with Municipality’s power to exercise the right of eminent domain is
reasonable expectations for use. When the owner of real property has expressly provided for in Batas Pambansa Blg. 337, (the LGC in force at
been called upon to sacrifice all economically beneficial uses in the the time expropriation proceedings were initiated)- Sec. 9. Eminent
name of the common good, that is, to leave his property economically Domain. — A local government unit may, through its head and acting
idle, he has suffered a taking. pursuant to a resolution of its sanggunian, exercise the right of eminent
domain and institute condemnation proceedings for public use or
purpose.
WHITE LIGHT CORPORATION v. CITY OF MANILA
January 20, 2009 Class Notes: Good law?
- YES! Same rule is found in the LGC
Summary: Manila City Ordinance No. 7774 prohibits two specific and
distinct business practices, namely wash rate admissions and
renting out a room more than twice a day. Petitioners who are motel VICENCIO v. VILLAR
operators argued that the Ordinance is unconstitutional and void since July 3, 2010
it violates the right to privacy and the freedom of movement; it is an
invalid exercise of police power; and it is an unreasonable and Summary: VM Vicencio entered into Contracts for Consultancy Services
oppressive interference in their business. RTC declared the Ordinance with three individuals, on the basis of an ordinance previously enacted
null and void. CA reversed the decision of the RTC and affirmed the by the City Sanggu. The ordinance granted the previous VM the
constitutionality of the Ordinance. SC reversed CA ruling and declared authority to enter into similar contracts. COA disallowed disbursement
the ordinance as null and void because it needlessly restrains the of funds for the contract. SC ruled that disallowance was proper; VM
operation of the businesses of the petitioners as well as restricting the Vicencio had no authority to enter into such contracts.
rights of their patrons without sufficient justification. There is no
reasonable relation between the objective and means. Doctrine: Under Sec. 456, LGC, there is no inherent authority on the part
of the city VM to enter into contracts on behalf of the LGU, unlike that
Doctrine: The test of a valid ordinance is well established. For an provided for the city mayor. Thus, the authority of the VM to enter into
ordinance to be valid, it must not only be within the corporate powers contracts on behalf of the city is strictly circumscribed by the ordinance
of the local government unit to enact and pass according to the granting it.
procedure prescribed by law, it must also conform to the following
substantive requirements: (1) must not contravene the Constitution or An old ordinance granting the VM to enter into certain contracts cannot
any statute; (2) must not be unfair or oppressive; (3) must not be be construed as a “continuing authority” for any person who enters the
partial or discriminatory; (4) must not prohibit but may regulate trade; Office of the VM, to enter into subsequent, albeit similar, contracts.
(5) must be general and consistent with public policy; and (6) must not
be unreasonable Class Notes:
- Note further: Sec. 103, PD 1445 declares expenditures of
It must appear that the interests of the public generally, as government funds or uses of government property in
distinguished from those of a particular class, require an interference violation of law or regulations shall be a personal liability of
with private rights and the means must be reasonably necessary for the the official or employee found to be directly responsible
accomplishment of the purpose and not unduly oppressive of private therefor
rights. It must also be evident that no other alternative for the  Here, Vicencio’s act of entering into these contracts was
accomplishment of the purpose less intrusive of private rights can in violation of the LGC since there was no requisite
work. More importantly, a reasonable relation must exist between the authority
 While he relied on the CLO in good faith, this does not
exculpate him from personal liabilities
- Note also: If the reviewing body does not act on the petition
timely (within the period granted by law)
 Enactment is deemed effective
Notice of Hearing (Sec. 62)
DISCIPLINARY ACTION - Within 7 days after the complaint is filed, the Office of the
President or the Sanggunian, as the case may be, shall
require respondent to submit verified answer
GROUNDS FOR DISCIPLINARY ACTION  Answer to be filed within 15 days from receipt of order
- Investigation to commence within 10 days after receipt of
Breakdown of Sec. 58, LGC: Enforcement of Disapproved Ordinances such answer
or Resolutions
NOTE: Verified answer means that the answer must contain a
Any attempt to enforce any ordinance or resolution approving local statement at the end thereof that:
development plan and public investment program, AFTER disapproval (1) Person answering is the very person who is being
- Sufficient ground for the suspension or dismissal of official investigated
or employee concerned (2) Answer is prepared by him or at his instance
(3) He has read the answer
Breakdown of Sec. 60, LGC: Grounds for Disciplinary Actions – (4) Statements made therein are true to his own knowledge

LOCAL ELECTIVE official may be disciplines, suspended, or removed Venue of Hearing and Investigation
from office: - Depends:
1. If respondent is elective official f province or HUC,
1. Disloyalty to the Republic hearing and investigation is conducted in the place
2. Culpable violation of the Constitution where he renders office
3. Dishonesty, oppression, misconduct in office, gross 2. For all other elective officials, place where sanggunian
negligence, or dereliction of duty concerned is located
4. Commission of any offense involving moral turpitude or an - Limitations:
offense punishable by at least prision mayor  Within 90 days immediately prior to any election:
5. Abuse of authority a. No investigation shall be held
6. Unauthorized absence for 15 consecutive working days b. No preventive suspension shall be imposed
a. EXCEPT: in case of members of the various  If preventive suspension has been imposed prior to the
Sanggunians 90-day period:
7. Application for, or acquisition of, foreign citizenship or  It shall be deemed automatically lifted upon the
residence or status of immigrant of another country start of such period
8. Such other grounds as may be provided by LGC and other
laws
PREVENTIVE SUSPENSION
An elective official may be removed from office on the grounds From Sec. 63 LGC
enumerated above BY ORDER of the proper court
Nature of Preventive Suspension – not a penalty but is merely
The enumeration is NOT EXCLUSIVE. preventive
- Art. 125 IRR adds that an elective local official may be - A means taken to insure the proper and impartial conduct of
disciplined on such other grounds provided by: investigation
1. The LGC - May be ordered even before charges as heard, or before
2. RA 6713 (Code of Conduct) official concerned is given opportunity to prove his
3. Admin Code of 1987 innocence
4. RA 3019
5. RPC Who may impose? Preventive suspension may be imposed by:
6. Other applicable general and special laws
IF RESPONDENT IS
SUSPENDING AUTHORITY
Administrative Sanctions AN ELECTIVE OFFICIAL OF
1. Censure Province
2. Suspension President1 HUC (Highly Urbanized City)
3. Forfeiture of salaries, etc. ICC (Independent Component City)
4. Removal = NOTE that this must be by proper court order CC (Component City)
Governor
Municipality
Mayor Barangay
COMPLAINTS AND HEARINGS
From Sec. 61 and 62, LGC When may it be imposed?
- It may be imposed at ANY TIME when the following
Form and Filing of Administrative Complaint (Sec. 61) conditions are present:
1. Complaint must be verified 1. After the issues are joined2
2. For place of filing, see table below: 2. Evidence of guilt is strong
3. Given the gravity of the offense, there is great
COMPLAINT probability that the continuance of office of the
WHERE TO FILE APPEAL?
AGAINST respondent could:
Elective official of a a. Influence the witnesses or
Office of the
province, HUCC, ICC
President
or CC 1
Decision appealable Pimentel believes that if the DILG Secretary suspends elective local
Elective official of a Sangguniang
to the Office of the authorities, he has to do so in behalf of the Office of the President and not
municipality Panlalawigan
President in his capacity as DILG Secretary
Sangguniang 2
“Issues are joined” means that the complaint has been answered and
Elective official of a Decision shall be final
Panlungsod or Bayan there are no longer any substantial preliminary issue that remains to be
barangay and executory
concerned threshed out
b. Pose threat to the safety and integrity of the - Upon exoneration and reinstatement, he shall be paid full
records and other evidence salary or compensation including such emoluments accruing
during such suspension
Period of Suspension:
1. Any single preventive suspension shall NOT exceed 60 days Rights of Respondents
2. In the event that several administrative cases are filed 1. Full opportunity to appear and defend himself in person or
against an elective official: by counsel
 He CANNOT be preventively suspended for more than 2. To confront and cross-examine the witnesses against him
90 days: 3. To require attendance of witnesses and production of
a. Within a single year, AND documentary evidence in his favor through compulsory
b. On the same grounds existing and known at the process of subpoenas
time of the first suspension

NOTE: During class discussion, the 90-day suspension limit vis-à-vis 60- DECISION AND PENALTY
day suspension came up as a contentious area. Sir gave us his take on From Sec. 66 LGC
the issue as follows:
Investigation – Shall be terminated within 90 days from the start
Premise: thereof
1. A 60-day suspension has already been imposed
2. We are now confronted with the issue of whether we can Decision
impose another period of suspension - When? Rendered within 30 days after end of investigation
3. Remember the conditions in Sec. 63(b) - Who? Office of President or Sanggunian shall render
4. NOTE: “NO” means No Constraints - Form of decision
1. In writing
a. Stating clearly and distinctly the facts and reasons
for such decision
2. Copies shall immediately be furnished to respondent
and all interested parties

Limitations on Penalty of Suspension3


1. Shall not exceed the unexpired term of respondent
2. Shall not exceed a period of 6 months for every
administrative offense
3. Shall not be a bar to the candidacy of respondent so
suspended as long as he meets the qualifications required for
the office
a. BUT the penalty of REMOVAL as a result of
administrative investigation shall be considered a BAR
to the candidacy of respondent for any elective position

ADMINISTRATIVE APPEALS AND EXECUTION PENDING APPEAL


From Sec. 67 and 68

Appeals in Administrative Cases (Sec. 67)


1. Period of appeal = 30 DAYS from receipt thereof
2. Where to appeal? See table below

FOR DECISIONS OF APPEAL TO


Sangguniang Panlungsod of
Effect of Expiration of Period - Upon expiration, elective official deemed Component Cities Sangguniang Panlalawigan
reinstated in office without prejudice to continuation of proceedings Sangguniang Bayan
Sangguniang Panlalawigan
Period of proceedings Sangguniang Panlungsod of Office of the President
- Proceedings shall be terminated within 120 days from the HUCs and ICCs
time official was formally notified of the case against him
- IF the delay is due to official’s fault, neglect, or request, other NOTE: Decisions of the Office of the President shall be final and
than appeal duly filed: executory
 Duration of such delay shall not be counted in - See also the case of Calingin v. CA where the court said that
computing the time of termination of the case decisions of the Office of the President are final and
executory, meaning no MR is allowed
Abuse of Authority – Any abuse of the exercise of the power of - BUT the parties may appeal the decision to the CA
preventive suspension shall be penalized as abuse of authority  However, the appeal does not stay the execution of the
decision
 See, however, Berces v. Guingona on Sec. 68
RIGHTS OF RESPONDENT
From Sec. 64 and 65

Salary Pending Suspension


- Respondent shall NOT receive any salary or compensation
during the period of suspension 3
NOTE: This is a penalty of SUSPENSION imposed AFTER investigation. Sec.
63 does not treat of preventive suspensions anymore
Execution Pending Appeal (Sec. 68) acquires jurisdiction to the exclusion of the
- Appeal SHALL NOT prevent a decision from becoming final other
or executory (b) This shall be considered as a bar to the
- Respondent considered to be under preventive suspension candidacy of the respondent for any elective
during pendency of appeal in the event he wins such appeal position
- In the event appeal results in exoneration
 He shall be paid his salary and such other emoluments
during the pendency of the appeal OFFICE OF THE OMBUDSMAN v. RODRIGUEZ
July 23, 2010
NOTE: Sec. 68 says that “An appeal SHALL NOT prevent a decision from
becoming final or executory” Summary: On Aug 26, 2003, a case was filed in the Ombudsman against
- However, see the case of Berces v. Guingona which says that Brgy Captain Rolson Rodriguez for abuse of authority, dishonesty,
this phrase is NOT mandatory oppression, misconduct in office, and neglect of duty. On Sept 1, 2003, a
 Prof. Gatmaytan does not agree with this but this is how similar case was filed with the SB of Binalbagan. Rodriguez was served
the Court applied Sec. 68 with notice of the complaint on Sept 8 by the SB, and on Sept 10 by the
Ombudsman. He filed a motion to dismiss the Ombudsman case on
SPECIAL LAWS cited in the Syllabus: ground of forum shopping and motion to dismiss the SB case alleging
1. RA 6770 – OMB Act (see compilation) that it has no factual basis. The complainants subsequently withdraw
2. Admin Order No. 23 (1992) as amended by Admin Order 159 the complaint with the SB to prioritize the Ombudsman case.
(1994) and Admin Order No. 66 (1999) – Rules and Ombudsman found Rodriguez guilty of dishonesty and oppression. CA
Procedures on Investigation of Admin Disciplinary Cases set aside such ruling for lack of jurisdiction. CA ruled that SB acquired
against Local Elective Officials primary jurisdiction over Rodriguez because it served the notice 2 days
a. Disciplining Authority: The President, who may act earlier than the Ombudsman. SC reversed the CA ruling. SC ruled that
through the Executive Secretary the complaint was first filed with the Ombudsman which opts to take
b. Investigating Authority: The DILG Secretary who may cognizance of the case and thus acquires jurisdiction to the exclusion of
create an Investigating Committee other courts with concurrent jurisdiction. Also, the rule against forum
c. Grounds for disciplining – same as LGC shopping applied only to judicial cases or proceedings, not to
d. Complaint: administrative cases.
(1) Who may initiate:
(a) Any private individual or Doctrine: The primary jurisdiction of the Ombudsman to investigate any
(b) Any government officer or employee act or omission of a public officer or employee applies only in cases
(c) Motu proprio by the Office of the President cognizable by the Sandiganbayan. In cases cognizable by regular courts,
or any government agency duly authorized the Ombudsman has concurrent jurisdiction with other investigative
by law to ensure that LGUs act within their agencies of government. Republic Act No. 8249, otherwise known as An
powers Act Further Defining the Jurisdiction of the Sandiganbayan, limits the
(2) Form cases that are cognizable by the Sandiganbayan to public officials
(a) Accompanied by affidavits of witnesses or occupying positions corresponding to salary grade 27 and higher.
evidence
e. Preliminary Investigation In administrative cases involving the concurrent jurisdiction of two or
(1) Commenced within 20 days from receipt of more disciplining authorities, the body in which the complaint is filed
Answer first, and which opts to take cognizance of the case, acquires
(2) Determination of probable cause within 15 days jurisdiction to the exclusion of other tribunals exercising concurrent
from receipt of answer jurisdiction.
(a) If none, dismiss motu proprio
(b) If there is, preliminary conference and formal
admin proceedings are commenced ESPIRITU v. MELGAR
(3) No preliminary investigation shall be conducted February 13, 1992
within 90 days immediately prior to any local
election Summary: Mayor Melgar allegedly attacked one Ramir Garing and had
f. Preventive Suspension – same as LGC rules him arrested and detained in abuse of his position as Mayor in Oriental
g. Formal Investigation Mindoro. Garing filed a complaint and so the Governor placed the
(1) Procedural due process observed Mayor under preventive suspension pending investigation of the
(2) Hearing conducted by the investigating authority administrative complaint. Mayor Melgar asked the RTC to issue a TRO
h. Decision on the Provincial Governor and alleged that the Provincial Governor
(1) Rendered within 30 days after receipt of the committed grave abuse of discretion. The RTC granted the TRO. The
report of the investigating authority and Governor appealed to the SC. The SC ruled that the court had no
transmittal of records jurisdiction to interfere and that the governor was empowered to
(2) Decision of disciplining authority shall preventively suspend the mayor.
immediately be final and executory upon receipt
of a copy thereof by complainant or respondent Doctrine: There is nothing improper in suspending an officer before the
(3) MFR shall not stay execution charges against him are heard and before he is given an opportunity to
i. Penalties prove his innocence. Preventive suspension is allowed so that the
(1) Suspension shall not exceed the unexpired term of respondent may not hamper the normal course of the investigation
the respondent or a period of 6 months for every through the use of his influence and authority over possible witnesses.
admin offense The office or body that is invested with the power of removal or
(a) If meted by 2 or more penalties for suspension should be the sole judge of the necessity and sufficiency of
suspension for 2 or more admin offenses, the cause. So, unless a flagrant abuse of the exercise of that power is
such penalties shall be served successively shown, public policy and a becoming regard for the principle of
(2) Removal separation of powers demand that the action of said officer or body
(a) Must be by order of proper court OR the should be left undisturbed.
Disciplining Authority whichever first
FLORES v. SANGGUNIANG PANLALAWIGAN OF PAMPANGA The procedure under the LGC and A.O. No. 23, however, is that when an
February 23, 2005 administrative complaint is filed, the Disciplining Authority shall issue
an order requiring the respondent to submit his verified answer within
Summary: An administrative complaint was filed against Mayor Flores 15 days from notice. And upon filing of the answer, the Disciplining
before the Sanggu of Pampanga. The Sanggu issued on order Authority shall refer the case to the Investigating Authority for
recommending to the Governor the preventive suspension of Flores. investigation.
Flores wrote a letter to Gov. Lapid requesting the veto of the order
without filing an MR before the Sanggu. He also filed a petition for Preventive suspension is authorized under Sec. 63 of the LGC. It may be
certiorari with the CA before waiting for any action from Gov. Lapid. CA imposed by the Disciplining Authority at any time (a) after the issues
dismissed the petition for prematurity (doctrine of exhaustion of are joined; (b) when the evidence of guilt is strong; and (c) given the
administrative remedies). SC affirmed. It ruled that Flores, before filing gravity of the offense, there is great probability that the respondent,
his petition for certiorari with the CA, should have waited for Gov. who continues to hold office, could influence the witnesses or pose a
Lapid’s action, in consideration of the fact that it is Gov. Lapid who is threat to the safety and integrity of the records and other evidence.
empowered to impose preventive suspension on Flores under Sec. 63,
LGC. Right to a formal investigation of is not satisfied when the complaint is
decided on the basis of position papers. There is nothing in the LGC and
Doctrine: Sec. 63, LGC provides that preventive suspension may be its IRR nor in A.O. No. 23 that provide that administrative cases against
imposed by the governor, if the respondent is a elective official of a elective local officials can be decided on the basis of position papers.
municipality (or component city). Resort straight to the courts before A.O. No. 23 states that the Investigating Authority may require the
awaiting action of the governor in such a case, is in effect, deprivation of parties to submit their respective memoranda but this is only after
the Governor’s duty to take appropriate action. formal investigation and hearing. A.O. No. 23 does not authorize the
Investigating Authority to dispense with a hearing especially in cases
involving allegations of fact which are not only in contrast but
JOSON v. EXECUTIVE SECRETARY contradictory to each other. These contradictions are best settled by
May 20, 1998 allowing the examination and cross-examination of witnesses.

Summary: Sangguinang Panlalawigan Board members of Nueva Ecija


filed with the Office of the President a letter-complaint charging Edno
Joson with grave misconduct and abuse of authority. Sec. Barbers BERCES v. GUINGONA
directed the Joson to submit an answer but he failed to submit even February 21, 1995
after several extensions granted by the DILG. Joson then filed a Motion
to Dismiss alleging that the complaint was not verified and that the Summary: Achilles Berces filed two administrative cases against Naomi
DILG has no jurisdiction over the case in July 1997, on recommendation C. Corral, the Mayor of the Municipality of Tiwi, Albay with the
of Sec. Barbers, Exec. Sec. Torres issued an order, by authority of the Sangguniang Panlalawigan of Albay
President, placing Joson under preventive suspension for 60 days
pending investigation of the charges against him. Vice-Governor Tinio The Sanggunian rendered a decision against Corral. He appealed to the
was designated as Acting Governor. Joson filed a Motion to Lift Order of Office of the President praying for the stay of the execution thereof
Preventive Suspension and the order of default. He also filed, among pursuant to Section 67 (b) of the LGC.
others, a Motion to Conduct Formal Investigation pursuant to the
provisions of the LGC and Rule 7 of Administrative Order No. 23. The The Office of the President stayed the execution pursuant to AO 18,
petition for certiorari with the CA was later dismissed. The Motion to which states that “in all cases, at any time during the pendency of the
Conduct Formal Investigation with the DILG was also denied. He filed a appeal, the Office of the President may direct or stay the execution of
Petition for Review of CA decision before SC Court. SC ruled that DILG the decision/resolution/order appealed from.”
Sec. did not err in recommending to the President the preventive
suspension of Joson during the investigation but Resolution finding The Office did not construe this provision as being repealed by Section
petitioner guilty and imposing the 6-month suspension was invalid. 67(b) of the LGC which states that appeals from decisions in
administrative disciplinary cases under the LGC shall not prevent a
Doctrine: Jurisdiction over administrative disciplinary actions against decision from becoming final or executory. The SC upheld the decision
elective local officials is lodged in 2 authorities: the Disciplining and the interpretation of the law of the Office of the President.
Authority and the Investigating Authority. Pursuant to Secs. 2 and 3 of
A.O. No. 23, the Disciplining Authority is the President, whether acting Doctrine: We find that the provisions of Section 68 of R.A. No. 7160 and
by himself or through the Exec. Secretary. The Sec. of the DILG is the Section 6 of Administrative Order No. 18 are not irreconcillably
Investigating Authority, who may act by himself or constitute an inconsistent and repugnant and the two laws must in fact be read
Investigating Committee. He is not, however, the exclusive together.
Investigating Authority for the DILG Sec. may designate a Special
Investigating Committee. The power of the President over The first sentence of Section 68 merely provides that an "appeal shall
administrative disciplinary cases against local officials is derived from not prevent a decision from becoming final or executory."
his power of general supervision over local governments as provided
under Sec. 4, Art. X, 1987 Constitution. As worded, there is ROOM to CONSTRUE said provision as GIVING
DISCRETION to the reviewing officials to stay the execution of the
The President’s power of general supervision means no more than the appealed decision.
power of ensuring that laws are faithfully executed, or that subordinate
officers act within the law. Supervision is not incompatible with Class Notes: We are upset by this case
discipline. This power must be construed to authorize the President to - Sec. 68 clearly says that an appeal SHALL not prevent a
order an investigation of the act or conduct of local officials when in his decision from becoming final or executory
opinion the good of the public service requires.A.O. No. 23 delegates - Yet the SC said
the power to investigate to the DILG or a Special Investigating  “Shall” may be read either as mandatory or directory
Committee, as may be constituted by the Disciplining Authority. This is depending upon a consideration of the entire provision
not undue delegation. The President remains the Disciplining Authority.  Here, there is no basis to justify that the construction of
And the power of the DILG to investigate administrative complaints is the word as mandatory
based on the alter-ego principle or the doctrine of qualified political
agency.
DON v. LACSA officers. Finding grave misconduct, the SC ordered Alejandro’s dismissal
August 7, 2007 from service.

Summary: Public school teachers charged Punong Barangay Ramon Doctrine: Sandiganbayan's jurisdiction extends only to public officials
Lacsa with grave threats, oppression, grave misconduct, and abuse of with salary grade 27 and higher. Any act or omission of a public officer
authority before the Sangguniang Bayan of Juban, Sorsogon. The SB occupying a salary grade lower than 27 is within the concurrent
passed a RESOLUTION peventively suspending Ramon for 2 months. jurisdiction of the Ombudsman and of the regular courts or other
After 2 months, pursuant to the report by the investigating committee, investigative agencies.
the SB passed RESOLUTION NO. 12, finding Lacsa guilty of the charges
and removing him from office. Subsequently, pursuant to said Section 15 of RA 6770 reveals the manifest intent of the lawmakers to
Resolution, Mayor Guab-Frugata issued EO No. 8, implementing the give the Office of the Ombudsman FULL administrative disciplinary
Resolution and installing Florencio, the highest ranking SB member, to authority. This includes, among others, the authority to
Ramon’s position. Ramon received a copy of the EO, together with the - receive complaints
SC RES. 12 on the same day. 21 days after receipt of the EO and RES, - conduct investigations
Ramon filed a certiorari petition before the RTC of Sorsogon. The RTC - hold hearings in accordance with its rules of procedure
nullified the EO and reinstated Ramon [after saying that he was not - summon witnesses and require the production of documents
afforded the opportunity to appeal because of the fact that he received a - place under preventive suspension public officers and
copy of both the EO and RES on the same day]. The SC disagreed with employees pending an investigation
the RTC and held that the SB RESOLUTION No. 12 and the EO are valid. - determine the appropriate penalty imposable on erring
public officers or employees as warranted by the evidence,
Doctrine: The phrase “final and executory” in Sections 67 and 68 of the and,
LGC, are not indicative of the appropriate mode of relief from the - impose the corresponding penalty
decision of the Sanggunian concerned. These phrases simply mean
that the administrative appeals will not prevent the enforcement of the These powers unmistakably grant the Office of the Ombudsman the
decisions. The decision is immediately executory but the respondent power to directly impose administrative sanctions; its power is not
may nevertheless appeal the adverse decision to the OP or to merely recommendatory.
the Sangg. Panlalawigan, as the case may be.
The lawmakers intended to provide the Office of the Ombudsman with
sufficient muscle to ensure that it can effectively carry out its mandate
GANZON v. CA as protector of the people against inept and corrupt government
1991 officers and employees.

Summary: Mayor Ganzon of Iloilo City was put in preventive suspension It was given disciplinary authority over all elective and appointive
by Secretary Santos due to 10 administrative complaints. Ganzo put officials of the government and its subdivisions, instrumentalities and
forward the possibility of simultaneous service of the 3rd and 4th agencies (with the exception only of impeachable officers, members of
suspension orders. SC: Simultaneous service is allowed in this case (3rd Congress and the Judiciary). Also, it can preventively suspend any
order was reckoned from finality of court decision, 4 th order reckoned officer under its authority pending an investigation when the case so
from issuance of order by the Secretary) insofar that the 2 orders warrants.
overlapped, in light of Sec. 63 LGC of 1991 and the ‘intriguing’ action of
the Secretary in imposing successive suspension orders.
AGUINALDO v. SANTOS
Doctrine: Successive suspension orders are discouraged especially August 21, 1992
when a consolidated response, as opposed to a piecemeal response, is
possible. Simultaneou Summary: Governor Aguinaldo was charged administratively for his
s service of suspension orders are allowed in cases where the facts call alleged participation in the 1989 coup. The Secretary issued a
for its application, in light of the LGC policy against unjust successive resolution removing Aguinaldo as Governor. Aguinaldo filed a petition
suspensions. in the SC. While such petition was pending, Aguinaldo won again as
Governor of Cagayan in the 1992 elections and the decision in the
Class Notes: In response to the case of Ganzon, the LGC provided a limit Disqualification case against him in the SC was rendered in Aguinaldo’s
as seen in Sec. 63 (see previous discussion on when to apply the 90-day favor.
limit)
Doctrine: The rule is that a public official cannot be removed for
administrative misconduct committed during a prior term, since his re-
ALEJANDRO v. OFFICE OF THE OMBUDSMAN FACT-FINDING AND election to office operates as a condonation of the officer's previous
INTELLIGENCE BUREAU misconduct to the extent of cutting off the right to remove him therefor.
April 3, 2013 The foregoing rule, however, finds no application to criminal cases
pending against Aguinaldo for acts he may have committed during the
Summary: The son of Brgy. Chairman Alejandro owned a carwash which failed coup.
illegally opened a fired hydrant of Manila Water Services Inc. Said
company coordinated with the PNP to conduct an anti-water pilferage Inasmuch as the power and authority of the legislature to enact a local
operation against the carwash. The PNP arrested the carwash boys government code, which provides for the manner of removal of local
illegally getting water from the fire hydrant and confiscated the water government officials, is found in the 1973 Constitution as well as in the
containers. It was then that Brgy. Chairman Alejandro whose son 1987 Constitution, then it cannot be said that BP Blg. 337 was repealed
owned the carwash, interfered and as a result, the carwash boys by the effective of the present Constitution.
escaped. An administrative case for grave misconduct was filed against
him with the Ombudsman which ordered his dismissal from service. At
the SC, Brgy. Chairman Alejandro assailed the jurisdiction and the GARCIA v. MOJICA
power of Ombudsman to order his dismissal from service. September 10, 1999

The SC ruled that the Ombudsman had concurrent jurisdiction with the Summary: The Ombudsman issued a preventive suspension order for 6
Courts and administrative agencies over the complaint. Moreover, it months, without allowing an answer first, against Mayor Garcia in
had the power to impose administrative sanctions upon erring public relation to administrative charges pending against him. Garcia, invoking
Aguinaldo, challenged (1) the Ombudsman’s authority to investigate the PEOPLE v. TOLEDANO
charges, since they were committed prior to his reelection, and (2) its May 18, 2000
authority to issue the preventive suspension, when the LGC provides
that such suspension may be issued only after issues have been joined, Summary: Private respondent Rolando Bunao, while a member of the
and should not exceed 60 days. The SC, while finding that the Sangguniang Bayan of Sta. Cruz, Zambales and its Committee on Bids
Ombudsman may investigate and issue the preventive suspension and Awards, entered into a lease contract with said municipality
based on the Ombudsman Law, not on the LGC, it may not hold Garcia covering two public market stalls. Two administrative charges for
administratively liable by virtue of his reelection, which effectively violation of Sec. 3(h), RA 3019 and RA 6713 were filed against Bunao.
condoned his past misconduct. This does not change even if the Both administrative complaints were dismissed. Later, an information
misconduct (signing of an unfavorable contract) happened just 4 days for violation of Sec. 41(1) in relation to Sec. 221 of BP Blg 337 (old LGC)
after the election, and to be implemented in his current term. was filed against Bunao before the RTC. Upon motion of Bunao, the RTC
dismissed the information and held that the dismissal of administrative
Doctrine: When the electorate put [the official] back into office, it is complaints against Bunao and his subsequent re-election extinguished
presumed that it did so with full knowledge of his life and character, his criminal liability. A petition for certiorari and mandamus was filed
including his past misconduct. If, armed with such knowledge, it still with the SC, which reversed the ruling of the RTC and ruled that the
reelects him, then such reelection is considered a condonation of his dismissal of the administrative liability did not extinguish Bunao’s
past misdeeds. Rulings to this effect do not distinguish the precise liability under the LGC. His subsequent re-election did not affect his
timing or period when the misconduct was committed, reckoned from criminal liability either.
the date of the official's reelection, except that it must be prior to said
date. Doctrine: Administrative cases are independent from criminal actions
for the same act or omission. The dismissal of the former does not
While under the LGC, preventive suspension may be imposed only after result in the extinction of criminal liability. Likewise, the ruling in
the issues are joined and only for a maximum of 60 days, (1) Aguinaldo v Santos that forbids the removal from office of a public
administrative complaints commenced under the Ombudsman Law are official for administrative misconduct committed during a prior term,
distinct from those initiated under the LGC, and (2) the limited period finds no application to criminal cases, pending against said public
under the LGC which may be imposed by the President, Governor, or officer. Lastly, despite the enactment of the new LGC, the same
Mayor, is to prevent partisan politics [citing Senate delibs on LGC, which provision under which Bunao was charged and penalize was reenacted.
stated that they wanted to avoid the case of Ganzon v. CA]. The Therefore, the act committed before the reenactment continues to be a
Ombudsman, however, is a constitutional body. crime.

Class Notes: The LGC has penal provisions so we can pursue a criminal
case against erring officials.
GARCIA v. MIRO
February 5, 2003
CALINGIN v. CA
Summary: Ombudsman Special Prosecution Officer Jesus Rodrigo July 12, 2004
Tagaan charged Cebu City Mayor Alvin Garcia with violation of the Anti-
Graft and Corrupt Practices Law as a result of his having entered into a Summary: The Office of the President issued a resolution suspending
contract with F.E. Zuellig for the supply of asphalt batching plant for Gov. Calingin of Misamis Oriental for 90 Days. Gov. Calingin filed his MR
three years. The joint affidavits of State Auditors Cabreros and Quejada before the OP. DILG issued a Memorandum implementing the said
alleged that petitioner entered into the contract without available funds resolution. Gov. Calingin filed a case for prohibition before the CA to
appropriated to cover the expenditure, in violation of Sections 85 and prevent the DILG from executing the suspension order. He alleged that
86 of P.D. 1445 or the State Audit Code of the Phil.; that petitioner Decisions of OP on cases where it has original jurisdiction become final
exceeded the authority granted him by the Sangguniang Panlungsod; and executory only after the lapse of 15 days from the receipt thereof
and that the contract is manifestly disadvantageous to the City. Filing of MR shall suspend the running of the said period (Admin Code
Thereafter, Special Prosecution Officer Tagaan resigned from his office 1987, Book VII, Chap. 3, Sec. 15). Furthermore, he argues that Sec. 67, of
and his name was withdrawn as complainant in the case. The Deputy LGC 1991 which provides that decisions of the OP shall be final and
Ombudsman issued an Order requiring Mayor Garcia to submit executory Applies only to decision of the OP on admin cases appealed
counter-affidavit. Instead of filing a counter-affidavit, Garcia filed with from the sangguniang panlalawigan, panlungsod (of HUCs and ICCs),
the Supreme Court a petition to prohibit the Ombudsman from and bayan of municipalities within Metro Manila. It does not cover
conducting the preliminary investigation on the ground that there is no decisions on cases where OP has original jurisdiction such as those
sufficient complaint. The SC held that the COA Special Audit Report and involving a Provincial Governor.
the affidavits of State Auditors constitute a valid complaint that is
sufficient to support a criminal proceedings. The SC disagreed and held that the LGC is the applicable law being the
law with special application. Under the LGC, OP decisions are
Doctrine: For purposes of initiating a preliminary investigation before immediately executory and the appeal to the CA does not suspend the
the Office of the Ombudsman, a complaint “in any form or manner” is execution pending such appeal.
sufficient. Section 12, Article XI of the Constitution states that the
Ombudsman and his Deputies, as protectors of the people, shall act Doctrine: (On applicability of LGC over Admin Code) The LGC is a law of
promptly on “complaints filed in any form or manner against public special application and must prevail over the Admin Code which is of
officials or employees of Government.” In Almonte v. Vasquez, it was general application. It is a principle of statutory construction that
held that even unverified and anonymous letters may suffice to start an where there are 2 statutes that apply to a particular case, that which
investigation. In permitting the filing of complaints “in any form or was specially intended for said case must prevail. Furthermore, LGC
manner,” the framers of the Constitution took into account the well- was enacted much later than the Admin Code of 1987
known reticence of the people which keep them from complaining
against official wrongdoings. The Office of the Ombudsman is different (On OP decisions) decisions of the OP are final and executory. No MR is
from the other investigatory and prosecutory agencies of the allowed BUT the parties may appeal the decision to the CA. However,
government because those subject to its jurisdiction are public officials the appeal does not stay the execution of the decision. Hence, the DILG
who, through official pressure and influence, can quash, delay or Secretary may validly move for its immediate execution.
dismiss investigations held against them.
Class Notes: This case is a review of procedure.
VALENCIA v. SANDIGANBAYAN
June 29, 2004
THE OMBUDSMAN
See Comparative Table of Preventive Suspensions on page 27
Summary: The petitioners in this case are the governor, vice-governor,
members of the Sangguniang Panlalawigan, and provincial Jurisdiction of the Ombudsman
administrator of Oriental Mindoro. They were charged with the
violation of Section 3 (e) in relation to Section 3 (g) of Republic Act No.
3019, the Anti-Graft and Corrupt Practices Act. It was alleged that the Acts or omissions of a public
said public official entered into a contract of loan granting Engr. Atienza officer or employee in cases
PRIMARY JURISDICTION
a loan of 2.5M to the injury of the province. During the pendency of the cognizable by the Sandiganbayan
criminal case, the administrative case against them was dismissed due (Salary Grade of 27 or higher)
to their reelection. The petitioners filed a motion to quash with the
Sandiganbayan alleging that the crim case should likewise be dismissed Cases cognizable by regular
on account of the dismissal of the admin case. The Sandiganbayan Concurrent Jurisdiction courts and other investigative
denied the motion so they filed a certiorari case with the SC. The SC agencies of government
ruled that the dismissal of the motion to quash was proper. The SC
further ruled that the dismissal of the admin case does not
automatically warrant the dismissal of the crim case because they have HAGAD v. GOZO-DADOLE
different purposes. December 12, 1995

Doctrine: The re-election of a public official extinguishes only the Summary: Two city councilors filed criminal and administrative
administrative, but not the criminal, liability incurred by him during his complaints against the Mayor, Vice-mayor and Sangguniang
previous term of office, thus: Panglungsod member (respondent officials) with the Ombudsman. The
Ombudsman issued a preventive suspension order. Respondent
The ruling that – “when the people have elected a man to his office it officials, through a petition filed in RTC, contend that the subsequent
must be assumed that they did this with knowledge of his life and enacted of LGC 1991 divested Ombudsman with jurisdiction over them.
character and that they disregarded or forgave his faults or misconduct The OP, as provided in LGC, has jurisdiction. RTC issued a writ of
if he had been guilty of any” – refers only to an action for removal from preliminary injunction against Ombudsman, sided with respondent
office and does not apply to a criminal case, because a crime is a public officials. SC reversed saying that OP and Ombudsman now have
wrong more atrocious in character than mere misfeasance or concurrent jurisdiction.
malfeasance committed by a public officer in the discharge of his duties,
and is injurious not only to a person or group of persons but to the State Doctrine: There is nothing in the Local Government Code to indicate
as a whole. This must be the reason why Article 89 of the Revised Penal that it has repealed, whether expressly or impliedly, the pertinent
Code, which enumerates the grounds for extinction of criminal liability, provisions of the Ombudsman Act. they have concurrent jurisdiction.
does not include reelection to office as one of them, at least insofar as a
public officer is concerned. Also, under the Constitution, it is only the The two provisions however govern differently. In order to justify
President who may grant the pardon of a criminal offense. preventive suspension by the Ombudsman, the evidence of guilt should
be strong and either
1) the charge against the officer or employee should involve
dishonesty, oppression or grave misconduct or neglect in the
performance of duty;
2) the charges should warrant removal from the service or
3) the respondent’s continued stay would prejudice the case
filed against him. He could also impose it on all public
officials.

While the LGC can only impose on elective local officials at any time
after the issues are joined and it would be enough that
(a) there is reasonable ground to believe that the respondent
has committed the act or acts complained of,
(b) the evidence of culpability is strong,
(c) the gravity of the offense so warrants, or
(d) the continuance in office of the respondent could influence
the witnesses or pose a threat to the safety and integrity of
the records and other evidence.

Class Notes:
- Can one file administrative cases against elective officials
with different disciplining authorities simultaneously?
 We are not sure; But it seems like this is possible
 Cases do NOT categorically say that the filing with one
excludes the filing with others
- Read Hagad with the case of Alejandro v. OMB-Fact Finding

CASTILLO-CO v. BARBERS
June 16, 1998

Summary: Quirino Congressman Cua filed a complaint before the


Ombudsman against Governor Castillo-Co and Provincial Engineer
Ringor for buying “reconditioned” instead of “brand new” heavy
equipment as required by the resolutions of the Sanguniang
Panlalawigan. They were preventively suspended for six months by the Doctrine: The powers of the Ombudsman are not merely
Deputy Ombudsman. They argued that the Deputy Ombudsman did not recommendatory. His office was given teeth to render this
have such authority because he is not the Ombudsman; that it was in constitutional body not merely functional bust also effective. The
violation of their right to due process for being issued hastily; that the Ombudsman has, therefore, the constitutional power to directly remove
conditions of the preventive suspension were not met; and that the from government service an erring public official other than a member
duration is excessive. The Court ruled that the Deputy Ombudsman has of Congress and the Judiciary [Estarija v. Ranada].
the authority to order preventive suspension; that its issuance seven
days after filing of the complaint is not grave abuse of discretion; that Class Notes: So can the Ombudsman remove officials?
all the conditions were met; and that six months of suspension is within - YES!
the limits in RA 6770. - But Sir is not satisfied with the legal basis for such power.
 The SC used the phrase “ensure compliance” in the
Doctrine: A preventive suspension can be decreed on an official under Constitutional provision on the COA as basis
investigation after charges are brought and even before the charges are  The Court extended this phrase to include the power to
heard since the same is not in the nature of a penalty, but merely a dismiss which Sir finds “funny”
preliminary step in an administrative investigation. The first requisite
for preventive suspension, that the evidence of guilt is strong, rests
upon the determination of the disciplining authority.

LAPID v. CA
June 29, 2000

Summary: An administrative case was filed against Governor Lapid


before the Ombudsman. The latter found the governor guilty and meted
out the penalty of 1 yr suspension without pay pursuant. Lapid filed a
TRO before the CA which was granted. When the TRO expired, the DILG
immediately executed the ombudsman’s ruling. The issue before the SC
is whether or not the ombudsman’s ruling is immediately executory
pending appeal. The SC ruled that the decision is stayed by the appeal
pursuant to the RA 6770.

Doctrine: Lapid was charged administratively before the ombudsman.


Ombudsman Act applies. Sec. 68 of the LGC applies to administrative
decisions rendered by the Office of the President or the
appropriate Sanggunian against elective local government officials.

Class Notes: Under the Rules of Procedure of the OMB:

Decision has the effect of:


1. Absolving charges
2. Penalty of public censure or
Final and Executory
reprimand
Immediately
3. Suspension of not more than
1 month, or a fine of not less
than 1 month salary
Final and enforceable
Inclusio unius est exclusion alterius
Unless MFR within 5 days All other cases decided by the OMB
from receipt; or appeal to that does not fall under the above
SC within 10 days from instances
receipt or denial of MR

COA REGIONAL OFFICE XIII v. HINAMPAS & CABANOS


August 7, 2007

Summary: These are five consolidated petitions, all assailing the CA’s
decisions in said petitions overturning the Ombudsman’s actions. The
decisions of the CA in the consolidated was premised on the so-called
Tapiador doctrine, an obiter which declared that the Ombudsman’s
powers are recommendatory, and thus had no power to dismiss a
public official. In ruling upon this issue, the SC herein declared that, as
the so-called Tapiador doctrine is a mere obiter, it cannot be cited as
doctrine [Ledesma v. CA]. Furthermore, it was the intent of the
lawmakers to give the Ombudsman the power to remove from
government service erring public officials, with the exception of
members of Congress and the Judiciary [Estarija v. Ranada]. The SC, in
view of this disquisition, reversed the decisions of the CA that merely
relied on the Tapiador doctrine, and reinstated the Ombudsman’s
decisions. The one case where the CA’s decision was affirmed was
based on the lack of substantial evidence.
authority in doing so does not constitute a violation of the law and the
THE COURTS imposition of the 90 day penalty is wrong for a single offense.
Sandiganbayan ruled that it violated public trust. Upheld by the SC but
See Comparative Table of Preventive Suspensions on page 27
amended penalty to 60 days.
Sec. 13, RA 3019 provides for a mandatory suspension pendente lite
Doctrine: Public office is a public trust - The good of the service and the
- Any public officer against whom any criminal prosecution
degree of morality which every official and employee in the public
under a valid information under this Act or under the
service must observe, if respect and confidence are to be maintained by
provisions of the Revised Penal Code on bribery is pending in
the Government in the enforcement of the law, demand that no
court, shall be suspended from office. Should he be convicted
untoward conduct on his part, affecting morality, integrity and
by final judgment, he shall lose all retirement or gratuity
efficiency while holding office should be left without proper and
benefits under any law, but if he is acquitted, he shall be
commensurate sanction, all attendant circumstances taken into
entitled to reinstatement and to the salaries and benefits
account.
which he failed to receive during suspension, unless in the
meantime administrative proceedings have been filed
Class Notes: At the end of the decision, the Court held that the 90-day
against him.
suspension was improper since Sec. 63 of the LGC applies. The Court
said that the for local government officials, the period should be 60
NOTE: Sec. 13 does not provide for the period of suspension
days.
- It will be unconstitutional if the law is applied strictly where
- Sir: “INSANE!”
the official can be suspended for the whole duration of the
- In all other cases the court separated the application of both
case
laws
- Possible remedies:
 LGC has NO effect (nor did it modify or repeal) RA 3019
1. Discretion is given to the court as to the length of the
suspension
2. Apply other laws (regardless of the nature of the case)
SEGOVIA v. SANDIGANBAYAN
a. For elective officials, apply LGC and impose 60
March 27, 1998
days as what happened in Rios (see, however, the
comments under Rios)
b. For non-elective officials,, apply CSC or RAC and Summary: Segovia, et al were designated as members of the Contracts
impose 90 days as what happened in Segovia Committee of NPC for the Mindanao Grid Projects. A bidding was held.
The lowest and second lowest bidders were the Joint Venture of
INPHASE and T & D, and Urban Consolidated Constructors,
Inc., respectively. Joint Venture was disqualified but instead of
BUNYE v. ESCAREAL
awarding the project to Urban it was also disqualified and a failure of
September 10, 1993
bidding was declared. Subsequently, the project was cancelled. Urban
charged Segovia, et al with violation of the Anti Graft and Corrupt
Summary: Petitioners are local elective officials of the Municipality of
Practices Act. After a preliminary investigation, the Ombudsman
Munitnlupa. An information was charged against them for violation of
recommended the filing of a case thus a case was filed with the
the Anti-Graft and Corrupt Practices Act. They enacted an ordinance
Sandiganbayan. Sandiganbayan issued a resolution placing Segovia, et
under which they took over the management of the public market and
al, under preventive suspension of 90 days. Segovia, et al questioned it
rescinded the lease agreement with the cooperative members.
claiming that preventive suspension is not mandatory but is subject to
Consequently, they were suspended by the Sandiganbayan. They
the sound discretion of the court. Also, their suspension is no longer
assailed the suspension, claiming that there was no need for such
necessary because the project was already cancelled, they are no longer
suspension since they already admit to doing the acts charged against
involved in the awarding of bids, and that all documents necessary for
them. The Court upheld the suspension, holding that it was mandatory
the investigation were already submitted. SC declared that preventive
under Sec 13, RA 3019.
suspension is mandatory and that it is also intended to prevent the
accused from committing further acts of malfeasance while in office.
Doctrine: Such preventive suspension is mandatory; there are no ifs and
buts about it. All told, preventive suspension is not violative of the
Doctrine: Under Section 13 of the Anti-Graft and Corrupt Practices Law,
Constitution as it is not a penalty. In fact, suspension particularly under
the suspension of a public officer is mandatory after a determination
Section 13 of Rep. Act 3019 is mandatory once the validity of the
has been made of the validity of the information in a pre-suspension
information is determined.
hearing conducted for that purpose. (Socrates vs. Sandiganbayan, and
Luciano, et al vs. Mariano)
Class Notes:
- Does RA 3019 provide for a period for the suspension?
 No, the law merely refers to a suspension PENDENTE
MIRANDA v. SANDIGANBAYAN
LITE
July 27, 2005
- The Court upheld the suspension since it is MANDATORY
 But was it really necessary? One of the defenses raised
Summary: The Ombudsman placed Jose C. Miranda, mayor of Santiago
was that the suspension serves no purpose since the
City, Isabela, under preventive suspension for alleged violations of the
issues are NOT factual but purely legal
Code of Conduct and Ethical Standards for Public Officials and
 The Court dismissed such contention and held that the
Employees. Mayor Miranda, despite the suspension order insisted on
suspension is mandatory
performing the functions and duties of Mayor despite Vice Mayor
 But what happened to the intent of the law behind
Navarrro’s requests to desist and in spite of the order of DILG
preventive suspensions?
Undersecretary Manuel Sanchez directing him to cease from
reassuming the position. The Ombudsman filed with the Sandiganbayan
an Information against Mayor Miranda for violation of Article 177 of the
RIOS v. THE 2ND DIVISION OF THE SANDIGANBAYAN RPC, penalizing usurpation of authority. The prosecution also filed
September 26, 1997 motion to suspend Mayor Miranda pendente lite based on Section 13 of
RA 3019, the Anti-Graft and Corrupt Practices Act. Miranda opposed the
Summary: Rios, mayor of San Fernando, Romblon was accused for motion on the ground that the offense of usurpation of authority or
violation of RA 3019 Section 3 (e) for confiscating tanguile lumber and official functions under Article 177 of the RPC is not embraced by
disposing of the same without prior authority from the DENR and Section 13 of RA 3019. The Sandiganbayan preventively suspended
Sangguniang Bayan . He alleged that the mere fact that he exceeded his
Mayor Miranda from office for 90 days. The SC agreed with the ruling of 2. Bribery under RPC
the Sandiganbayan on the propriety of the suspension order. 3. Book II, Title VII, RPC
4. Fraud on the government
Doctrine: Under Section 13 of the Anti-Graft and Corrupt Practices Law, 5. Violations involving public funds/property
the suspension of a public officer by the Sandiganbayan is mandatory - Here, the violation was under OEC but the Court held that the
after a determination has been made of the validity of the Information. act also constitutes fraud on the government
Once the information is found to be sufficient in form and substance,
then the Court must issue the order of suspension as a matter of
course. There are no ifs and buts about it. LLORENTE, JR. v. SANDIGANBAYAN
January 19, 2009
The Sandiganbayan properly construed Section 13 of R.A. No. 3019 as
covering two types of offenses: (1) any offense involving fraud on the Summary: While trial of 2 criminal cases against municipal mayor
government; and (2) any offense involving public funds or property. Llorente has not begun before the Sandiganbayan, Congress enacted RA
Nothing limits Section 13 only to acts involving fraud on public funds or 7975, amending Section 4 of Presidential Decree No. 1606. Llorente
property. filed with the Sandiganbayan, a motion to dismiss or transfer Criminal
Case to the RTC of Sindangan, Zamboanga. He averred that the
Also, the difference between suspensions by the Ombudsman and the enactment of RA 7975 divested the Sandiganbayan of its jurisdiction
President, governor and mayor under the LGC are clear. The latter are over criminal cases against municipal mayors for violations of Republic
political personages and so the possibility of extraneous factors Act No. 3019, as amended, who receive salary less than that
influencing their decision to impose preventive suspensions is not corresponding to Grade 27, pursuant to the Index of Occupational
remote. The Ombudsman, on the other hand, is given the independence Services prepared by the DBM. Motion to refer and transfer criminal
of the office which is protected by the Constitution. case were denied. SC affirmed Sandiganbayan stating that it had
jurisdiction.
The two provisions govern differently. In order to justify the preventive
suspension by the Ombudsman, the evidence of guilt should be strong, Doctrine: Sandiganbayan has jurisdiction over violations of Republic Act
and (a) the charge against the officer or employee should involve No. 3019, as amended, against municipal mayors. It is the official’s
dishonestly, oppression or grave misconduct or neglect in the grade that determines his or her salary, not the other way around. To
performance of duty; (b) that the charges should warrant removal from determine whether the official is within the exclusive jurisdiction of the
the service; or (c) the respondent's continued stay in office would Sandiganbayan, therefore, reference should be made to Republic Act
prejudice the case filed against him. No. 6758 and the Index of Occupational Services, Position Titles and
Salary Grades. An official’s grade is not a matter of proof, but a matter of
On the other hand, the LGC requirements for suspension (at any time law which the court must take judicial notice.
after the issues are joined), are that (a) there is reasonable ground to
believe that the respondent has committed the act or acts complained Section 444 (d) of the Local Government Code provides that "the
of, (b) the evidence of culpability is strong, (c) the gravity of the offense municipal mayor shall receive a minimum monthly compensation
so warrants, or (d) the continuance in office of the respondent could corresponding to Salary Grade twenty-seven (27) as prescribed under
influence the witnesses or pose a threat to the safety and integrity of Republic Act No. 6758 and the implementing guidelines issued
the records and other evidence. pursuant thereto." Additionally, both the 1989 and 1997 versions of the
Index of Occupational Services, Position Titles and Salary Grades list the
municipal mayor under Salary Grade 27.Consequently, the cases against
JUAN v. PEOPLE OF THE PHILIPPINES petitioner as municipal mayor for violations of RA 3019, as amended,
January 18, 2000 are within the exclusive jurisdiction of the Sandiganbayan.

Summary: Petitioners are barangay officials of Talipapa, Novaliches. Class Notes:


They were accused of using barangay government-owned property in - So even if one is not a public officer, Sandiganbayan still has
their election campaigns. Private complainants filed a motion for jurisdiction:
removal of said local officials from office. TC directed the suspension of 1. If his co-accused is a public officer
petitioners. CA and SC affirmed. Before the SC, one of petitioners’ 2. If related to his office even if he is not a public officer
arguments was that the RTC did not have jurisdiction to hear the cases anymore
against them. SC rejected this contention and ruled that the RTC had - Which officials fall under Salary Grade 27 and above? As per
jurisdiction. Sec. 443-486:

Doctrine: By virtue of the exception in Sec. 32, BP 129 (“Except in cases In Municipalities Municipal mayor
falling within the exclusive original jurisdiction of the RTCs and of the City Mayor
Sandiganbayan, the MTCs shall exercise….”), the exclusive jurisdiction In Cities Vice mayor
of first-level courts does not cover criminal cases, which, by specific Sangguniang Panlungsod members of HUCs
provisions of the law, fall within the exclusive original jurisdiction of Governor
the RTCs and Sandiganbayan, regardless of the penalty. Provinces Vice governor
Sangguniang Panlalawigan members
Jurisdiction is conferred by the Constitution or Congress. Outside the
cases enumerated in the Constitution, Congress has the plenary power
to define, prescribe, and apportion the jurisdiction of various courts by
ESQUIVEL v. OMBUDSMAN
the creation of a law.
September 17, 2002
Sec. 268, OEC is one such law. Thus, election offenses fall within the
Summary: Policemen Eduardo and Catacutan charged Antonio Prospero
exception in Sec. 32. RTCs have jurisdiction to hear and decide cases for
Esquivel, municipal mayor of Jaen and his brother, Mark Anthony
violation of the OEC.
"Eboy" Esquivel, barangay captain of barangay Apo, Jaen, with:
Alleged illegal arrest, arbitrary detention, maltreatment, attempted
Class Notes: Even if it is not an RA 3019 violation, the suspension
murder, and grave threats before Philippine National Police – Criminal
pendente lite in Sec. 13 may still be applied.
Investigation and Detection Group (PNP-CIDG), in Camp Olivas, San
- RA 3019, Sec. 13 covers:
Fernando, Pampanga.
1. Violations of RA 3019
After the initial investigation, the PNP-CIDG Third Regional Office Doctrine: Under the last paragraph of Sec. 2 of RA 7975, if the position
forwarded the pertinent records to the Office of the Deputy of ONE of the principal accused is classified as SG 27, the SB has original
Ombudsman for Luzon for appropriate action. The Office of the Deputy and exclusive jurisdiction over the offense. In cases where NONE of the
Ombudsman for Luzon conducted a preliminary investigation. The
accused are occupying positions corresponding to SG 27 or higher, as
Deputy Ombudsman for Luzon issued the impugned resolution,
recommending that both Mayor Esquivel and Barangay Captain Mark prescribed in RA 6758, (or military and PNP officers mentioned in Sec.
Anthony "Eboy" Esquivel to be indicted for the crime of less serious 2(a)(1), exclusive jurisdiction thereof shall be vested in the regular
physical injuries, and Mayor Esquivel alone for grave threats. courts.

Thereafter, separate informations docketed as Criminal Case No. 24777 Under RA 8249, the SB has original jurisdiction over crimes or felonies
for less serious physical injuries against Mayor Esquivel and Mark in which the public office is a CONSTITUENT ELEMENT as defined by
Anthony "Eboy" Esquivel, and Criminal Case No. 24778 for grave statute and the relation between the crime and the offense is such that,
threats against Mayor Esquivel, were filed with the Sandiganbayan. in a legal sense, the offense committed cannot exist without the office.
Since the public office of the accused is by statute a constituent element
The brothers Esquivel contest the authority of the Ombudsman to of the crime charged, there is no need to state in the INFO specific
conduct preliminary investigations, and the jurisdiction of the factual allegations of the intimacy between the office and the crime
Sandiganbayan to take congizance of criminal cases filed against the charged, or that the accused committed the crime in the performance of
municipal mayors and barangay captains. his duties.

Doctrine: On the power of the Ombudsman to conduct preliminary SB also has original jurisdiction over offenses or felonies which are
investigations: The Ombudsman is empowered to determine whether INTIMATELY CONNECTED with the public office and are perpetrated by
there exists reasonable ground to believe that a crime has been the public officer or employee while in the performance of his official
committed and that the accused is probably guilty thereof and, functions, through improper or irregular conduct.
thereafter, to file the corresponding information with the appropriate
courts. A public officer who is not in charge of public funds or property by
virtue of her official position, or even a private individual, may be liable
On the jurisdiction of the Sandiganbayan: for malversation or illegal use of public funds or property if such public
In previous cases, the Court has already held that municipal mayors fall officer or private individual conspires with an accountable public
under the original and exclusive jurisdiction of the Sandiganbayan, officer to commit malversation or illegal use of public funds or
being of a salary grade 27. Barangay Captain Mark Anthony Esquivel property.
cannot claim that since he is not a municipal mayor, he is outside the
Sandiganbayan’s jurisdiction.

R.A. 7975, as amended by R.A. No. 8249, provides that it is only in cases INDING v. SANDIGANBAYAN
where "NONE of the accused are occupying positions corresponding to July 14, 2004
salary grade ‘27’ or higher" that "exclusive original jurisdiction shall be
vested in the proper regional trial court, metropolitan trial court, Summary: Ricardo S. Inding is a member of the Sangguniang
municipal trial court, and municipal circuit court, as the case may be, Panlungsod (SP) of Dapitan City. He was charged with violation of
pursuant to their respective jurisdictions as provided in Batas Section 3(e) of RA No. 3019 before the Sandiganbayan. He allegedly
Pambansa Blg. 129, as amended." faked buy bust operations against drug dealers to enable him to
reimbursement for actual expenses incurred during the alleged buy-
Since Barangay Captain Esquivel is the co-accused in Criminal Case No. bust operations. Inding filed a Motion to Dismiss assailing the
24777 of Mayor Esquivel, whose position falls under salary grade 27, jurisdiction of the Sandiganbayan. He insisted that the Sandiganbayan
the Sandiganbayan committed no grave abuse of discretion in assuming has jurisdiction only over officials with salary grade 27 and that as a
jurisdiction over said criminal case, as well as over Criminal Case No. member of the SP, he has only a salary grade of 25. The controversy
24778, involving both of them. was further complicated by the fact that the jurisdiction of the
Sandiganbayan under PD 1606 was amended twice: by RA 7975 and by
RA 8294.

BARRIGA v. SANDIGANBAYAN The SC ruled that the Sandiganbayan had jurisdiction over Inding
April 26, 2005 notwithstanding the fact that he has a salary grade lower than 27.

Summary: The OMB filed before the SB 3 Informations against Carmen, Doctrine: RA 7975 which amended PD 1606, took effect on May 16,
Cebu Municipal Mayor Villamor and Municipal Accountant Barriga for 1995. Subsequently, RA 7975 was amended by RA 8249 which took
malversation and illegal use of public funds. The SB admitted the Infos. effect on February 23, 1997. For purposes of determining which of the
Barriga filed a motion to quash the Infos on the ground that the SC has two laws, RA 7975 or RA 8249, applies in the present case, the
no jurisdiction over the offenses charged. She claimed, among others, reckoning period is the time of the commission of the offense.
that the allegations in the INFOs were not sufficient because it failed to
allege the intimate relations between the crimes charged and the Violation of RA 3019 committed by officials in the executive branch
official duties of her position as municipal accountant. She also claimed with SG 27 or higher, and by the officials specifically enumerated,
that since her position was only classified as Salary Grade 24, the SB has regardless of their salary grades, fall within the original jurisdiction of
no jurisdiction. The SB denied her motion, so she filed a Rule 65 petition the Sandiganbayan.
with the SC. The SC denied her petition and upheld the jurisdiction of
the SB. It said that since the crimes charged were classified as offenses All other public officials below SG 27 shall be under the jurisdiction of
in which public office is a constituent element, it was not necessary to the proper trial courts where none of the principal accused are
allege the intimate relations between the crime charged and the occupying positions corresponding to SG 27 or higher, except for those
position of accused in the INFO. Also, since Barriga was charged IN officials specifically included in Section 4, regardless of their salary
CONSPIRACY with the mayor, whose position as classified as SG 27, grades, over whom the Sandiganbayan has jurisdiction.
then regardless of her being classified as SG 24, the SB has original
jurisdiction over the crimes charged.
RODRIGUEZ v. SANDIGANBAYAN
March 3, 2004
REMOVAL

Summary: Logs ordered by Mayor Rodriguez to be cut without prior


AGUINALDO v. SANTOS
permits were confiscated by a team of law enforcement officers. Some
August 21, 1992
logs were hauled in the RAC compound. Mayor Rodriguez ordered
Barangay Captain Abonita and 2 police officers to forcibly take the logs
Summary: Governor Aguinaldo was charged administratively for his
from the RAC compound. A complaint was filed against Rodriguez and
alleged participation in the 1989 coup. The Secretary issued a
Abonita for Obstruction of Justice under P.D. 1829. Petitioners question
resolution removing Aguinaldo as Governor. Aguinaldo filed a petition
the jurisdiction of the Sandiganbayan to try the case arguing that the
in the SC. While such petition was pending, Aguinaldo won again as
offense charged against them is not in relation to their office.
Governor of Cagayan in the 1992 elections and the decision in the
Disqualification case against him in the SC was rendered in Aguinaldo’s
Doctrine: Montilla v. Hilario, provides that for an offense to be
favor.
committed in relation to the office, the relation between the crime and
the office must be direct and not accidental, in that in the legal sense,
Doctrine: The rule is that a public official cannot be removed for
the offense can not exist without the office. An exception to Montilla, is
administrative misconduct committed during a prior term, since his re-
in People v. Montejo, where the SC held that although public office is not
election to office operates as a condonation of the officer's previous
an element of an offense charged, as long as the offense charged in the
misconduct to the extent of cutting off the right to remove him therefor.
information is intimately connected with the office and is alleged to
The foregoing rule, however, finds no application to criminal cases
have been perpetrated while the accused was in the performance,
pending against Aguinaldo for acts he may have committed during the
though improper or irregular, of his official functions, there being no
failed coup.
personal motive to commit the crime and had the accused would not
have committed it had he not held the aforesaid office, the accused is
Inasmuch as the power and authority of the legislature to enact a local
held to have been indicted for "an offense committed in relation" to his
government code, which provides for the manner of removal of local
office.
government officials, is found in the 1973 Constitution as well as in the
1987 Constitution, then it cannot be said that BP Blg. 337 was repealed
by the effective of the present Constitution.

PABLICO v. VILLPANDO
July 31, 2002

Summary: Pablico, Vice Mayor of San Vicente, Palawan, who took an


oath as Mayor, assails the decision of the CA, nullifying the decisions of
the SP of Palawan and the OP imposing removal upon Villapando,
Mayor, for alleged violation of the Constitution. The SC denied the
petition, ruling that, based on the clear text of the law and the
legislative intent, the power to remove is exclusively vested in the
proper courts.

Doctrine: It is clear from the last paragraph of Section 60, LGC, that the
penalty of dismissal from service upon an erring elective local official
may be decreed only by a court of law. Such power is exclusively vested
in the proper courts (Salalima v. Guingona).

Class Notes: Implications of the case: that remedies other than dismissal
may be imposed by the Disciplining Authority, and if the remedy is
removal one must go to court
COMPARATIVE TABLES OF PREVENTIVE SUSPENSION
UNDER THE LGC, OMBUDSMAN ACT, AND RA 3019

LOCAL GOVERNMENT CODE OMBUDSMAN ACT (RA 6770) RA 3019

Basis Sec. 63 Sec. 24 Sec. 13

President, Governor, or Mayor


Suspending
Depending on the position of the The Ombudsman The Sandiganbayan
Authority
respondent

1. Evidence of guilt must be


1. Reasonable ground to believe strong, AND
that respondent has committed 2. The:
Mandatory suspension PENDENTE LITE
the act/s complained of a. Charge should involve
Grounds to 2. Evidence of culpability is strong dishonesty, oppression, or
Upon existence of a valid information
justify 3. Gravity of offense so warrants, grave misconduct or
determined at pre-suspension hearing
imposition of or neglect in performance of
preventive 4. Continuance in office of the duty, or
Pre-suspension hearing need not be an
suspension respondent could influence the b. Charge should warrant
actual hearing, pleadings may suffice
witnesses or pose a threat to removal from service, or
(Juan v. People)
the safety and integrity of the c. Continued stay in office
records and other evidence would prejudice case filed
against him

Any public official who has been charged


with violation of RA 3019, Book II, Title
ALL public officials, elective or appointive,
Coverage Local Elective Officials VII of RPC, or any offense involving fraud
who are under investigation
upon government or public funds or
property

Until the case is terminated but RA 3019 does not provide for a period
6-MONTH preventive suspension is
A single preventive suspension cannot maximum In Rios v. Sandiganbayan, the Court used
Period of exceed 60 days the 60-day limit for local elective officials
Suspension 90-day limit in case of several complaints The Court in Miranda v. Sandiganbayan in the LGC
under specific circumstances, see page 16 ruled that the 60-days cap in LGC does not
apply to the OMB’s authority to impose 6- But in Segovia v. Sandiganbayan, the Court
month suspension held that it must not exceed 90 days as
per Sec. 52 Admin Code

NOTES:
- Remember, in Hagad v. Gozo-Dadole, the SC held that there is nothing in the LGC to indicate that it has repealed the pertinent provisions of the
Ombudsman Act
 Two statutes are not inconsistent nor irreconcilable
 The Office of the President and the Ombudsman have concurrent jurisdiction in the matter of conducting administrative investigation and
the imposition of preventive suspension
- See Llorente Jr v. Sandiganbayan for the rule on salary grades in determining jurisdiction of Ombudsman
EFFECTIVITY OF RECALL AND LIMITATIONS
RECALL From Sec. 72 – 75 LGC

POWER OF RECALL Effectivity of Recall (Sec. 72)


- Recall of an elective local official shall be effective only upon
Nature of Recall the election and proclamation of a successor in the person of
- See Garcia v. Comelec the candidate receiving the highest number of votes cast
- Recall is a mode of removal of a public officer by the people during recall election
before the end of his term of office - If official sought to be recalled receives highest votes:
- People’s prerogative to remove a public officer is an incident  Confidence in him is affirmed
of their sovereign power and in the absence of constitutional  He shall continue in office
restraint, the power is implied in all governmental
operations Prohibition from Resignation (Sec. 73)
- The elective local official sought to be recalled shall NOT be
Who Exercises Recall Power? Sec. 69, LGC allowed to resign while the recall process is in progress
- The REGISTERED VOTERS of LGU to which the local elective
official subject to such recall belongs Limitations on Recall (Sec. 74) – NO recall shall take place:
1. Within 1 year from date of the official’s assumption to office,
Ground for Recall: LOSS OF CONFIDENCE (Sec. 69) or
- This is the sole ground 2. Within 1 year immediately preceding a regular local election
- In Evardone v. Comelec:
 Defined loss of confidence as “the formal withdrawal by “Next Regular Elections” is defined in Paras v. Comelec – One referring
an electorate of their trust in a person’s ability to to an election where the office held by the local elective official sought
discharge his office previously bestowed on him by the to be recalled will be contested and be filled by the electorate
same electorate”
 SC also said said that loss of confidence is a POLITICAL In Claudio v. Comelec, the Court held that the term “recall” in Sec. 73(b)
QUESTION refers to the recall election and not to the preliminary proceeding to
initiate recall.
Who may be the subject of Recall? Any ELECTIVE provincial, city, - So the people can meet and discuss on the possibility of recall
municipal, or barangay official (Sec. 70(a)) even within the 1 year ban from date of assumption to office
of the official as long as they initiate the recall (by filing the
petition) AFTER the 1st year of office
INITIATION OF RECALL PROCESS
See Sec. 70 LGC as amended by RA 9244 Expenses incident to Recall Elections
- All expenses incident to recall elections shall be borne by the
Recall shall be commenced by a PETITION: COMELEC
1. Of a registered voter in the LGU concerned - The GAA shall include a contingency fund at the disposal of
2. Supported by the registered voters in the LGU concerned the COMELEC for the conduct of recall elections
subject to the following percentage requirements:

VOTING POPULATION MINIMUM PERCENTAGE


OF LGU REQUIREMENT
Not more than 20,000 25%
20%
At least 20,000
But in no case shall the required
But not more than 75,000
petitioners be less than 5,000
15%
At least 75,000
But in no case shall the required
But not more than
number of petitioners be less than
300,000
15,000
10%
Over 300,000 But in no case shall the required
petitioners be less than 45,000

NOTE: Take note of the case of Angobung v. Comelec which declared


that the petition cannot be filed by one person only.

Contents of the Written Petition (Sec. 70(b))


1. Names and addresses of the petitioners written in legible
form and their signatures
2. The barangay, city or municipality, local legislative district,
and province to which the petitioners belong
3. Name of the official sought to be recalled; and
4. Brief narration of the reasons and justifications therefor

THE RECALL PROCESS


See Flowchart on next page

NOTE: RA 9244 eliminated the preparatory recall assembly as a mode


of instituting recall of elective local government officials
- For this purpose, RA 9244 amended Sec. 70 and 71 of LGC
EVARDONE v. COMELEC ANGOBUNG v. COMELEC
December 2, 1991 March 5, 1997

Summary: Evardone was the elected mayor of the Municipality Sulat in Summary: A petition for recall was filed by the losing mayoralty
Eastern Samar. A petition for recall was filed against him. COMELEC candidate against Mayor Angobung. Comelec recommended the
issued a resolution for the conduct of the signing of the petition for the approval of the petition and the Comelec En Banc issued a Resolution
recall of Evardone on July 14, 1990. Evardone filed a petition for approving the same, setting the signing process and scheduling the
prohibition. SC issued a TRO dated July 12, 1990 which was received by recall election. Mayor Angobung attacks the validity of the Resolution
the COMELEC field agent on July 15, 1990. The signing was conducted on the ground, among others, that the petition was void for it was not
in good faith. Evardone now assails the constitutionality of Resolution signed by at least 25% of the registered voters as required by Sec. 69(d)
2272 embodying the rules for the conduct of the signing of the petition of the LGC. The SC agreed with Angobung and held that the petition
for recall of a local official. He argues that the 1987 Constitution should be filed by at least 25% of the registered voters and not just by
repealed the old Local Government Code(BP 337). Hence, there is no one person.
law from which Resolution 2272 was based. SC ruled that the old LGC
was not repealed by the 1987 Constitution. Thus, it remains in force as Doctrine: Sec. 69(d) LGC provides that recall of any elective municipal
of the moment. The old LGC was repealed by the LGC of 1991 which official may also be validly initiated upon petition of at least 25% of the
took effect on January 1, 1992. total number of registered voters in the LGU concerned during the
election in which the local official sought to be recalled was elected. The
Doctrine: Sec. 54-59 of BP 337 provides for the mechanism for recall of law is plain and unequivocal! The initiatory recall proceedings must be
local elective officials. Under Sec. 59, Comelec is authorized to conduct a petition of at least 25% of the total number of registered voters.
and supervise the process of and election on recall; it may promulgate
necessary rules and regulations. The confusion is understandable since the signing of the petition is
statutorily required to be undertaken “before the election registrar or
Election Code contains no special provisions on recall. Thus, any such his representative, and in the presence of a representative….” THUS,
election shall be conducted in the manner and under the rules on while the initiatory recall petition MAY NOT YET contain the signatures
special elections, unless otherwise provided by law or rule of Comelec. of at least 25% of total number of registered voters, it must contain the
Pursuant to the express grant of rule-making power of the Comelec, it names of at least 25% of the total number of registered voters in
promulgated Res. 2272. THUS, Res. 2272 as promulgated is valid and WHOSE BEHALF only one person may sign the petition in the
constitutional. Comelec had the authority to approve the petition for meantime.
recall and set the date for signing.

PARAS v. COMELEC CLAUDIO v. COMELEC


November 4, 1996 May 4, 2000

Summary: Petitioner Danilo Paras is the incumbent Punong Barangay of Summary: Jovit Claudio is the Mayor of Pasay City. The Barangay
Pula, Cabanatuan City. A petition for his recall was filed and the Chairmans, Kagawads, and Sangguniang Kabataan of Pasay convened to
COMELEC approved the same. A petition signing was scheduled and create the Preparatory Recall Assembly of Pasay City. Claudio argued
29.30% of the registered voters signed the petition, which is above the that preparations for his recall were laid down prior to the one year
25% requirement provided by law. The recall election was however prohibition stated in Section 74, LGC. He contends that “recall” as stated
deferred because of several oppositions made by Paras. In the present in the aforesaid provision refers to all the initiatory acts of recall to the
petition, Paras cited Sec. 74(b) of RA 7160 which prohibits the conduct filing to the election. SC ruled that “recall” refers to the actual day of
of recall within 1 year immediately preceding a regular election. In this recall election.
case, petitioner insists that the recall election is barred as the
Sangguniang Kabataan election was set on the the first Monday of May Doctrine: The term "recall" in paragraph (b), Section 74 of the LGC
1996, which is 4 months away from the scheduled recall election refers to the recall election and not to the preliminary proceedings to
(January 1996). The SC upheld the validity of the recall election. The SK initiate recall.
election is not the “regular election” contemplated in Sec. 74(b) of RA
7160. To sum up, the term "recall" in paragraph (b) refers to the recall
election and not to the preliminary proceedings to initiate recall -
Doctrine: The SK election does not fall within the term “regular 1. Because §74 speaks of limitations on "recall" which,
election” contemplated in Sec. 74(b) of RA 7160. To subscribe to according to §69, is a power which shall be exercised by the
petitioner’s interpretation of the phrase regular local election to include registered voters of a local government unit. Since the voters
the SK election will unduly circumscribe the novel provision of the do not exercise such right except in an election, it is clear that
Local Government Code. In effect, there would be no more recall the initiation of recall proceedings is not prohibited within
elections allowed since the SK elections are to be held every three years the one-year period provided in paragraph (b);
from 1996 (it does not coincide with the elections of other regular local
officials). It would be more in keeping with the intent of the recall 2. Because the purpose of the first limitation in paragraph (b) is
provision of the Code to construe regular local election as one referring to provide voters a sufficient basis for judging an elective
to an election where the office held by the local elective official sought local official, and final judging is not done until the day of the
to be recalled will be contested and be filled by the electorate. election; and

Class Notes: It defined what “election” means under the concept of 3. Because to construe the limitation in paragraph (b) as
Recall. including the initiation of recall proceedings would unduly
- Since there is an overlap (considering that elective officials curtail freedom of speech and of assembly guaranteed in the
are not elected at the same time), it must refer to an election Constitution.
where the office held is sought to be recalled to ensure that
there is time to file the petition. Class Notes: Is Angobung good law?
- As to the process, no since there is a new one prescribed by
law
- NOTE that the petition must be WITH signatures
 Basis: Sec. 70(b)(2) on contents of the petition requires
the presence of signatures
AFIADO v. COMELEC
September 18, 2000

Summary: Navarro was elected as the Vice-Mayor of Santiago City.


Recall proceedings were instituted against her by Afiado and others.
The Preparatory Recall Assembly which convened for this purpose
passed PRA Resolution 1 to this effect. Before the COMELEC acted upon
PRA Resolution 1, Navarro became the new Mayor by the law on
succession. COMELEC, SC: PRA Resolution 1 is replete with statements
which show that the purpose of the assembly was to recall Navarro as
Vice Mayor for her official acts as Vice Mayor. Her succession to the
office of Mayor renders this proceeding moot and academic.

Doctrine: The vacating of a local official of his old position and the
assumption of a new position is a supervening event which renders
recall proceedings against him moot and academic when it appears that
the recall is intended for said local official in relation to his former
position.

Class Notes: How do you lose confidence in an elective official? By the


way he performs his functions and duties
HUMAN RESOURCES & DEVELOPMENT
Note: This part of the syllabus focuses more on cases. Just browse through the provisions.
The following discussion and notes are some key points in Pimentel’s commentary.

Compensation of Local Officials and Employees (Sec. 81)


ORGANIZATIONAL STRUCTURE - Determined by the Sanggunian concerned
- Elective local officials shall be entitled to the same leave
Overview of Sec. 76 to 78 LGC privileges as those enjoyed by appointive local officials
- Sanggunian also effects increases in compensation
Organizational Structure and Staffing Pattern of LGU (Sec. 76)
 When will the increase take effect?
- Every LGU shall design and implement its own organizational
1. If elective official – effective only after their terms
structure and staffing pattern
are over
 Take into consideration (1) service requirements, and 2. If appointive – effective as the ordinance provides
(2) financial capability
- SUBJECT to minimum standards and guidelines by the CSC Grievance Procedure (Sec. 83) - Local chief executives of every LGU
shall establish a procedure to inquire into, act upon, resolve or settle
NOTE: LGUs can create offices other than those mentioned in the LGC complaints and grievances presented by local government employees
- But subject to CSC rules and regulations

Responsibility for Human Resources and Development (Sec. 77)


- WHO? The local chief executive of every LGU
 For office structures and staffing patterns
RESIGNATION
- Limits to the powers of the local chief executive’s power:
1. Constitution
OF ELECTIVE LOCAL OFFICIALS
2. CSC Rules and Regulations Sec. 82 LGC
3. Other pertinent laws
When Effective:
Applicability of CSC Rules (Sec. 78) 1. ONLY UPON ACCEPTANCE
- All matters pertinent to human resources and development  To be effective, there MUST be acceptance
in LGUs shall be governed by the civil service law and such  Who accepts? The following authorities:
other rules and regulations promulgated pursuant thereto
ACCEPTING
NOTE: CSC Law and its IRR continue to protect the civil servant even if RESIGNING OFFICIAL
AUTHORITY
he and his office may be devolved to an LGU Governor
Vice-Governor
The President
Mayors of HUCs and ICCs
Vice-Mayors of HUCs and ICCs
APPOINTMENTS, VACANCIES, Mayors of CCs
Vice-Mayors of CCs
COMPENSATION, GRIEVANCE Mayors of Municipalities
Governor
Overview of Sec. 79 to 83 LGC Vice-Mayors of Municipalities
The Sanggunian
Sanggunian members
Limitation on Appointments (Sec. 79) – NO person shall be appointed in concerned
the CAREER SERVICE of the LGU if he is related within the 4 th civil Barangay officials City/Municipal Mayor
degree of consanguinity or affinity to the appointing or recommending
authority 2. Resignation shall also be deemed accepted if NOT acted upon
- What is a career service? As defined by the Admin Code of by the authority concerned within 15 days from receipt
1987, it is characterized by: thereof
1. Entrance based on merit and fitness to be determined
as far as practicable by competitive examination, or Formalities
based on highly technical qualifications; 1. The resignation must be in writing
2. Opportunity for advancement to higher career  Implied from the fact that Sec. 82(b) requires the
positions; and submission to the DILG of copies of the resignation
3. Security of tenure letter
- Coverage: 4th civil degree of relationship 2. DILG must be furnished with:
 FROM the appointing OR recommending authority a. Copies of resignation letters,
b. Together with the action taken by the aforesaid
Public Notice of Vacancy (Sec. 80) authorities
1. Whenever a local chief executive decides to fill a vacant 3. For IRREVOCABLE resignations by sanggunian members:
career position, posting of notices of vacancy is required a. Deemed accepted upon presentation before an OPEN
 In at least 3 conspicuous public places in the LGU session of the sanggunian concerned AND duly entered
 For a period not less than 15 days in its records
2. Every province, city or municipality shall have a PERSONNEL b. NOTE: This rule does not apply to:
SELECTION BOARD (1) Sanggunian members who are subject to recall
 Composition: elections or (see Sec. 73)
a. Headed by the local chief executive (2) Cases where existing laws prescribe the manner of
b. Members determined by the Sanggunian acting upon such resignations
c. CSC representative, if any, and personnel officer of
LGU as ex-officio members
When Resignation Not Allowed: Sec. 73 Execution Pending Appeal (Sec. 88)
- Note that Sec. 73 prohibits local elective officials who are - Appeal shall not prevent execution of a decision of removal
subject to recall elections from resigning while recall or suspension
elections are on the way - In case respondent is exonerated, he shall be reinstated to his
position with all rights and privileges appurtenant thereto
from the time he had been deprived thereof

ADMINISTRATIVE DISCIPLINE
AGAINST APPOINTIVE LOCAL OFFICIALS SECTIONS 91 – 97, LGC
Sec. 84 - 88 LGC Statement of Assets and Liabilities (Sec. 91)- Local government officials
and employees are required to file SWORN:
Administrative Discipline (Sec. 84) 1. Statements of assets, liabilities, and net worth
- Investigation and adjudication of administrative complaints 2. Lists of relatives within the f4th civil degree of consanguinity
against appointive local officials and employees as well as or affinity in government service
their suspension and removal 3. Financial and business interests
 Governed by CIVIL SERVICE LAWS and rules 4. Personnel data sheets as required by law
- Results of investigations reported to CSC
Oath of Office/Affirmation (Sec. 92)
Preventive Suspension of Appointive Local Officials and Employees - All ELECTIVE and APPOINTIVE local officials and employees
(Sec.85) shall, upon assumption to office, subscribe to an oath or
- NOTE: for elective local officials, follow Sec. 63 affirmation of office in the prescribed form
- Oath or affirmation to be filed with the office of the local chief
Who executive concerned
Local Chief Executive
suspends?  A copy shall be preserved in the individual personnel
Any subordinate official or employee under his records file under
Coverage
authority pending investigation
1. If the charge involves dishonesty, Partisan Political Activity (Sec. 93)
oppression, or grave misconduct, or - No local official or employee in the career civil service shall
neglect in the performance of duty engage directly or directly in any partisan political activity
Conditions
2. OR if there is reason to believe that the - But they may express their views on current issues or even
respondent is guilty of charges which mention names of the candidates
would warrant his removal from service
Period Not exceeding 60 days Appointment of Elective and Appointive Local Officials Candidates Who
Lost in an Election (Sec. 94)
Upon expiration of preventive suspension 1. No elective or local official shall be eligible for appointment
1. Suspended official or employee automatically reinstated in or designation in ANY capacity to any public office or
office without prejudice to continuation of proceedings position during his tenure
a. They shall not hold any other office or employment in
NOTE: If delay is due to the fault, neglect, or request of respondent, time any government or any of its subdivision, agency, or
of delay shall not be counted in computing period of suspension instrumentality, or GOCC
(1) UNLESS:
Administrative Investigation- PROCEDURE (Sec. 86) (a) Allowed by law4 or
1. May be conducted by a person or committee duly authorized (b) Allowed by the primary functions of his
by local chief executive position
2. They SHALL conduct hearings on cases against appointive
local officials or employees 2. No candidate who lost in any election shall, within 1 year
a. Submit findings and recommendations to local chief after such election, be appointed to any office in the
executive within 15 days from conclusion of hearings government or any GOCC or in any of their subsidiaries
3. Cases shall be decided within 90 days from time respondent a. EXCEPTION: Prohibition does not apply to losing
is formally notified of the charges candidates in barangay elections

Disciplinary Jurisdiction (Sec. 87) Additional or Double Compensation (Sec. 95)


1. Except as otherwise provided by law, the local chief - Local officials and employees are not authorized to receive
executive may impose the penalty of: additional, double, or indirect compensation
a. Removal from service  UNLESS specifically authorized by law
b. Demotion in rank - Neither can they accept any present, emolument, office or
c. Suspension for not more than 1 year without pay title of any kind from any foreign government without
d. Fine in an amount not exceeding 6 months’ salary consent of Congress
e. Reprimand - NOTE:
f. And otherwise discipline subordinate officials and  Pensions and gratuities shall not be considered as
employees under his jurisdiction additional, double, or indirect compensations
2. Appeals  Simple presents from visiting dignitaries are
a. If penalty is suspension without pay for not more than traditionally exempted from this prohibition (Pimentel)
30 days, decision is FINAL
b. If penalty imposed is heavier than suspension for 30
days, decision appealable to CSC
(1) CSC to decide appeal within 30 days from receipt
thereof 4
For example, a mayor may represent the NAPOLCOM in his city or
municipality because the law allows him to do so. Also, a mayor may hold
the position of Chairman of a Regional Development Council
Permission to Leave Station (Sec. 96) – This section requires that Government. There were two other employees who were considered
APPOINTIVE officials of the province, city, municipality, and barangay for the position. CSC received a letter from the Congressman on
secure the permission of their local chief executives whenever they go Victoria’s appointment and ordered a report to be submitted on it.
on official travel which must be secured before departure Acting on the report, CSC disapproved Victoria’s appointment because
- Failure of local chief executive to act on the request within 4 it violated the statutory prohibition against nepotic appointments. The
working days from receipt thereof, it shall be deemed Debulgados assailed the disapproval, saying that the prohibition against
approved nepotic appointments does not apply in the case, and that Victoria
cannot be removed from office without the opportunity to be heard.
Annual Report (Sec. 97) – every local chief executive shall submit an Victoria was qualified and highly-recommended by the OIC Treasurer of
annual report to the sanggunian concerned on or before March 31 of the City. The Sangguniang Panlugnsod also concurred with her
each year on the conditions of the LGU for the period covering the appointment. The Mayor also said that one Agadon, a personnel
immediately preceding year specialist o the CSC. Informed him that the promotional appointment
was not covered by the prohibition because it was a promotion and not
an appointment. The SC ruled against the Debulgados and said that the
Special Laws cited in the Syllabus prohibition in Sec. 59, Book V of the Revised Administrative Code was
A. See also Sec. 469-490 LGC – These provisions refer to the cast in comprehensive and unqualified terms and should be construed
appointive local officials common to all municipalities, cities, as it is worded.
and provinces
DOCTRINE: The original appointment of a civil service employee and
B. RA 6713 – Code of Conduct and Ethical Standards for Public all subsequent personnel actions undertaken by or in respect of that
Officials and Employees; (see special laws compilation) employee such as promotion, transfer, reinstatement, reemployment,
etc., must comply with the Implementing Rules. The prohibition applies
without regard to the actual merits of the proposed appointee and to
MACALINAG v. CHANG the good intentions of the appointing or recommending authority. The
May 6, 1992 prohibition against nepotism in appointments whether original or
promotional, is not intended by the legislative authority to penalize
Summary: Acting secretary of Finance Macalincag issued an order of faithful service.
preventive suspension against Roberto Chang. Chang filed a petition for
prohibition with writ of preliminary injunction before the RTC. Chang The purpose of Section 59 is to take out of the discretion of the
contends that EO 392 transferred the power to suspend from the appointing and recommending authority the matter of appointing or
Secretary of Finance to the President of the Philippines. RTC sided with recommending for appointment a relative
Chang. SC reversed.
Class Notes: Here’s a detailed discussion by the SC:
Doctrine: Secretary of Finance is an alter ego of the President and 1. Prohibitory norm against nepotism in public service is in Sec.
therefore, it is within his authority, as an alter ego, to preventively 59, Book V, RAC
suspend the acting municipal treasurer of Makati. a. It says that “All appointments in the national,
provincial, city and municipal governments … made in
Preventive suspension is governed by Sec. 41 of PD 807 or the Civil favor of a relative (within 3rd degree) of the appointing
Service. In the provision, it will be noted that under the aforesaid law, or recommending authority, or of the chief of
designation of the replacement is not a requirement to give effect to the bureau/office, or of persons exercising immediate
preventive suspension. supervision over him, are hereby prohibited”
2. Sec. 6, Rule XVIII RAC IRR
On the contrary, Section 156, Article 5, Chapter 3, Title II LGC provides a. Substantially the same as Sec. 59
for the automatic assumption of the assistant municipal treasurer or b. In fact substantially identical form and language for at
next in rank officer in case of suspension of the municipal treasurer. least 30 years
3. Textual examination of Sec. 59 reveals its comprehensive
Class Notes: nature:
1. When we deal with appointive officials, what laws apply? a. First, it covers ALL appointments explicitly
a. CSC Laws, Revised Administrative Code, and the Local (1) No distinction as to kinds or types
Government Code b. Second, it covers all appointments in the national,
provincial, city, and municipal governments, as well as
2. Why are they treated differently from elective local officials? any branch or instrumentality thereof and all GOCCs
a. For one, the electorate chose the local elective officials c. Third, there is a short list of exceptions which is
while appointments are discretionary on the part of the EXCLUSIVE
appointing authority (1) Persons employed in a confidential capacity
b. More importantly, appointive officials are expressly (2) Teachers
covered by civil service laws (3) Physicians
(4) Members of AFP
3. Is Macalinag good law?
a. YES! They retained the power of the Finance Secretary
over the local treasuries CIVIL SERVICE COMMISSION v. TINAYA
b. This is excluded from the devolution due to the February 16. 2005
sensitive nature of handling with finances.
Summary: Tinaya was appointed by the Mayor as city assessor, the
appointment was permanent. The CSC approved the appointment but
DEBULGADO v. CIVIL SERVICE COMMISSION only as temporary appointment because Tinaya is lacking some
September 26, 1994 requirements. 15 days after the approval, Tinaya married the Mayor’s
daughter. After the expiration of the temporary appointment, Tinaya
Summary: Mayor Debulgado of the City of San Carlos, Negros was reappointed by the acting mayor, this was initially disapproved by
Occidental, appointed his wife Victoria Debulgado as General Services the CSC but the mayor appealed which resulted in the approval. The
Officer or head of the Office of General Services of the City of San Carlos, said reappointment was permanent in character. CSC discovered that
after rising from the ranks in her 32 years of service in the City the reappointment was in violation of the law against nepotism and
recalled it. The Court ruled that the original appointment was only or by the primary functions of his position, the first paragraph appears
temporary because the CSC’s approval was necessary to make it to be more stringent by not providing any exception to the rule against
permanent while the reappointment, although permanent, was in appointment or designation of an elective official to the government
violation of the law on nepotism. post, except as are particularly recognized in the Constitution itself, e.g.,
the President as head of the economic and planning agency; the Vice-
Doctrine: While the appointing authority has the discretion to choose President, who may be appointed Member of the Cabinet; and, a
whom to appoint, the choice is subject to the caveat that the appointee member of Congress who may be designated ex officio member of the
possesses the required qualifications. Judicial and Bar Council.

The Civil Service Commission is empowered to take appropriate action Sec. 7 of Art. IX-B should not be confused with Sec. 13, Art. VI, of the
on all appointments and other personnel actions and that such power Constitution where "no Senator or Member of the House of
“includes the authority to recall an appointment initially approved in Representatives may hold any other office or employment in the
disregard of applicable provisions of the Civil Service law and Government . . . during his term without forfeiting his seat . . . ." The
regulations difference between the two provisions is significant in the sense that
incumbent national legislators lose their elective posts only after they
have been appointed to another government office, while other
DIMAANDAL v. COA incumbent elective officials must first resign their posts before they can
June 26, 1998 be appointed, thus running the risk of losing the elective post as well as
not being appointed to the other post.
Summary: Zosimo Dimaandal was designated as Provincial Treasurer
for Administration by the Governor of Batnagas. Pursuant to such, he Class Notes: Sec. 94 LGC provides for a contentious provision. Sec. 94
filed a claim for the difference in salary and RATA due for the position LGC permits appointment of local elective official to another post if so
he was designated to. The Provincial Auditor disallowed such, only allowed by law or by primary functions of his office
allowing the allowances attached to the designated position. The - But the SC said that Sec. 94 LGC is not determinative of
disallowance was premised on, among other things, the Governor’s constitutionality of Sec. 13(d)
absence of authority in designating Dimaandal to the position of - Sir said that Sec. 94 MAY be struck down but no case yet.
Assistant Provincial Treasurer for Administration and the fact that he
was merely designated. Upon reconsideration, the COA diallowed the
RATA and salary on the same grounds. The SC affirmed the COA. SANGGUNIANG BAYAN OF SAN ANDRES, CATANDUANES v. CA
January 16, 1998
Doctrine: Under Sec. 471(a) of the LGC , in relation to Sec. 2077 of the
RAC , the Provincial Governor is not authorized to appoint or even Summary: Barangay Captain was elected president of Association of
designate a provincial officer in case of absence, disability or a vacancy Barangay Councils and in line with that position was appointed by the
in a provincial office. That power resides in the President of the President as member of the Sangguniang Bayan of the Municipality of
Philippines or the Secretary [of Finance]. San Andres. Due to lack of quorum in the Federation of the Association
of Barangay Councils, the DILG secretary temporarily designated him to
De facto officer: one who is in possession of an office in the open the position in the Sangguniang Panlalawigan. He then resigned from
exercise of its functions under color of an election or an appointment, Sangguniang Bayan. The vice president of the ABC was then appointed
even though such election or appointment may be irregular. to the post he vacated. Then he tried to reassume his office with the
Sangguniang Bayan. RTC and CA held that his resignation was not
[P]ending approval of said appointment by the President, the appointee properly accepted by authorities hence it wasn’t a valid resignation,
may assume office and receive salary for services actually rendered [Cui enabling him to reassume his post. SC Reversed.
et al. v. Ortiz]
Doctrine: Although a resignation is not complete without an acceptance
Class Notes: Note the difference between appointment and designation thereof by the proper authority, an office may still be deemed
relinquished through voluntary abandonment which needs no
APPOINTMENT DESIGNATION acceptance.
Merely connotes an imposition of
additional duties, usually by law, upon To constitute a complete and operative resignation from public office,
Selection by proper there must be: (a) an intention to relinquish a part of the term; (b) an
a person already in public service by
authority of an individual act of relinquishment; and (c) an acceptance by the proper authority.
virtue of an earlier appointment
who is to exercise powers The last one is required by reason of Article 238 of the Revised Penal
Imposition of new or additional duties
and functions of a given Code.
to be performed in a special manner
office
Does not entail payment of additional
benefits or grant There are two essential elements of abandonment: first, an intention to
abandon and, second, an overt or "external" act by which the intention
is carried into effect.
FLORES v. DRILON
June 22, 1993
MATHAY v. CIVIL SERVICE COMMISSION
Summary: Mayor Richard Gordon was appointed as Chairman and Chief August 9, 1999
Executive Officer of the Subic Bay Metropolitan Authority pursuant to
Sec 13(d) of RA 7227. Petitioners assail the validity of Sec 13(d) for Summary: Mayor Mathay appointed Tabernilla to the newly created
being contrary to the constitutional prohibition on the appointment of position of Electrical Engineer V. Under the Ordinance which created
any elective official to any public office or position during his tenure. the new position, the only qualification for that position is to be a
The Court held the subject proviso unconstitutional. Professional Electrical Engineer which according to law requires only
two years of resident collegiate engineering training. However, under
Doctrine: The view that an elective official may be appointed to another CSC Memorandum Circular, a bachelor’s degree in engineering is a
post if allowed by law or by the primary functions of his office, ignores qualification for that position. Enriquez, another engineer aiming for
the clear-cut difference in the wording of the two (2) paragraphs of Sec. the position questioned the appointment of Tabernilla before the CSC.
7, Art. IX-B, of the Constitution. While the second paragraph authorizes CSC revoked the appointment despite being approved initially by the
holding of multiple offices by an appointive official when allowed by law regional director. MRs were filed but were denied. CA also denied the
petition for certiorari. SC upheld the validity of the resolutions revoking DE RAMA v. CA
the appointment of Tabernilla and denying the MRs. At the time the February 28, 2001
questioned CSC resolution was issued, the remedy available for
petitioner is certiorari brought to the SC within 30 days after the Summary: Fourteen municipal employees were allegedly “midnight
receipt of the resolution. The petitioner failed to do this. Thus, the appointees” of the former town Mayor. So, when Mayor De Rama
questioned CSC resolution had long attained finality. SC upheld the assumed office, the first thing he did was to request their recall from
authority of CSC to take appropriate action on all appointments and the CSC, on the solitary ground that they were midnight appointees.
other personnel actions which includes the authority to recall an However, when the CSC ruled that the prohibition on midnight
appointment initially approved in disregard of applicable provisions of appointments applies only to appointments by the outgoing President,
Civil Service law and regulations. De Rama filed a supplemental pleading before the CSC, alleging for the
first time that the appointments suffered some irregularities for failure
Doctrine: CSC has the power to determine the qualification standards to comply with CSC rules and regulations. The CSC held that such did
for the various positions in the local government and review whether not constitute new evidence, and denied De Rama’s petition. The CA
the appointments meet these standards. The qualification standards and SC affirmed. The SC held that the fourteen employees, having
for new offices, which local governments have the authority to create, already been lawfully and regularly appointed, have acquired a legal
must not be lower than those prescribed by the CSC. right to their positions, and cannot just be recalled without cause.

Notwithstanding the initial approval of an appointment, the same may Doctrine: Upon the issuance of an appointment, and the appointee’s
be recalled by the Civil Service Commission for violation of other assumption of the position in the civil service, the appointee acquires a
existing Civil Service laws, rules and regulations legal (not merely equitable) right which cannot be taken away by
revocation of the appointment or removal, except for cause and with
previous notice and hearing.
MATHAY v. CA
December 15, 1999 It is the CSC that is authorized to recall an appointment initially
approved, and even then, only when such appointment and approval
Summary: Mayor Simon appointed personnel to positions in Civil are proven to be in disregard of applicable provisions of the civil
Service Unit of the local government of QC. Civil Service Units were service law and regulations.
created under PD 51. The Sec. of Justice found that this was never
published in the Official Gazette. Thus, it is deemed never “in force or
effect and therefore cannot at present, be a basis for establishment of LAMEYRA v. PANGILINAN
the CSUs …” June 18, 2000

CSC issued a memo directing recall and revocation all appointments Summary: Mayor Pangilinan dropped Lameyra from the roll of
under PD 51. For QC CSU employees, the effects were cushioned by City employees of pursuant to the Memorandum Circular No. 12, Series of
Ordinance No. NC-140, Series of 1990, which established the 1994 of the Civil Service Commission because of Insubordination and
Department of Public Order and Safety (“DPOS”). Section 3 provides: AWOL. CSC affirmed the Mayor’s decision. MR was filed alleging that he
Sec. 3. The present personnel of the Civil Security Unit, had not earlier been furnished copy of Mayor’s comment reported for
Traffic Management Unit, Anti-Squatting and Surveillance and work at the office of the Vice Mayor Constancio Fernandez, as he was
Enforcement Team, and Disaster Coordinating Council are hereby not allowed by the Personnel Officer, Benito Vicencio, to sign his name
absorbed into the department of public order and safety established… in log book. MR was denied. CA affirmed denial. SC reversed stating that
he should be given a last full opportunity to prove his contention that
Mayor Simon offered respondents temporary contractual the termination of his service was illegal.
appointments. Mayor Mathay, Jr. was subsequently elected Mayor, but
did not appoint respondents. The case was brought to CSC and CSC Doctrine: It is clear that no prior notice is required to drop from the
order the Mayor to reappoint the respondents. CA affirmed CSC, but SC rolls an employee who has been continuously absent AWOL at least 30
reversed. calendar days. While it is settled doctrine findings of fact of an
administrative agency must be respected and this Court should not be
Doctrine: The law applicable is B.P. 337 or the old Local Government tasked to weight once more the evidence submitted before the
Code. Section 3 of the Ordinance is invalid for being inconsistent with administrative body, it is axiomatic that such findings of facts should be
this law. The power to appoint rests exclusively with the local chief supported by substantial evidence. The certification of the personnel
executive and cannot be usurped by the city council or sanggunian officer does not constitute substantial evidence that Lameyra did not
through enacting ordinances that provide for the “absorption” of report for work from July 6, 1995 to August 6, 1995 in light of the his
specific persons to certain positions. submission that said personnel officer precisely prevented him from
signing the log book , that he has been replaced by one Leynes in July,
The power of the city council is limited to creating, consolidating and 1995, and that he has been asked to submit his resignation which he
reorganizing city officers and positions supported by local funds. The refused to do. He should be given a last full opportunity to prove his
city council has no power to appoint. Also, the CSC cannot order the contention that the termination of his service was illegal.
mayor to “reinstate” since its power is limited to approving or
disapproving an appointment. It does not have the authority to direct
that an appointment of a specific individual be made. The power of CITY GOVERNMENT OF MAKATI CITY v. CIVIL SERVICE
appointment is a discretionary power and cannot be controlled even by COMMISSION
the courts as long as it is properly and not arbitrarily exercised by the February 6, 2002
appointing authority.
Summary: Galzote, a clerk in the City Government of Makati City, was
Even assuming the validity of the Ordinance, the absorption is not detained for more than three years pursuant to a criminal case filed
possible since the CSU never legally came into existence. The against her for kidnapping for ransom with physical injuries. Because
appointments were invalid ab initio. could not report for work, the City Government of Makati City, through
its Municipal Personnel Officer (MPO), suspended her from office “until
the final disposition of her case.” While the trial for her criminal case
was ongoing, the MPO lifted her suspension and removed her from her
position on the ground that, because of her detention and her failure to
file an application for leave of absence under Secs. 20 and 35 of the Civil
Service Commission Rules, she was on absence without approved leave GARCIA v. PAJARO
(AWOL). July 5, 2002

The removal was done without prior notice served on Galzote and Summary: Garcia was employed as a Revenue Collector of Dagupan City
without her knowledge. After her acquittal and release from detention, for 16 years until he was suspended by the City treasurer for two years.
Galzote requested the City Government, for the lifting of her suspension During such period, his salary was withdrawn. Several investigations
and for her reinstatement. Galzote sought recourse in the CSC, which and subpoenas were issued but Garcia refused to attend and to honor
ordered her immediate reinstatement. The CA affirmed the SC. the investigation claiming that the City treasurer had no authority to
discipline and investigate him because under the LGC, such power is
Doctrine: On how an automatic leave of absence is an exception to the vested with the mayor. He filed this present claim for damages alleging
general rule requiring application for leave absence: As a general rule, sleepless nights and mental anguish caused by charging him with
Secs. 52 and 63 require an approved leave of absence to avoid being on neglect of duties.
AWOL. They require an employee to apply for an application for leave
of absence formalities and all, before exceeding thirty (30) days of The SC ruled in favor of the City Treasurer Pajaro and upheld the power
absence in order to avoid being dropped from the rolls. However, these of the City Treasurer to institute disciplinary actions against
provisions cannot be interpreted as EXCLUSIVE and referring only to subordinate officers or employees.
one mode of securing the approval of a leave of absence. There are
other means of seeking and granting an approved leave of absence, one Doctrine: Under Section 455 (b-1-x) 28 of the 1991 LGC, the city mayor
of which is the CSC recognized rule of AUTOMATIC LEAVE OF ABSENCE "may cause to be instituted administrative or judicial proceedings
under specified circumstances. against any official or employee of the city." This rule is not inconsistent
with the 1987 Administrative Code, which authorizes the heads of
On the extent of the power of the CSC to interpret: It is hinted that the agencies to discipline subordinate employees. Likewise, the old LGC
purported automatic leave of absence is a NON-EXISTENT rule in the does not vest in city mayors the sole power to discipline and to institute
CSC Rules and hence the CSC has no power to interpret such non- criminal or administrative actions against any officers or employees
existent provision; further, that the CSC has no power to provide for under their jurisdiction.
exemptions since none is stated in the CSC rules.
My restatement: The power to discipline employees of the city was
Actually, what the CSC interpreted in the instant case were Secs. 52, 60, granted by the LGC to the city mayor. However, this grant of power
63 and other related provisions of the CSC rules on the requirement of does not exclude the exercise by other public officials of their power to
an approved leave of absence. The CSC Rules do not limit the powers of discipline their subordinates.
the CSC to allow exemptions in the cases of illness only, although it is
the only exemption seen in the text of the rules.

With reasonableness as the standard, the CSC is far from being


presumptuous when it states that other instances of force majeure
(such as the arrest and detention of a civil servant for a crime she did
not commit) may excuse the prior filing of an approved leave of
absence.

This power logically flows from the task of the CSC to regulate civil
service in the country as ordained in the Constitution and mandated in
the Administrative Code of 1987.

PASTOR v. CITY OF PASIG


May 9, 2002

Summary: Pastor was the Budget Officer of Pasig. Through Mayor


Eusebio’s Memo, she was reassigned to the Office of the Municipal
Administrator due to a report that she issued Advice of Allotments w/o
sufficient cash collections. After 3 years, no investigation re: the charge
against her was made. Because of this, she a complaint with the CSC,
praying for reinstatement to the Budget Officer position. CSC ruled in
favour of Pastor. Instead of filing an MR, Pasig designated Pastor as
head of the Pasig City Hall Annex. Pastor was not satisfied and so she
asked the CSC for a clarification of its resolution. CSC held that Pastor’s
reassignment was not in compliance with its order because the position
was not equivalent to her original position. Upon appeal, the CA set
aside the CSC decision and held that Pasig substantially complied with
the CSC Resolution. Pastor filed a certiorari petition with the SC. The SC,
finding that Pastor’s reassignment was indefinite and there was a
diminution in rank, ruled in her favour and ordered Pasig to reinstate
her to the position of Budget Officer.

Doctrine: A reassignment that is indefinite and results in a reduction in


rank, status, and salary is in effect a CONSTRUCTIVE REMOVAL from
the service.
PROHIBITED BUSINESS PRACTICE OF PROFESSION
AND PECUNIARY INTEREST Sec. 90 LGC
Sec. 89 LGC ABSOLUTE PROHIBITION: The following officials are prohibited from
practicing their profession or engaging in any occupation other than the
It shall be unlawful for ANY local government official or employee, exercise of their functions as local chief executives:
DIRECTLY or INDIRECTLY to: 1. Governors
1. ENGAGE in any business transaction with: 2. City Mayors
a. The LGU in which he is an official or employee or 3. Municipal Mayors
b. The LGU over which he has the power of supervision
c. Any of its authorized boards, officials, agents, or RELATIVE PROHIBITION: Sanggunian Members
attorneys whereby money is paid, or property or any A. General Rule: They MAY practice their professions, engage in
other thing of value is too be transferred, directly or any occupation, or teach in schools
indirectly, out of the resources of the LGU to such 1. Exception: They cannot do so during SESSION HOURS
person or firm B. Special Rules:
2. HOLD such interests in any: 1. For sanggunian members who are also lawyers, they
a. Cockpit or SHALL NOT:
b. Other games licensed by LGU a. Appear as counsel in ANY civil case
3. PURCHASE any real estate or other property forfeited in (1) Wherein LGU or any office, instrumentality,
favor of such LGU: or agency of the government is the adverse
a. For unpaid taxes, or party
b. By virtue of legal process at the instance of LGU b. Appear as counsel in ANY criminal case
4. Be a SUREY for any person contracting or doing business (1) Wherein an officer or employee of the
with the LGU for which a surety is required national of local government is accused of an
5. POSSESS or use any public property of the LGU for private offense committed in relation to his public
purposes office
c. Collect any fee for their appearance in
All other prohibitions governing the conduct of national public officers administrative proceedings involving the LGU of
relating to prohibited business and pecuniary interest so provided which he is an official
under RA 6713 (Code of Conduct for Public Officials and Employees) d. Use property and personnel of the government
and other laws shall also be applicable to LGU officials and employees. (1) EXCEPT when the sanggunian member
concerned is defending the interest of the
government
TEVES v. SANDIGANBAYAN
December 17, 2004 2. For sanggunian members who are also DOCTORS OF
MEDICINE
Summary: Edgar Teves and his wife Teresita were charged with the 1 st a. They may practice their profession EVEN during
mode of violating Section 3(h) of the Anti-Graft Law but were convicted official hours of work ONLY on occasions of
of the offense proved which is the 2nd mode of violating said provision emergency
by applying the variance doctrine which provides that when there is a (1) BUT they cannot derive monetary
variance between the offense charged and that proved, and the offense compensation therefrom
as charged is included in or necessarily includes the offense proved, the
accused shall be convicted of the offense proved which is included in Class Notes: What is the penalty for violating this provision?
the offense charged, or of the offense charged which is included in the - The LGC provides no penal sanction
offense proved.  Maybe the LGC does not intend to penalize it?
 Or maybe we can anchor the penalty on some other
Doctrine: The SC held that the Variance Doctrine applies since The first statute like the CPR, or Code of Ethics of the professions
and second elements of the offense charged constitute the offense
proved. But there are two laws that punish prohibited interests of
public officials, the Anti-Graft Law and the LGC. The LGC, which is a JAVELLANA v. DILG
special law that specifically prohibits local officials from possessing August 10, 1992
pecuniary interest in a cockpit licensed by the local government unit
and which, in itself, prescribes the punishment for violation thereof, is Summary: An administrative case was filed against Javellana, Bago City
paramount to the Anti-Graft Law, which penalizes possession of Sanggunian member, for representing Javiero and Catapang in an illegal
prohibited interest in a general manner. Moreover, the LGC is a later dismissal case against City Engineer Divinagracia, based on circulars of
law than the Anti-Graft Law. the DLG requiring prior authorization for private employment or
practice of profession and prohibiting practice of law adverse to the
Class Notes: Penalty for violating this provision? Sec. 514 LGC interest of the LGU. Javellana moved to dismiss on the grounds that the
- Persons Liable: circulars are unconstitutional for (1) impairing the authority of the
1. Any local official AND Supreme Court to regulate the practice of law and (2) being an invalid
2. Any person/s dealing with him class legislation against officials who are lawyers and doctors. The DLG
- Penalties dismissed. The Supreme Court affirmed the dismissal, ruling that the
1. Imprisonment for 6 months and 1 day to 6 years, OR circulars, along with Section 90, LGC (1) validly regulate the practice of
2. A fine of not less than P3,000 nor more than P10,000 profession allowed by law to avoid conflict of interest, and (2) provides
3. Or both, at the discretion of the court particular limitations to the practice of law since it is likely to affect
public service.

Doctrine: No violation of Art. VIII, Sec. 5. Neither the statute nor the
circulars trenches upon the SC’s power and authority to prescribe rules
on the practice of law. They simply prescribe rules of conduct of pub
offs to avoid conflicts of interest between private practice and public Doctrine: Punong Barangays who are also lawyers are not prohibited
office from practicing their profession. Sec. 7(b) (2) of RA 6713 does not
apply to them because it is only a general law that applies to the private
The circular is non-discriminatory. It applies to all provincial and practice of profession by public officials and employees. Sec. 90 of RA
municipal officials in professions or occupation. Sec. 90 explicitly 7160, on the other hand, is a special provision that applies specifically
provides that sanggunian members may practice their professions, to the practice of profession by elective local officials. Under Sec. 90 of
engage in occupation, or teach in schools except during session hours. If RA 7160, there was no express mention of Punong Barangays. Since
there are prohibitions that apply particularly to lawyer, it is because of they are excluded from the prohibition, the presumption is that they are
all professions, practice of law is more likely than others to relate to, or allowed to practice their profession which stems from the fact that they
affect, area of pub service. are not mandated to serve full time. However, they are required to
obtain prior permission or authorization from the head of his
Department (DILG) as required by civil service regulation (Sec. 12, Rule
XVIII Revised Civil Service Rules).
SOCIAL JUSTICE SOCIETY v. LINA
December 18, 2008

Summary: Social Justice Society, a registered political party, filed an REPUPLIC v. RAMBUYONG
petition for declaratory relief against the DILG Secretary, for the proper October 4, 2010
construction of Sec. 90 of RA 7160. Said section states that “All
governors, city and municipal mayors are prohibited from practicing Summary: Alfredo Chu filed a case for collection of sum of money
their profession or engaging in any occupation other than the exercise and/or damages against the NPC. Appearing as his counsel was Atty.
of their functions as local chief executives.” Thereafter, SJS impleaded as Rambuyong who was then the incumbent Vice Mayor of Ipil,
additional respondents Lipa City Mayor Vilma Santos, Pampanga Zamboanga Sibugay. NPC moved to inhibit Rambuyong on the ground
Provincial Governor Lito Lapid, and Paranaque City Mayor Joey that he is disqualified from appearing as counsel pursuant to Sec. 90(b)
Marquez. SJS argues that said respondents cannot appear in movies and of the LGC. SC agreed with NPC and held that NPC is a GOCC which falls
television programs for this would give them undue advantage over under the term “instrumentality” in Sec. 90(b). Since Rambuyong was
their political opponents and would reduce the time they must devote the Vice Mayor who presides over sanggunian meetings, he cannot be a
to their constituents. The RTC and SC dismissed the petition for counsel of a party where the adverse party is the government.
declaratory relief.
Doctrine: Sec. 2 of Admin Code is clear and unambiguous.
Doctrine: A petition for declaratory relief is not an appropriate remedy “Instrumentality” includes GOCCs. It is undisputed that NPC is a GOCC
to enforce compliance with Sec. 90 of RA 7160 and is not an so following Sec. 2, it is an instrumentality of the government. Following
appropriate remedy to prevent local chief executives from taking roles Maceda v. Macaraig, Jr., NPC is a governmental instrumentality. Given
in movies and television shows. the categorical words of the law and jurisprudence, to hold that NPC (or
GOCCs for that matter) is not included within the term “instrumentality
For a declaratory relief action to prosper, it must be shown that (1) of the government” is grave abuse of discretion.
there is a justiciable controversy; (2) the controversy is between
persons whose interests are adverse; (3) the party seeking the relief Atty. Rambyung cannot appear as counsel for Chu. Under Sec. 446 LGC,
has a legal interest in the controversy; and (4) the issue is ripe for the Sangguniang Bayan shall be presided over by themunicipal Vice
judicial determination. Suffice it to state that, in the petition filed with Mayor. THUS, Sec. 90(b)(1) applies – as member of the sanggunian he
the trial court, SJS failed to allege the ultimate facts which satisfy these cannot appear as counsel of a party adverse to the NPC, which is an
requisites. Not only that, as admitted by the SJS, the provision the instrumentality of government.
interpretation of which is being sought has already been breached by
the respondents. Declaratory relief cannot thus be availed of.

Class Notes: So who are the real parties in interest? The constituents!

CATU v. RELLOSA
February 19, 2008

Summary: Regina Catu and Antonio Catu filed a complaint before the
Lupong Tagapamayapa of Barangay 723, Manila against Elizabeth Diaz-
Catu and Antonio Pastor. Allegedly, Elizabeth Catu and Antonio Pastor
refused to vacate one of the units in the building co-owned by the
complainant. The parties failed to arrive at an amicable settlement and
Respondent Punong Barangay Vicente Rellosa, Punong issued a
certification for the filing of the appropriate action in court. Regina Catu
and Antonio Catu filed a complaint for ejectment before the MTC of
Manila. Punong Barangay Rellosa entered his appearance as counsel for
the defendants, which resulted in the filing of the present
administrative case against Rellosa alleging impropriety on his acts as a
lawyer and as a public officer when he stood as counsel for the
defendants despite having presided over the conciliation proceedings
between the litigants as punong barangay. The IBP recommended a
suspension of 1 month for having violated Canon 1 and Rule 6.03 of the
Code of Professional Responsibility in relation to Sec. 7(b)(2) of RA
6713. The SC modified the ruling of the IBP and held that it is Sec. 90 of
RA 7160, not Sec. 7(b)(2) of RA 6713 that applies. The SC imposed a 6-
month suspension on Rellosa.
LOCAL BOARDS, DEVELOPMENT COUNCILS
& SPECIAL ECONOMIC ZONES
Note: This part of the syllabus focuses more on cases. Just browse through the provisions.
The following discussion and notes are some key points in Pimentel’s commentary.

COA – PROVINCE OF CEBU v. PROVINCE OF CEBU


LOCAL SCHOOL BOARDS November 2, 2001
Sec. 98-101 LGC Summary: COA conducted an audit in the Province of Cebu and found
that the compensation and personal benefits of public school teachers
School Boards
under the extension classes were charged under the Special
- School boards have been created for every province, city, and
Educational Fund (SEF). It was also found that college scholarship
municipality
grants were charged on the same funds. COA suspended the province.
- They DO NOT have personality separate and distinct from
The province of Cebu filed a declaratory relief with the TC. TC ruled that
the LGU
the expenses were authorized. SC modified TC ruling declaring that
while the compensation of public teachers under the extension classes
Co-Chair and Membership (in Provinces5)
were allowed to be charged under the SEF, the college scholarship
1. Co-Chairs:
grants were not and should be charged in the general funds of the
a. Governor and
province.
b. Division Superintendent of Schools of the Province
2. Members
Doctrine: The salaries and personnel-related benefits of the teachers
a. Chair of the education committee of the Sangguniang
appointed by the provincial school board of Cebu in connection with the
Panlalawigan
establishment and maintenance of extension classes are declared
b. Provincial treasurer
chargeable against the Special Education Fund of the province.
c. Representative of the Pederasyan ng mga SK in the
However, the expenses incurred by the provincial government for the
Sangguniang Panlalawigan
college scholarship grants should not be charged against the Special
d. President of provincial federation of parents-teachers
Education Fund, but against the General Funds of the province of Cebu.
associations
e. Representative of the teachers’ organizations in the
province
OSEA v. MALAYA
f. Representative of the non-academic personnel of public
January 30, 2002
schools in the province

Functions Summary: Dr. Malaya was appointed by President Ramos as Schools


1. Determine the annual supplementary budget needs for the Division Superintendent without any specific division. Subsequently,
operation and maintenance of public schools within the LGU DECS Secretary Gloria designated Dr. Malaya as Schools Division
and the supplementary local cost Superintendent of Camarines Sur. Dr. Osea opposed this, arguing that
Dr. Malaya’s appointment was without consultation with the local
 In accordance with DepEd criteria
2. Authorize treasurer to disburse funds from the Special school boards, in violation of Sec. 99 (LGC 1991). CSC, CA, SC: Sec. 99
Education Fund pursuant to the budget prepared (LGC 1991) applies only to appointments made by the DECS. It does not
apply to appointments by the President, or to reassignments made by
3. Serve as advisory committee to the sanggunian concerned on
the DECS.
educational matters
4. Recommend changes in the names of public schools within
Doctrine: Appointment is the selection, by the authority vested with the
territorial jurisdiction
power, of an individual who is to exercise the functions of a given office.
Meetings, Quorum, Budget When completed, usually with its confirmation, the appointment results
- The board shall meet at least once a month or as often as in security of tenure for the person chosen unless he is replaceable at
necessary pleasure because of the nature of his office. On the other hand, a
- Any of the co-chair may call a meeting reassignment is merely a movement of an employee from one
- Majority of all members shall constitute quorum organizational unit to another in the same department or agency which
does not involve a reduction in rank, status or salary and does not
- The division superintendent, city superintendent, or district
require the issuance of an appointment. A designation connotes merely
supervisor, as the case may be, shall prepare the budget of
the imposition of additional duties on an incumbent official.
the school board
Section 99 (LGC 1991) applies only to appointments made by the DECS.
Compensation
- Co-chairmen and members shall perform their duties It does not apply to appointments by the President, or to reassignments
without compensation or remuneration made by the DECS.
- Members who are not government officials or employees
shall be entitled to necessary travelling expenses and
allowances
 Chargeable against funds of the board

5
Counterparts of the above named persons make up the membership of
the city and municipality school boards
LOCAL HEALTH BOARDS LOCAL DEVELOPMENT COUNCILS
Sec. 102-105 LGC Sec. 106-115 LGC
Local Health Boards Local Development Councils for ALL LGUs 6
- LGC provides for the creation of a local health board for - LGC establishes a local development council (LDC) for every
every province, city and municipality province, city, municipality, and barangay
- PRIMARY DUTY: Initiate comprehensive multisectoral
Membership development plan for the LGU concerned
1. Governor or mayor as Chair
2. Provincial, city, or municipal health officer as Vice Chair Composition of the LDC
3. Other members:
a. Chair of Committee on Health in the sanggunian Provincial LDC City/Municipal LDC Barangay LDC
b. Representative from private sector or NGO involved in Local Chief Executive as Chairman
health services All mayors of
c. Representative of the DOH in the LGU concerned All punong barangays
component cities
in the
and municipalities Members of
Functions city/municipality
concerned Sangguniang
1. Proposing annual budget for the LGU for the operation and Chair of committee on appropriations of the Barangay
maintenance of its health facilities and services in sanggunian
accordance with the standards and criteria of the DOH Congressman or his representatives
2. Advising on health matters as a committee to the sanggunian Representatives of Representatives of
3. Advising local health agencies on health personnel selection Representatives of
NGOs in the NGOs in the
and promotion and other matters related to basic health NGOs in the barangay
province who shall city/municipality
delivery services who shall constitute
constitute not less who shall constitute
not less than ¼ of
than ¼ of members not less than ¼ of
Meetings and Quorum members of the fully
of the fully members of the fully
- Board shall meet at least once a month or as often as organized council
organized council organized council
necessary
- Majority of members of the board shall constitute a quorum Functions
 NOTE: Chair or Vice-Chair MUST be present during - For Provincial, City and Municipal LDC
meetings where budgetary proposals are being 1. Formulate:
proposed or considered a. Long term, medium and annual socioeconomic
- Affirmative vote of all the majority of members shall be plans and policies
necessary to approve such proposals b. Annual public investment programs
c. Local incentives
Compensation and Remuneration c. Prioritize socioeconomic development programs a
- Members shall perform their duties without compensation or d. Coordinate, monitor, evaluate implementation of
remuneration development programs and projects
- Members who are not government officials or employees e. Perform such other functions as may be provided by
shall be entitled to necessary travelling expenses and law or by competent authority
allowances - As for the Barangay LDC
 Chargeable against funds of the board  Basically the same as the other LDCs
 But it has one SPECIFIC power specially granted to it: to
Limited Power of DOH Secretary over LGUs mobilize people’s participation in local development
- With the devolution of power to deliver basic health services, efforts
the DOH Sec. no longer has authority to supervise and
control health operations of LGUs Meetings and Quorum
- HOWEVER, in the following instances, the DOH Sec. may - LDC shall meet at least once every 6 months or as often as
temporarily assume direct supervision and control, upon necessary
direction of President: - The Executive Committee in Sec. 111 represents and acts in
1. Epidemics behalf of the LDC when the latter is not meeting
2. Pestilence  The Executive Committee shall also serve as the Local
3. Other widespread health dangers Countrywide Industrialization Board by virtue of Sec. 6,
RA 7368 (see Special Law compilation)
NOTE: But in no case shall the control be for a period of
more than 6 months (but this is extendible if the LGU Relation to Sanggunian and Regional Development Council
concerned concurs) - Plans, programs, and projects prepared by the LDCs do not
automatically acquire force of law
 They must be submitted to the sanggunian concerned
which enacts the corresponding ordinance
- If adopted by the sanggunian, the said plans, programs, and
projects may be integrated with the development plans of
the next higher LDC
- Note also that approved development plans of a (1) province,
(2) HUC, or (3) ICC shall be submitted to the Regional
Development Council (RDC) which integrates them into the
regional development plans submitted to NEDA

6
Note that the barangays do not have Local School and Health Boards but
ALL LGUs have a Local Development Council
LOCAL PEACE AND ORDER COUNCILS AUTONOMOUS SPECIAL
Sec. 116 LGC ECONOMIC ZONES
The Local Peace and Order Councils (LPOC) Sec. 117 LGC and Special Laws
- LGC establishes in every province, city, and municipality a
local peace and order council Sec. 117 LGC – Establishment by law of autonomous special economic
- Composition and functions are prescribed by EO 309 zones in selected areas of the country shall be subject to
CONCURRENCE by the LGUs included therein
Composition of LPOC (following EO 309)
Special Laws cited in the Syllabus
PROVINCIAL LPOC CITY/MUNICIPAL LPOC 1. RA 7916, The Special Economic Zone Act of 1995
Governor as Chair Mayor as Chair a. Sec. 44 – Relationship with the LGUs
Members: Members: (1) The LGUs comprising the EcoZone shall retain
1. Representative of the 1. Sangguniang their basic autonomy and identity
Sanggunian, chosen Panlungsod or Bayan (2) Cities shall be governed by their respective
by its members representative, chosen charters
2. DSWD Officer by its members (3) Municipalities shall operate and function in
3. Information Officer 2. DSWD Officer accordance with LGC
4. Health Officer 3. Information Officer
5. Representatives of 4. Health Officer 2. RA 7903, The Zamboanga City Special Economic Zone Act of
the central 5. Representatives of the 1995 – See Special Laws compilation
government office or central government
agency in the office or agency in the
province who are city/municipality who
appointed by their are appointed by their
respective heads respective heads

Functions
1. Formulate plans and recommend such measures to improve
or enhance the peace and order and public safety in their
respective areas
2. Monitor the implementation of peace and order programs
and projects in the provincial, city, or municipal levels, and
the operation of Civilian Volunteer Self-Defense
Organizations and such other counter-insurgency programs
and activities
3. Make periodic assessments of the prevailing peace and order
situation and submit a report with recommendations to the
chair of the national peace and order council
4. Perform all other functions assigned by law to the peace and
order council

Regional Peace and Order Council


- In between the LPOC and the national council, there is
another council called the Regional Peace and Order Council
- Performs essentially the same functions as the LPOC
OTHER PROVISIONS APPLICABLE TO
LOCAL GOVERNMENT UNITS
Implementing Rules and Regulation (Art. 15 – 19)
SETTLEMENT OF BOUNDARY DISPUTES 1. Art. 15 – Declaration of Policy; Boundary disputes between
or among LGUs shall be settled amicably
Sec. 118-119 LGC 2. Art. 16 – Jurisdictional responsibility (same with Sec. 118,
LGC)
Jurisdictional Responsibility for Settlement of Boundary Disputes 3. Art. 17 – Procedures
- Boundary disputes between and among LGUs shall, AS MUCH a. File a petition initiated by the sanggunian concerned
AS POSSIBLE, be settled amicably, following: with the sanggunian having jurisdiction over the
dispute
BOUNDARY DISPUTE (1) State the grounds and reasons therefor; attach
REFER TO
BETWEEN OR AMONG pertinent documents like technical descriptions,
Barangay/s v. Barangay/s Sangguniang maps, statute creating the LGU, etc.
Panlungsod or b. Adverse party answers within 15 working days
Of the SAME city or municipality Bayan concerned c. Hearing within 5 days after receipt of answer
Municipality/s v. Municipality/s Sangguniang (1) When 2 or more sanggunians jointly hear a case,
Panlalawigan they may sit en banc or designate their
Of the SAME province concerned representatives
Municipality or Municipality or JOINT referral d. If there is failure to amicable settlement, sanggunian
v.
Component City Component City To the sanggunians shall issue a certification to this effect
of the provinces e. Within 60 days from date of certification, the dispute
Of DIFFERENT provinces concerned shall be formally tried by the sanggunian concerned
JOINT referral to the (1) Copies of decision furnished to DILG, Local
Component City/ Highly Urbanized respective Assessor, COMELEC, NSO, other NGAs concerned
v.
Municipality City sanggunians of the within 15 days from promulgation
parties f. Party may appeal to the proper RTC following ROC
JOINT referral to the (1) RTC to decide within 1 year from filing
Highly Highly Urbanized respective 4. Art. 18 – Pending final resolution of the dispute, status quo is
v.
Urbanized City/s City/s sanggunians of the maintained
parties 5. Art. 19 – DILG shall be the official custodian of copies of all
documents on boundary disputes of LGUs
NOTE: The table above reflects the cases enumerated in Sec. 118 LGC
- For boundary disputes between 2 or more LGUs that do not
fall under any of the categories in Sec. 118, jurisprudence tell PASIG v. COMELEC
us that prior referral to Sanggunian is not required September 10, 1999

PROCEDURE (Sec. 118(c) and Sec. 119) Summary: The City Council of Pasig passed two ordinances creating two
barangays. Two separate plebiscites for the creation of the two
barangays were scheduled on two different dates. The Municipality of
Cainta moved to suspend or cancel the respective plebiscites scheduled
by filing a petition with the COMELEC. The Municipality of Cainta called
the attention o the COMELEC to a pending case before the RTC of
Antipolo for settlement of boundary dispute. According to the
Municipality of Cainta, the proposed barangays encroached upon areas
within its own jurisdiction/territory while the City of Pasig claims these
areas as part of its jurisdiction/territory. The COMELEC ruled in favor
of Cainta and suspended one of the plebiscites but dismissed the
petition with respect to the other barangay for being moot because the
plebiscite was already held. The Supreme Court ruled in favor of the
Municipality of Cainta and upheld the validity of the suspension
ordered by the COMELEC.

Doctrine: The civil case involving the boundary dispute is a prejudicial


question which must first be decided before plebiscites for the creation
of the proposed barangays may be held.

Pasig argues both cases are civil so prejudicial question does not apply.
But SC held that it can very well suspend action in one case pending the
final outcome of another case closely interrelated or linked to the first.
It is also fare more prudent to hold in abeyance the conduct of
plebiscite pending final determination of the civil case considering the
expenses involved.
PROVINCE OF CAMARINES NORTE v. PROVINCE OF QUEZON
October 11, 2001 LOCAL INITIATIVE AND REFERENDUM
Summary: There was a boundary dispute between Camarines Norte and Sec. 120-127 and 398 LGC
Quezon. SC issued a resolution resolving with finality the dispute in
favor of Camarines Norte. A marker was placed to designate the new
IN GENERAL
boundary. Quezon removed the marker. Camarines Norte went to court
asking that the Quezon officials be held in contempt. SC sided with
Local Initiative – Local initiative is the legal process whereby the
Camarines Norte.
registered voters of a local government unit may directly propose,
enact, or amend any ordinance
Doctrine: Everyone is bound by the said final decision of this Court and
- The initiative discussed here is the DIRECT initiative where
that the boundary dispute there resolved is no longer a dispute and that
the voters start the legislative process for the enactment,
all the attendant legal issues have been resolved with finality. That
amendment or repeal of an ordinance or resolution in the
decision of this Court constitutes res adjudicata in respect of all offices
absence of positive action of the sanggunian
and agencies of the Executive Department.
Subject Matter – All sorts of measures may be the subject of direct
MUNICIPALITY OF KANANA v. MADRONA initiative for as long as these are within the competence of the
April 30, 2003 sanggunian to enact
- Garcia v. Comelec – Resolutions are proper subjects of
Summary: There was a boundary dispute between the Municipality of initiative even if Sec. 120 mentions of ordinance only
Kananga and the City of Ormoc. Their repective Sanggunians tried to
reach an amicable settlement but failed. The City of Ormoc filed a civil RA 6735 – An Act Providing for a System of Initiative and Referendum
case before the RTC of Ormoc City to settle the dispute. The - Enacted before the LGC
Municipality of Kananga filed a Motion to Dismiss on the ground of lack - Include a system of initiative on local legislation that would
of jurisdiction over the subject matter because Sec. 118 of the LGC allow petitions proposing the enactment of a regional,
governs as the boundary dispute is between Kananga, a municipality, provincial, city, municipal, or barangay law
and Ormoc, an independent chartered city. Based on the LGC provision, - It recognizes the right of “any duly accredited people’s
the boundary must be settled amicably. The RTC denied the MTD and organization to file a petition for INDIRECT INITIATIVE with
ruled that it had jurisdiction under BP 129; Sec. 118 of the LGC has been the [appropriate] legislative bodies”
substantially complied with; and that the LGC provision is on venue  It is INDIRECT because the people course their action
which the parties could waive. The SC held that Sec. 118 of the LGC is through their elected representatives in the national
not applicable in this case because it is a dispute between a legislature or local councils
municipality and an independent component city –a situation not  NOTE! LGC speaks of LOCAL INITIATIVE which,
covered by the said LGC provision. The RTC thus had jurisdiction based according to Pimentel, refers to DIRECT initiative;
on Sec. 19 of BP 129 that gives it the exclusive and original jurisdiction Furthermore, the power of local initiative and
over all cases not within the exclusive jurisdiction of any court, tribunal, referendum is vested in ALL registered voters
person or body.
Local Referendum - Legal process whereby the registered voters of
Doctrine: Settlement of boundary disputes not covered by Sec. 118 of the local government unit may approve, amend, or reject any ordinance
the LGC is governed by general rules governing jurisdiction found in BP enacted by the Sanggunian.
129.
The following tables are taken from the discussion in SBMA v. COMELEC
Since there is no law providing for exclusive jurisdiction of any court or
Distinction as found in RA 6735
agency over the settlement of boundary disputes between a
INITIATIVE REFERENDUM
municipality and an independent component city of the same province,
Power of the people to propose
the RTC has original and exclusive jurisdiction. Power of the electorate to
amendments to the constitution
approve or reject a legislation
or to propose and enact
MUNICIPALITY OF PATEROS v. CA through an election called for
legislations through an election
June 16, 2009 the purpose
called for that purpose
Systems of initiative:
Summary: The municipality of Pateros filed a petition before the Makati
1. Initiative on the Classes of referendum:
RTC for the judicial declaration of its territorial boundaries claiming
Constitution which 1. Referendum on
that Fort Bonifacio is within its territory and not Makati’s. When it filed
refers to a petition statutes which refers
the petition, Makati was still a municipality but the bill converting it to a
proposing to a petition to
city was already pending before the Senate. The RTC dismissed the case
amendments to it approve or reject an
for lack of jurisdiction to determine boundary disputes. The SC ruled
2. Initiative on statutes act or law, or part
that following the LGC, and now that Makati is a city, the RTC only has
which refers to a thereof, passed by
an appellate jurisdiction over the matter.
petition proposing to Congresss
enact a national 2. Referendum on local
Doctrine: Now that Makati is already a highly urbanized city, the parties
legislation law which refers to a
should follow Section 118(d) of the LGC and should opt to amicably
3. Initiative on local petition to approve or
settle this dispute by joint referral to the respective sanggunians of the
legislation which refers reject a law,
parties. In the event that no amicable settlement is reached, as
to a petition proposing resolution, or
envisioned under Section 118(e) of the LGC, a certification shall be
to enact a regional, ordinance, enacted by
issued to that effect, and the dispute shall be formally tried by the
provincial, city, regional assemblies
Sanggunian concerned within sixty (60) days from the date of the
municipal, or barangay and local legislative
aforementioned certification. In this regard, Rule III of the Rules and
law, resolution, or bodies
Regulations Implementing the LGC shall govern. Only upon failure of
ordinance
these intermediary steps will resort to the RTC follow, as specifically
Indirect initiative – exercise of
provided in Section 119 of the LGC.
initiative by the people through a
proposition sent to Congress or
local legislative body for action
Distinction made by the Court in SBMA VOTE REQUIRED
PERIOD STATUS8
INITIATIVE REFERENDUM (For the sanggunian to repeal,
(Counted from approval in
Resorted to by the people directly amend, or modify the approved
initiative)
either because: proposition)
The law-making body submits to
1. The law-making body
the registered voters of its From day 1 to 6 months from Absolute Protection
fails or refuses to enact
territorial jurisdiction, for adoption Sanggunian can’t change anything
the law, ordinance,
approval or rejection, any
resolution, or act that From the end of the 1st 6
ordinance or resolution which is
they desire; or months PLUS 3 years
duly enacted or approved by
2. They want to amend or In case of municipal, city, or Qualified Protection
such law-making body
modify one already provincial initiative To change, ¾ of all the members of
existing From the end of the 1st 6 the sanggunian is needed
Sec. 13, RA 6735, local legislative months PLUS 12 months
body is given the opportunity to In case of barangay initiative
enact the proposal After 3 years and 6 months
- If it refuses/neglects to from adoption of a municipal,
do so within 30 days No More Protection
city, or provincial initiative
from presentation, The usual simple majority is
After 1 year and 6 months from
proponents through needed
adoption of a barangay
their duly-authorized initiative
and registered
representatives may This referendum shall be
invoke their power of conducted also under the PROCEDURE ON LOCAL INITIATIVES (Sec. 124)
initiative with notice to control and direction of the
the legislative body Comelec
concerned
- If proponents manage
to collect the number
of signed conformities
within period granted
by statute, Comelec
shall set a date for the
initiative (not
referendum)
Entirely the work of the Begun and consented to by the
electorate law-making body
This is a process of law-making by
This is merely the electorate
the people themselves without
approving or rejecting what has
participation and against the
been drawn up or enacted by a
wishes of their elected
legislative body
representatives
Here, the voters will simply
More complex than referendum
write either yes or no

Who May Exercise Initiative and Referendum? ALL REGISTERED


VOTERS of the provinces, cities, municipalities, and barangays

LIMITATIONS ON LOCAL INITIATIVES7

LIMITATIONS on Local Initiatives (Sec. 124)


1. Local initiative shall not be exercised more than once a year
2. Initiative shall extend only to subjects or matters which are
within the legal powers of the sanggunians to enact
3. If at any time before the initiative is held, sanggunian
ADOPTS IN TOTO the proposition presented and the local
chief executive approves the same, initiative shall be
CANCELLED
 However, those AGAINST such action may, if they so
desire, apply for initiative STEP 1: THE PETITION
- In proposing the enactment, repeal, or amendment of an
NOTE: Although initiative can only be exercised once a year, the ordinance (or resolution), the LGC requires a certain number
petition may contain more than one subject matter. of petitioners depending on LGU (same with RA 6735)
1. At least 1,000 registered voters for provinces and cities
LIMITATIONS on the Sanggunian (Sec. 125) 2. At least 100 registered voters in case of municipalities
- Once a resolution, ordinance, or proposition is approved by 3. At least 50 registered voters in case of barangays
the exercise of the LGU of local initiative, there are certain
limitations placed upon the Sanggunian
- See table on the next page

8
This table is based on Pimentel’s discussion. He comments that the
7
NOTE: The limitations in Sec. 124 and 125 apply to LOCAL INITIATIVES, phrasing of Sec. 125 is incorrect and vague. He believes that his
not local referendum interpretation is correct. No SC ruling yet.
STEP 2 and 3: INVOKE INITIATIVE and NOTICE TO SANGGUNIAN GARCIA v. COMELEC
- If no favorable action from the sanggunian within 30 days September 30, 1994
- NOTE: Failure of the Sanggunian to act on the petition
favorably is a CONDITION PRECEDENT before initiative can Summary: The Sangguniang Bayan of Morong issued a resolution
be invoked agreeing to the inclusion of Morong to the SSEZ. Petitioners filed a
petition to the Sanggunian. For failure of the same to act on it, the
STEP 4: THE PROPOSITION petitioners resorted to local initiative. The COMELEC denied the
- Proposition shall be numbered serially starting with (I) petition for local initiative, in view of Vice Mayor De Leon’s letter
- COMELEC assists in the formulation of the petition petition asking the said agency to deny the same. The SC, in setting
aside the resolution of the COMELEC, declared that resolutions can be
STEP 5: COLLECTION OF SIGNATURES the proper subject of local initiatives.
- Counted from the filing of notice to the sanggunian, the
proponents have a fixed number of period to collect the Doctrine: Through initiative, the people were given the power to amend
necessary signatures: the Constitution under Sec. 2 Art. 17 which provides “amendments to
1. 90 days for provinces and cities this Constitution may likewise be directly proposed by the people
2. 60 days for municipalities through initiative upon a petition of at least 12% of the total number of
3. 30 days for barangays registered voters, of which every legislative district must be
- Manner of signing represented by at least 3% of the registered voter therein.”
 Signed before the election registrar, or designated
representatives Sec. 32(a) of RA No. 6735 provides for the 3 systems of initiative,
 In the presence of a representative of the proponent namely:
and a representative of the sanggunian 1. Initiative on the Constitution – petition to amend the
 Must be in a public place in the LGU Constitution
 Stations for collecting signatures may be 2. Initiative on statutes – petition proposing to enact a national
established in as many places as may be legislation
warranted 3. Initiative on local legislation – petition proposing to enact a
regional, provincial, city, municipal, or barangay law,
STEP 6: VERIFICATION BY COMELEC resolution or ordinance.
- COMELEC verifies if the required number of signatures are
obtained
 If not, then dismiss the proposition SBMA v. COMELEC
 If yes, then it issues a certification attesting to this fact September 26, 1996

STEP 7 and 8: INITIATIVE Summary: RA 7227 or the Bases Conversion and Development Act of
- If required signatures are obtained, COMELEC sets the date 1992 created the Subic Bay Metropolitan Authority and the Subic
for the initiative Special Economic Zone (SSEZ). The Sangguniang Bayan of Morong
 In the initiative, the proposition is submitted to the passed a resolution expressing concurrence to join the SSEZ.
registered voters for approval Respondent filed a petition with the Sanggunian to annul the resolution.
- Periods (computed from date of COMELEC certification in So the Sanggunian requested Congress to amend certain provisions of
STEP 6) RA 7227. Not satisfied, respondents resorted to their power of
1. Within 60 days in case of provinces initiative. COMELEC promulgated guidelines for the conduct of a
2. Within 45 days in case of municipalities Referendum. The Court said that the proper process was that of an
3. Within 30 days in case of barangays Initiative.

Effectivity of Local Propositions (Sec. 123) Doctrine: "Initiative" is the power of the people to propose amendments
- If proposition is approved by a MAJORITY of the votes cast, it to the Constitution or to propose and enact legislations through an
shall take effect 15 days AFTER certification by the COMELEC election called for the purpose.
as if affirmative action thereof had been made by the
Sanggunian and Local Chief Executive There are three (3) systems of initiative, namely:
- If it fails to obtain said number of votes, proposition defeated 1. Initiative on the Constitution which refers to a petition
proposing amendments to the Constitution;
2. Initiative on statutes which refers to a petition proposing to
PROCEDURE IN LOCAL REFERENDUM (Sec. 126) enact a national legislation; and
3. Initiative on local legislation which refers to a petition
STEP 1: INITIATION BY SANGGUNIAN proposing to enact a regional, provincial, barangay law,
- Ordinances passed by the Sanggunian may be submitted by it resolution or ordinance.
for the approval, amendment, or rejection by the registered
voters "Indirect initiative" is exercise of initiative by the people through a
- NOTE that in Referendum the process begins and ends with proposition sent to Congress or the local legislative body for action.
the sanggunian
"Referendum" is the power of the electorate to approve or reject a
STEP 2: CONDUCT OF REFERENDUM legislation through an election called for the purpose. It may be of two
- COMELEC supervises and directs the conduct of the classes, namely:
referendum: 1. Referendum on statutes which refers to a petition to approve
1. Within 60 days for provinces and cities or reject an act or law, or part thereof, passed by Congress;
2. Within 45 days for municipalities and
3. Within 30 days in case of barangays 2. Referendum on local law which refers to a petition to
approve or reject a law, resolution or ordinance enacted by
STEP 3: COMELEC CERTIFICATION regional assemblies and local legislative bodies.
- COMELEC certifies and proclaims the results of the
referendum Initiative is the "power of the people to propose bills and laws, and to
enact or reject them at the polls independent of the legislative
assembly." On the other hand, referendum is defined as "the right
reserved to the people to adopt or reject any act or measure which has
been passed by a legislative body and which in most cases would
without action on the part of electors become a law."

Initiative is resorted to by the people directly either because the law-


making body fails or refuses to enact the law, ordinance, resolution or
act that they desire or because they want to amend or modify one
already existing. If the local legislative authority fails to enact the
proposal, the proponents may invoke their power of initiative. If the
required signatures are collected then the COMELEC shall set a date for
the initiative.

In contrast, in a local referendum, the law-making body submits to the


registered voters of its territorial jurisdiction, for approval or rejection,
any ordinance or resolution which is duly enacted or approved by such
law-making authority. Said referendum shall be conducted also under
the control and direction of the Commission on Elections.

In contrast, in a local referendum, the law-making body submits to the


registered voters of its territorial jurisdiction, for approval or rejection,
any ordinance or resolution which is duly enacted or approved by such
law-making authority. Said referendum shall be conducted also under
the control and direction of the Commission on Elections.

While initiative is entirely the work of the electorate, referendum is


begun and consented to by the law-making body. Initiative is a process
of law-making by the people themselves without the participation and
against the wishes of their elected representatives, while referendum
consists merely of the electorate approving or rejecting what has been
drawn up or enacted by a legislative body. The process and the voting
in an initiative are understandably more complex.

In initiative and referendum, the COMELEC exercises administration


and supervision of the process itself, akin to its powers over the
conduct of elections. These law-making powers belong to the people,
hence the respondent Commission cannot control or change the
substance or the content of legislation. In the exercise of its authority, it
may issue relevant and adequate guidelines and rules for the orderly
exercise of these "people-power" features of our Constitution.
LOCAL TAXATION AND FISCAL MATTERS
Constitutional Grant - Each LGU shall have the power to create new
LOCAL GOVERNMENT TAXATION sources of funds and to levy taxes, fees, and charges subject to
limitations as Congress may provide, consistent with the basic policy of
GENERAL CONCEPTS local autonomy. Such taxes, fees, and charges accrue exclusively to the
local governments. (Sec 5, Article X, 1987 Constitution)
Special Laws cited in the Syllabus
1. See Art. X, 1987 Constitution, Sec. 3(d), 5, 6, and 7 Sources of LGU Funds
(1) Own sources of revenue
2. RA 7643 – An act to empower the CIR to require payment of (2) Taxes, fees, and charges which accrue exclusively for their
VAT every month and to allow LGUs to share in VAT revenue use and disposition
a. Sec. 2 – In addition to the IRA, 50% of the national taxes (3) Just share in national taxes which shall be automatically and
collected under Sec. 100, 102, 112, 113, 114 NIC shall directly released to themselves
be distributed as follows: (4) Equitable share in the proceeds from utilization and
(1) 20% - city or municipality where such taxes are development of national wealth and resources within their
collected (follow Sec. 150 LGC in allocating) territorial jurisdiction
(2) 80% - to the national government
Fundamental Principles on Taxation by an LGU (Sec. 130)
3. RA 7716 – Restructuring VAT system (1) Taxation shall be uniform;
a. Sec. 11 amended Sec. 115 NIRC on percentage tax on (2) Taxes, fees, and charges shall be equitable and based as far as
carriers and keepers of garages practicable on the taxpayer's ability to pay;
(1) Gross receipts of common carriers derived from (3) Levied and collected only for a public purpose;
their incoming and outgoing freight shall not be (4) Shall not be unjust, excessive, oppressive, or confiscatory;
subjected to local taxes in LGC (5) The collection of taxes, fees, and charges shall in no case be
b. Sec. 15 – Imposed VAT on services performed in the left to any private person;
exercise of profession or calling subject to the (6) The revenue shall insure solely to the LGU, unless otherwise
professional tax under the LGC, among others specified;
(7) Each LGU shall, as far as practicable, evolve a progressive
4. RA 8241 – Act amending the E-VAT and NIRC (see special system of taxation;
law compilation) (8) Shall not be contrary to law, public policy, national economic
5. NIRC, Sec. 117, 283, 287 – Percentage taxes (see special law policy, or in restraint of trade;
compilation)
LGUs, in addition to administrative autonomy, also enjoy fiscal
6. RA 8245 – Act appropriating P15.455M for the increased autonomy. LGUs have the power to create their own sources of revenue,
share in 1997 of LGUs in the national internal revenue taxes in addition to their equitable share in the national taxes and their
pursuant to Sec. 284 LGC power allocate resources in accordance with their own priorities.
(Pimentel v. Aguirre)
7. RA 7942 – Act instituting a new system of mineral resources
exploration, development, utilization, and conservation Taxing Power of LGUs
a. Sec. 82 – Allocation of government share; Follow Sec. - Municipal corporations, unlike the sovereign state, is clothed
290 and 292 LGC with no inherent power of taxation
 Power must be granted
8. RA 7916, Sec. 24, as amended by RA 8748  Once granted, it is construed strictissimi juris against
a. No taxes, local and national, shall be imposed on the LGU
business establishments operating within the EcoZone - Taxing power of LGUs subject to congressional limitations
(1) EXCEPT for real property taxes on land owned by (Estanislao v. Costales)
developers - Taxing power is limited to its territorial jurisdiction
b. In lieu thereof, 5% of gross income earned by all
business enterprises within the EcoZone shall be paid Local Taxing Authority (Sec. 132)
and remitted as follows: - Power to impose a tax, fee, or charge or to generate revenue
(1) 3% to National Government under the LGC shall be exercised by the sanggunian
(2) 2% directly remitted by business establishments - Exercised through appropriate ordinance
to treasurer’s office of the municipality or city  But there must be prior public hearings for the
where enterprise is located ordinance to be valid
- Procedure:
9. RA 8975 – Act prohibiting lower courts from issuing TROs,  A public hearing must be conducted prior to the
preliminary injunctions to ensure expeditious enactment of a tax ordinance. (Sec. 187, LGC)
implementation and completion of government  Within ten (10) days after the approval of the
infrastructure projects ordinance, certified true copies of all tax ordinances or
a. Sec 7 – Issuance of Permits; Upon payment in cash of revenue measures shall be published in full for three
necessary fees levied under LGC, governor of province (3) consecutive days in a newspaper of local circulation.
or mayor of HUC shall immediately issue the necessary In provinces, cities and municipalities where there are
permit to extract sand, gravel, and other quarry no newspapers of local circulation, it must be posted in
resources needed in government projects at least two (2) conspicuous and publicly accessible
places. (Sec. 188, LGC)
10. RA 9165 – Comprehensive Dangerous Drugs Act
a. Sec. 51 – LGUs shall appropriate substantial portion of
their respective annual budgets to assist in or enhance
implementation of the law
Scope of Taxing Power of LGUs9 In Summary:
A. PROVINCES (Sec. 134)
 Province may only levy the taxes, fees, charges provided LGU Scope of Taxing Power
in Sec. 134 – 141, except as otherwise provided in LGC May levy only:
 These are: (1) Transfer of Real Property Ownership
1. Transfers of real property ownership (2) Business of Printing and Publication
2. Printing and publication business (3) Franchise Tax
3. Franchises Provinces (Sec. (4) Tax on Sand, Gravel and Other Quarry
4. Sand, gravel, and other quarry resources 134, LGC) Resources
5. Professionals (5) Professional Tax
6. Amusement (6) Amusement Tax
7. Delivery trucks or vans of manufacturers or (7) Annual Fixed Tax for every delivery
producers, wholesalers, dealers, or retailers or truck or van
liquor, soft drinks, cigars, and cigarettes and other May levy taxes, fees and charges not otherwise
products as may be determined by the Sanggunian Municipalities
levied by provinces (Sec. 142, LGC)
B. MUNICIPALITIES (Sec. 142) May levy taxes, fees and charges which the
 May levy taxes, fees, charges not otherwise levied by Cities province or municipality may impose (Sec. 151,
provinces, except as otherwise provided in LGC LGC)
 Eight general categories of people fall within the
municipality’s power to tax:
May levy only:
1. Manufacturers, assemblers, or repackers of any
(1) Taxes on stores or retailers
article of commerce or processors, brewers,
Barangays (2) Service fees or charges
rectifiers and compounders of liquors, distilled
(3) Barangay clearance
spirits and wines of whatever kind or nature
(4) Other fees and charges (Sec. 152, LGC)
2. Wholesalers, distributors or dealers in any article
of commerce of whatever kind or nature
3. Exporters or manufacturers, millers, producers,
distributors, dealers in or retailers or essential
commodities
4. Retailers of any article of commerce BASCO v. PAGCOR
5. Contractors and other independent contractors May 14, 1991
6. People who engage in banking or financing
7. Peddlers of any merchandise or article of Summary: Atty. Basco, Balce, Maranan and Sanchez filed a petition
commerce seeking to annul the PAGCOR Charter PD 1869 for being contrary to
8. People who engage in any other business not morals, public policy and order. Also, that it is 1) a waiver of a right
included in the above enumeration which the prejudicial to Manila City government's right to impose taxes and
sanggunian may deem proper to tax license fees, which is recognized by law; 2) that this, in contravention of
the constitutionally enshrined principle of local autonomy; 3)that it
C. CITIES (Sec. 151) violates the equal protection clause of the constitution in that it
 May impose taxes, fees, charges which the province or legalizes PAGCOR- conducted gambling, while most other forms of
municipality may impose gambling are outlawed, together with prostitution, drug trafficking and
other vices; 4) that it violates the avowed trend of the Cory government
 Rates of taxes that the city may levy may exceed the
away from monopolistic and crony economy, and toward free
maximum rates for the province or municipality by not
enterprise and privatization. The Court ruled in favor of PAGCOR.
more than 50%
 Except the rates of professional and amusement
Doctrine: Petitioners argue that PD 1869 constitutes a waiver of the
taxes
right of Manila City to impose local taxes and fees; that the exemption
clause is violative of local autonomy. Contention is without merit
D. BARANGAYS (Sec. 152)
 Minimal powers:
Manila, being a mere municipal corporation, has no inherent right to
1. Taxes on stores or retailers with fixed businesses
impose taxes. So it must yield to a legislative act. Manila Charter is
establishments under the conditions in Sec. 152(a)
subject to control by Congress. Congress can grant Manila the power to
2. Fees or charges for services rendered in
tax and it can also provide exemptions or even take back that power
connection with the regulation or use of barangay-
owned properties or service facilities
Manila’s power to impose license fees on gambling has long been
3. Fees for barangay clearance for any business or
revoked. PD 771 of 1975 withdrew the power of LGUs to regulate
activity
gambling thru the grant of franchise, license, or permits; this power is
4. Other fees and charges on:
now with the national government
(1) Commercial breeding of fighting cocks,
cockfights, cockpits
LGUs have no power to tax instrumentalities of the national
(2) Places of recreation which charge admission
government. PAGCOR is a GOCC with an original charter; stocks owned
fees
by government. Being a government instrumentality it is exempt from
(3) Billboards, signboards, neon signs, outdoor
local taxes, otherwise its operation might be burdened, impeded or
advertisements
subjected to control by mere LGU

Class Notes: The case of Basco is not controlling anymore. See Batangas
Corporation v. Batangas

9
Not in the syllabus but we think this is important since an LGU can only
impose a tax when the LGC grants it such power
ESTANISLAO v. COSTALES protest” and accordingly appeal to the Board of Assessment Appeals by
May 8, 1991 filing a petition under oath together with copies of the tax declarations
and affidavits or documents to support his appeal.
Summary: Zamboanga City imposed at P0.01 tax on every liter of
softdrink produced. Issue arose as to whether or not the municipal [Publication] In view of §188 and 511(a) of the LGC, an ordinance fixing
government is allowed to impose such a tax. RTC upheld it on the the assessment levels applicable to the different classes of real property
ground that the period to review and overturn it by the Secretary of in a local government unit and imposing penal sanctions for violations
Finance already lapsed. SC rules that under the Local Tax Code such act thereof (such as Ordinance No. 125) should be published in full for
is ultra vires as the tax should be on the gross sale, not on the output three (3) consecutive days in a newspaper of local circulation, where
and that the Secretary cannot be estopped from reviewing the said available, within ten (10) days of its approval, and posted in at least
Ordinance. two (2) prominent places in the provincial capitol, city, municipal, or
barangay hall for a minimum of three (3) consecutive weeks.
Doctrine: A city, like public respondent Zamboanga City may impose, in
lieu of the graduated fixed tax prescribed under Section 19 of the Local
Tax Code, a percentage tax on the gross sales for the preceding calendar CITY OF OLONGAPO v. STALLHOLDERS OF THE EAST BAJAC-
year of non-essential commodities at the rate of not exceeding two per BAJAC PUBLIC MARKET OF OLONGAPO CITY
cent and on the gross sales of essential commodities at the rate of not October 19, 2000
exceeding one per cent.
Summary: The Olongapo City Council enacted Ordinance 14, fixing
Class Notes: You can only tax as you are told to tax. monthly rental fees for different stalls in the new public market. The
stallholders questioned the validity of Ordinance 14 by filing an appeal
to the Secretary of Justice, pursuant to Sec. 187, LGC. While a
FLORO CEMENT v. GOROSPE reconsideration was pending before the Secretary, a case was filed
August 12, 1991 before the SC questioning the validity of Sec. 187, LGC. So, the Chief
State Council informed the Stallholders that in the meantime, the
Summary: The Municipality of Lugait assessed Floro Cement Secretary was refraining from taking any action on the matter, and
Corporation for manufacturer’s and exporter’s taxes imposed by advised them to file their appeal with the “court of competent
Municipal Ordinances 5 and 10. Floro Cement refused to pay claiming jurisdiction.” Hence, the Stallholders filed an action before the RTC to
that Sec 52 of PD 463 withdraw the power of municipality to levy such declare Ordinance 14 void. The RTC sustained the validity of Ordinance
taxes because cement is a mineral and is covered by the tax exemption. 14. The CA held that the question of whether the rental rates were
It also insisted on the tax exemption certificate granted by DENR. CFI unjust, excessive, confiscatory, and inequitable was a factual issue,
ordered Floro Cement to pay such taxes. SC affirmed ruling that requiring the presentation of evidence. Thus, the CA remanded the case
manufacture and the export of cement do not fall under the claimed to the RTC. Before the Supreme Court, Olongapo City claims that the
exception for it is not a mineral product. RTC’s action is merely a review of the decision of the Secretary, and as
such, the review was confined to the evidence already presented in the
Doctrine: It is a rule that as the power of taxation is a high prerogative administrative proceedings. The Supreme Court disagreed, holding that
of sovereignty, the relinquishment is never presumed and any the action before the RTC was an original action, and that trial is
reduction or diminution thereof with respect to its mode or its rate, necessary.
must be strictly construed, and the same must be coached in clear and
unmistakable terms in order that it may be applied. More specifically Doctrine: The Secretary of Justice abdicated his jurisdiction over the
stated, the general rule is that any claim for exemption from the tax appeal. Ergo, the “appeal” to the RTC was actually an original action,
statute should be strictly construed against the taxpayer. He who claims necessarily calling for a presentation of evidence and a trial. The scope
an exemption must be able to point out some provision of law creating of the RTC’s review could not be limited to the evidence presented
the right; it cannot be allowed to exist upon a mere vague implication or before the Secretary.
inference. It must be shown indubitably to exist, for every presumption
is against it, and a well-founded doubt is fatal to the claim. Class Notes: When can a tax law be considered oppressive?
- Wide area of discretion
- But this is a factual issue which needs to be threshed out
FIGUERRES v. CA during trial
March 25, 1999
Summary: Belen C. Figuerres is the owner of of in
Mandaluyong. Pursuant to Ordinance Nos. 119 and 125, series of 1993, BERDIN v. MASCARINAS
and Ordinance No. 135, series of 1994, of the Sangguniang Bayan July 6, 2007
of Mandaluyong, she received a notice of assessment for the land. She
sought relief from the CA to prohibit the enforcement of the tax against Summary: Berdin, President of the Tubigon Market Vendors Association
her, to no avail. She appealed to the SC, which likewise ruled against filed a protest before Municipal Mayor and Municipal Treasurer
her. assailing the validity of Tax ordinance passed by the Sangguniang
Bayan of Tubigon. They also filed a protest before the RTC. Provincial
Doctrine: [Appeals] Section 187 of the LGC provides that the taxpayer Treasurer approved the ordinance. 4 years later RTC upheld the validity
may question the constitutionality or legality of a tax ordinance on of the ordinance. Petitoners appealed stating that Provincial Fiscal
appeal within thirty (30) days from effectivity thereof, to the Secretary already made a contrary ruling earlier. CA affirmed the ruling of the
of Justice. The petitioner after finding that his assessment is unjust, RTC. SC affirmed validity but held that same cannot be enforced unless
confiscatory, or excessive, may bring the case before the Secretary of there is publication under section 43 of the local Tax code.
Justice for questions of legality or constitutionality of the city ordinance.
Under Section 226 of the LGC, an owner of real property who is not Doctrine: A tax ordinance may either be (1) reviewed or suspended by
satisfied with the assessment of his property may, within sixty (60) the Provincial Treasureror the Secretary of Finance, (2) the subject of a
days from notice of assessment, appeal to the Board of Assessment formal protest with the Secretary of Finance, or (3) questioned as to its
Appeals. legality and referred for opinion to the Provincial Fiscal. In this case.
Berdin did not even bring the issues relative to the legality or validity of
Should the taxpayer question the excessiveness of the amount of tax, he Tax Ordinance No. 88-11-36 to the Provincial Fiscal. What they brought
must first pay the amount due, in accordance with Section 252 of the for the consideration of the Provincial Fiscal was Tax Ordinance No. 89-
LGC. Then, he must request the annotation of the phrase “paid under 10-49. The remaining issues were also not brought to the Secretary of
Finance because what was filed was a petition with the Municipal COMMON LIMITATIONS ON THE TAXING POWERS
Mayor requesting for a suspension of the implementation of the
ordinance. Validity must be upheld subject to the compliance with OF LOCAL GOVERNMENT UNITS
publication requirement.
Unless otherwise provided herein (LGC), the exercise of the taxing
powers of provinces, cities, municipalities, and barangays shall not
OLIVARES v. MARQUEZ extend to the levy of the following:
September 22, 2004
(a) Income tax
Summary: Petitioners filed a petition for certiorari, prohibition and  EXCEPT when levied on banks and other financial
mandamus with the RTC questioning the assessment and levy made by institutions
the Office of City Treasurer Paranaque City on their properties:
Respondents are the Mayor, City Treas., Asst. City Treas., City Assessor (b) Documentary stamp tax
and Asst. City Assessor. They filed MTD alleging that RTC has no
jurisdiction over tax assessment matters and that petitioners failed to (c) Taxes on estates, inheritance, gifts, legacies and other
comply with requirements of a tax protest. RTC granted the MTD. Thus, acquisitions mortis causa
petitioners went to SC on petition for review. The SC denied the
petition. (d) Customs duties, registration fees of vessel and wharfage on
wharves, tonnage dues, and all other kinds of customs fees,
Doctrine: The LGC provides for the admin remedies available to a charges and dues
taxpayer or real property owner who is not satisfied with the  EXCEPT wharfage on wharves constructed and
assessment of reasonableness of the real property tax sought to be maintained by the local government unit concerned
collected:
A. Sec. 252 Payment Under Protest (e) Taxes, fees, and charges and other impositions upon goods
1. Taxpayer must first pay the tax carried into or out of, or passing through, the territorial
2. Annotated on the tax receipts: “paid under protest” jurisdictions of local government units in the guise of charges
3. Protest in writing must be filed within 30 days from for wharfage, tolls for bridges or otherwise, or other taxes,
payment to provincial, city, or municipal treasurer fees, or charges in any form whatsoever upon such goods or
a. Decide the protest within 60 days from receipt merchandise
4. If protest decided in favor of taxpayer: Amount or
portion protested shall be refunded or applied as tx (f) Taxes, fees or charges on agricultural and aquatic products
credits when sold by marginal farmers or fishermen
5. If protest is denied or upon lapse of the 60-day period
a. Taxpayer may avail of Chap. 3, Title II, Book II, LGC (g) Taxes on business enterprises certified to by the Board of
Investments as pioneer or non-pioneer for a period of six
B. Chap. 3, Title II, Book II on Assessment Appeals (6) and four (4) years, respectively from the date of
1. Appeal to Local Board of Assess App (LBAA) within 60 registration;
days from denial of protest or receipt of notice of
assessment (h) Excise taxes
2. If taxpayer is not satisfied with LBAA decision, appeal to
Central Board of Assessment Appeals (CBAA) via notice (i) Percentage or value-added tax
of appeal within 30 days from receipt
(j) Taxes on the gross receipts of transportation contractors and
C. From the CBAA, the dispute may then be taken to the CA by persons engaged in the transportation of passengers or
filing verified petition for review under Rule 43, ROC freight by hire and common carriers by air, land or water,
except as provided in this Code
SC is not convinced with petitioners’ argument that their recourse of
filing before the RTC is proper as they are questioning the very (k) Taxes on premiums paid by way or reinsurance or
authority of respondents to assess and collect taxes and not merely the retrocession;
correctness of the amount. But a perusal of the RTC petition shows that
what is being assailed is the correctness made by local assessors on (l) Taxes, fees or charges for the registration of motor
their property. Nowhere can an allegation of lack of authority is seen vehicles and for the issuance of all kinds of licenses or
permits for the driving thereof
Under the doctrine of primacy of admin remedies, an error in  EXCEPT tricycles
assessment must be administratively pursued to the exclusion of
ordinary courts whose decisions would be void for lack of jurisdiction (m) Taxes, fees, or other charges on Philippine products actually
exported,
Even assuming that the authority is indeed an issue, in order for the  EXCEPT as otherwise provided herein
RTC to resolve, issues on correctness of the tax and assessment and
collection must necessarily be dealt with (n) Taxes, fees, or charges, on Countryside and Barangay
Business Enterprises and cooperatives duly registered
In Ty v. Trampe – the SC held that jurisdiction over the case was under R.A. No. 6810 and the Cooperative Code, respectively
properly vested with the RTC because what was being questioned was
the very authority and power of the assessor, acting solely and (o) Taxes, fees or charges of any kind on the National
independently, to impose the assessment and of the treasurer to collect Government, its agencies and instrumentalities, and
the tax, and not merely of amounts of increase in tax local government units

Ultimately, the petitioners do not question the authority of the assessor; NOTE: Exemption of the National Government from Taxation
the petition involves the correctness of assessments which are - Sec. 133 of the LGC provides a limitation on the power of an
questions of fact and improper for a certiorari, prohibition, mandamus LGU to levy taxes, fees, or charges on the national
petition. government, its agencies and instrumentalities, unless
otherwise provided. Sec. 234 of the LGC on the other hand,
grants to LGUs the power to impose real property tax on FIRST PHILIPPINE INDUSTRIAL CORP. v. CA
properties of the Republic of the Philippines and its political December 29, 1998
subdivisions when its beneficial use is granted to a taxable
person. Summary: FPIC was taxed by Batangas City on FPIC’s operation of an oil
pipeline arguing that FPIC is not a common carrier but a special carrier
which makes FPIC ineligible for the tax exemption for common carrier
CAGAYAN ELECTRIC POWER AND LIGHT CO. v. CITY OF under the LGC. SC held that FPIC is a common carrier.
CAGAYAN DE ORO
November 14, 2012 Doctrine: Art. 1732, Civil Code makes no distinction between a carrier
offering its services to the "general public," i.e., the general community
Summary: CDO City Council passed an Ordinance imposing a tax on the or population, and one who offers services or solicits business only
lease or rental of electric and/or telecommunication posts, poles or from a narrow segment of the general population. Under the LGC, the
towers by pole owners to other pole users at 10% of the annual rental LGU’s taxing power cannot levy on “Taxes on the gross receipts of
income derived from such lease or rental. CEPALCO, after being transportation contractors and persons engaged in the transportation
informed of the passage of the Ordinance, filed a petition for of passengers or freight by hire and common carriers by air, land or
declaratory relief assailing its validity before the CDO City RTC, on the water, except as provided in this Code.” A pipeline concessionaire is a
ground that the tax imposed by the disputed ordinance is in reality a common carrier and is therefore, tax exempt under the LGC.
TAX ON INCOME which CDO City may not impose, the same being
expressly prohibited by Section 133(a), LGC.
PALMA DEVELOPMENT CORP. v. MUNICIPALITY OF MALANGAS
Doctrine: [According to Sir, the important ruling in this case was that October 16, 2003
made by the RTC]. The ordinance is NOT an imposition of income tax
prohibited by Sec.133(a), LGC. Since CEPALCO’s business of leasing its Summary: Palma Devt. Corp. uses the municipal port of Malangas for its
posts to pole users is what is directly taxed, the tax is not upon the business of milling and selling rice and corn to wholesalers in
income but upon the privilege to engage in business. Zamboanga City. The municipality collected fees from Palma, who paid
under protest, pursuant to Section 5G.01, Municipal Revenue Code No. 09
of the municipality, authorizing collection of service fee for the use of
BULACAN v. CA roads leading to the port and for police surveillance of goods sheltered
November 27, 1998 in the wharf. Palma contested the fee on police surveillance, arguing
that it was an imposition, not on the use of roads, but on goods passing
Summary: The Sangguniang Panlalawigan of Bulacan enacted an through the jurisdiction of the municipality, in violation of Section
ordinance providing for the Revenue Code of the Province of Bulacan. 133(e), RA 7160. The RTC nullified the provision. The CA reversed and
Under such ordinance, the province may impose taxes on quarry remanded. The SC ruled that the provision effectively imposed fees on
resources extracted from public lands and beds of bodies of water movement of goods, not vehicles, and was therefore contrary to the
within the territorial jurisdiction of the province. Pursuant to this LGC. It also held that remand was unnecessary.
ordinance, Republic Cement was assessed a tax liability of around
P2.5M for the quarry resources it extracted from private lands. Under Doctrine: By express language of Sections 153 and 155 of RA No. 7160,
the threat of a warrant of levy secured by the Province, Republic LGUs, through their Sanggunian, may prescribe the terms and
Cement paid half of this amount under protest, insisting that the conditions for the imposition of toll fees or charges for the use of any
Province had no authority to impose taxes on quarry resources public road, pier or wharf funded and constructed by them. A service
extracted from private lands. fee imposed on vehicles using municipal roads leading to the wharf is
thus valid. However, Section 133(e) prohibits the imposition, in the
The SC ruled in favor of Republic Cement. The Province of Bulacan is guise of wharfage, of fees – as well as all other taxes or charges in any
without legal authority to impose and assess taxes on quarry resources form whatsoever – on goods or merchandise. It is irrelevant if the fees
extracted from private lands imposed are actually for police surveillance on the goods, because any
other form of imposition on goods passing through the territorial
Doctrine: General Rule: A province may not levy excise taxes on articles jurisdiction of the municipality is clearly prohibited.
already taxed by the National Internal Revenue Code. The National
Internal Revenue Code provides for the imposition of 2% excise tax Class Notes: You can’t circumvent the limitation by calling the tax in
upon minerals, mineral products and quarry resources, excise tax as another name.
follows

The National Internal Revenue Code levies a 2% excise tax on all quarry BATANGAS POWER CORP. v. BATANGAS CITY and NPC
resources, regardless of origin, whether extracted from public or April 28, 2004
private land. Thus, a province may not ordinarily impose taxes on
quarry resources, as the same are already taxed under the National Summary: In the early 1990′s, the country suffered from a crippling
Internal Revenue Code. power crisis. The government, through the National Power Corporation
(NPC), sought to attract investors in power plant operations by
Exception: The province can, however, impose a tax on quarry providing them with incentives, one of which was the NPC’s assumption
resources extracted from public land because it is expressly of their tax payments in the Build Operate and Transfer (BOT)
empowered to do so under the LGC. As to quarry resources extracted Agreement. Enron Power Development Corporation (Enron) and NPC
from private land, however, it may not do so. entered into a BOT Project. The BOT Agreement provided that NPC shall
be responsible for the payment of all taxes imposed on the power
Class Notes: Taxing power of national government vis-à-vis taxing station except income & permit fees. Subsequently, Enron assigned its
power of LGU obligation under the BOT Agreement to Batangas Power Corporation
- General Rule: Mutually exclusive (BPC).
The BOI issued a certificate of registration to BPC as a pioneer
 If something is already taxed by the national enterprise entitled to a tax holiday of 6 years. Batangas City demanded
government, it can no longer be taxed by the LGU against BPC the payment of business taxes & penalties. BPC refused to
- Exception: Unless otherwise provided in the LGC pay citing its tax exemption as a pioneer enterprise for 6 years. Also,
BPC asserted that the city should collect the taxes from NPC since the
latter assumed responsibility for their payment under the BOT
Agreement. The NPC intervened that while it admitted assumption of
the BPC’s tax obligations under the BOT Agreement, it refused to pay FRANCHISE TAXES
BPC’s business tax as it allegedly constituted an indirect tax on NPC
which is a tax-exempt corporation under its Charter. The SC ruled that Sec. 137 LGC – Notwithstanding any exemption granted by law or other
with the passage of the LGC, the NPC no longer enjoys its tax-exempt special law, the PROVINCE may impose a tax on businesses, enjoying a
status provided in its own Charter. franchise
- RATE: Not exceeding 50% of 1% of gross annual receipts for
Doctrine: The effect of the LGC on the tax exemption privileges of the the preceding calendar year
NPC and other government instrumentalities has already been
extensively discussed and settled in National Power Corporation v. City NOTE: Now, provinces may impose a tax on any business enjoying a
of Cabanatuan. In said case, the Court recognized the removal of the franchise even if the business has been exempted from taxes by virtue
blanket exclusion of government instrumentalities from local taxation of any law
as one of the most significant provisions of the 1991 LGC. Specifically,
the SC stressed that Section 193 of the LGC, an express and general Withdrawal of Tax Exempt Privileges
repeal of all statutes granting exemptions from local taxes, withdrew - Sec. 193 provides that tax exemptions or incentives granted
the sweeping tax privileges previously enjoyed by the NPC under its to, or presently enjoyed by all persons, whether natural or
Charter. juridical, including GOCCs, are WITHDRAWN upon effectivity
of LGC
In recent years, the increasing social challenges of the times expanded
 UNLESS otherwise provided in the LGC
the scope of state activity, and taxation has become a tool to realize
- EXCEPTIONS (meaning they still enjoy exemption):
social justice and the equitable distribution of wealth, economic
1. Local water districts
progress and the protection of local industries as well as public welfare
2. Cooperatives duly registered under RA 6938
and similar objectives. Taxation assumes even greater significance with
3. Non-stock and non-profit hospitals and educational
the ratification of the 1987 Constitution. Thenceforth, the power to tax
institutions
is no longer vested exclusively on Congress; local legislative bodies are
- But Congress may still grant exemption after the LGC
now given direct authority to levy taxes, fees and other charges. The
effectivity
only way to shatter the culture of dependence is to give the LGUs a
 This will not be covered by the withdrawal provision in
wider role in the delivery of basic services, and confer them sufficient
Sec. 193
powers to generate their own sources for the purpose.
See also the repealing clause in Sec. 534
LEPANTO CONSOLIDATED MINING v. AMBANLOC
June 29, 2010
MERALCO v. PROVINCE OF LAGUNA
May 5, 1999
Summary: Lepanto Consolidated Mining Company (Lepanto) was issued
by the national government a mining lease contract covering, among
Summary: The Province of Laguna enacted an Ordinance which levies
others, a mining claim in the Municipality of Mankayan, Buenget. The
franchise tax on businesses enjoying a franchise at a rate of 50% of 1%
contract granted Lepanto the right to extract and use for its purposes all
of the gross annual receipts. MERALCO was a grantee of a franchise
mineral deposits within the boundary lines of its mining claim. The
given by various municipalities of Laguna. MERALCO paid in protest
Mines and Geo-Sciences Bureau of the Department of Environment and
and subsequently asked a refund of the tax it paid arguing in the main
Natural Resources also advised Lepanto that under its contract, it did
that The imposition of the franchise tax by the ordinance contravened
not have to get a permit to extract and use sand and gravel from within
Sec. 1 of PD 551 which states that the franchise tax in PD 551 shall be in
the mining claims for its operational and infrastructure needs. Lepanto
lieu of all taxes and assessments of whatever nature imposed by any
used the quarried materials to back-fill portions of the earth excavated
national or local authority on earnings, receipts, income, and privilege.
as a result of the mining. It also used sand and gravel to construct
SC denied and held that LGC should govern and that MERALCO is liable
structures needed in its mining operations. Respondent Ambanloc,
for the tax imposed by the province.
provincial treasurer sent a demand letter to Lepanto for the payment of
P1,901,893.22 as sand and gravel tax. Lepanto filed a petition with the
Doctrine: Indicative of the legislative intent to carry out the
RTC to question the assessment.
Constitutional mandate of vesting broad taxing powers:
1. Sec. 193 of LGC effectively withdrawn tax exemptions or
The RTC found Lepanto liable, which was affirmed by the CTA and ruled
incentives enjoyed by certain entities
that sand and gravel taxes may be imposed even on non-commercial
2. See also Sec. 534 (f) on the general repealing clause
extractions.
On the phrase “in lieu of all taxes” in PD 551. In City Gov. of San Pablo v.
The SC affirmed the ruling of the CTA and found that non-commercial
Reyes the Court held that the phrase “in lieu of all taxes” have to give
extractions are covered by Revised Benguet Revenue Code. Lepanto is
way to the peremptory language of the LGC specifically providing for
not exempted from the payment of the taxes despite the fact that the
the withdrawal of such exemptions, privileges. Upon effectivity of LGC
extraction was intended for its personal use since the Revised Benguet
all exemptions except only as provided therein can no longer be
Revenue Code does not make any distinction between commercial and
invoked by MERALCO to disclaim local tax liability.
non-commercial uses.
Class Notes:
Doctrine: Sec. 138, RA 7160 is a general law that delegates to provinces
1. In Basco, the SC said that the LGU has no inherent power to
the power to impose taxes on the extraction of quarry resources.
tax. This is inaccurate today.
However, the question of determining Lepanto’s liability shall be
determined based on the revenue measure itself, which is the Revised  In the present Constitution, LGUs are given broad taxing
Benguet Revenue Code. The said Code does not distinguish between powers but subject to limitations imposed by Congress
commercial and non-commercial purposes. Under the Code, even  Congress can limit but it cannot deprive the LGU of such
entities that extract sand and gravel for personal purposes are required power
to pay the taxes before they are issued a permit. Likewise, it is settled 2. Bases for the withdrawal of the exception
that provincial governments can levy excise taxes on quarry resources a. Sec. 137 – “notwithstanding any exception…”
independently from the national government. b. Sec. 193 – blanket withdrawal of exemptions granted
before LGC effectivity
c. Sec. 534 – repealing clause
3. Is it accurate to say that all franchises are liable to franchise
tax? Class Notes: What if NPC fails to pay?
 NO. A franchise grantee may be given exemptions after - Similar tax remedies but take note that because of the nature
LGC effectivity, in which case it is not covered by the of NPC, being an instrumentality of the government, there
blanket removal provision in Sec 193 are properties which are exempt from levy

PLDT v. CITY OF DAVAO DIGITEL v. PROVINCE OF PANGASINAN


August 22, 2001 February 23, 2007

Summary: PLDT was applying with the City of Davao for a franchise. Its Summary: Digitel has the provincial franchise to install, maintain and
application was withheld pending the payment by PLDT of its local operate a telecommunications system within the territorial jurisdiction
franchise tax amounting to P3,681,985.72. This amount was protested of the Province of Pangasinan. Section 137 of the LGC expressly
by PLDT alleging that it was exempted from payment of franchise tax authorized local governments to impose franchise tax on businesses in
and that it was entitled to a refund of its previous payment. It relied on their jurisdiction. Pangasinan imposed franchise tax on Digitel. Digitel
the Bureau of Local Government Finance (BLGF) opinion and on Sec. 23 refused to pay contending that since its competitors were exempted
of RA 7925 which allegedly extended to PLDT the exemption granted to from paying and an act by congress was passed providing for equity
Smart and Globe. SC ruled that nothing in the said provision provides among competitors then it was also exempted from paying franchise
for a tax exemption. It pertains to regulatory and reportorial tax. SC ruled for the Province of Pangasinan. In view of the unequivocal
requirements. In sum, PLDT was not exempted from local franchise. It is intent of Congress to exempt from real property tax those real
also not entitled to a tax refund. properties actually, directly and exclusively used by petitioner DIGITEL
in the pursuit of its franchise, Pangasinan can only levy real property
Doctrine: Tax exemptions are highly disfavored. Tax exemptions must tax on the remaining real properties of the grantee located within its
be expressed in the statute in clear language that leaves no doubt of the territorial jurisdiction not part of the above-
intention of the legislature to grant such exemption. And, even if it is
granted, the exemption must be interpreted in strictissimi juris against Doctrine: Tax exemptions are highly disfavored. The tax exemption
the taxpayer and liberally in favor of the taxing authority. must be expressed in the statute in clear language that leaves no doubt
of the intention of the legislature to grant such exemption. And, even if
SC said that the interpretation of PLDT is absurd! This could not have it is granted, the exemption must be interpreted in strictissimi juris
been the intent of Congress. This will leave the Government with the against the taxpayer and liberally in favor of the taxing authority.
burden of having to keep track of all granted telecom franchises.
The grant of taxing powers to local government units under the
“Exemption” in Sec. 23 is too general; so SC ascertained legislative Constitution and the LGC does not affect the power of Congress to grant
intent. What is apparent is that RA 7925 is a legislative enactment exemptions to certain persons, pursuant to a declared national policy.
designed to set the national policy on telecoms and provide the The legal effect of the constitutional grant to local governments simply
structures to implement it to keep up with the technological advances means that in interpreting statutory provisions on municipal taxing
in the industry and needs of the public. There is nothing in the language powers, doubts must be resolved in favor of municipal corporations.
of RA 7925 nor in the House and Senate proceedings which show that it
contemplates the grant of tax exemptions to all telecom entities, The effect of Section 23 of Republic Act No. 7925 – that the word
including those whose exemptions were withdrawn by the LGC "exemption" as used in the statute refers or pertains merely to an
exemption from regulatory or reporting requirements of the DOTC or
“Exemption” could simply refer to exemption from certain regulatory or the NTC and not to the grantee’s tax liability.
reporting requirements bearing in mind the policy of the law. When
exemption is claimed it must be shown indubitably to exist Class Notes: NOTE the pronouncement of the court about the
abolishment of franchise tax to give way to VAT:
- However, in light of the passage of RA 7716 (VAT
NPC v. CITY OF CABANATUAN restructuring law), it abolished the franchise tax imposed to
April 9, 2003 telecom companies and in its place imposed a 10% VAT
 So the IN LIEU class in the legislative franchises of
Summary: Cabanatuan City assessed NAPOCOR a franchise tax based on Globe. Smart, among others have now become functus
its gross income in the city for the year 1992. NAPOCOR argues that it is officio
exempt citing its charter. Cabanatuan City cites Sec. 193 (LGC 1991) as  Thus, from Jan 1, 1996 (effectivity of the VAT law)
a repeal of NAPOCOR’s tax exemption. RTC: NAPOCOR is still exempt, Digitel ceased to be liable for national franchise tax and
RA 7160 is a general law which can’t repeal a specific law, and anyway in its stead imposed a 10% VAT
it is only an implied repeal which is frowned upon. CA: RTC overruled.
RA 7160 is an express withdrawal of NAPOCOR’s exemption. SC: CA
ruling upheld. SMART v. CITY OF DAVAO
September 16, 2008
Doctrine: As a general rule, LGUs cannot impose taxes, fees or charges of
any kind on the National Government, its agencies and Summary: Smart filed an action for declaratory relief, contending that it
instrumentalities. This rule now admits an exception, when specific is exempt from local taxation because of the phrase “in lieu of all taxes”
provisions of the LGC authorize the LGUs to impose taxes, fees or in its franchise
charges on the aforementioned entities. In enacting the LGC, Congress
exercised its prerogative to tax instrumentalities and agencies of The SC ruled that Smart is not exempt from local franchise tax. (1) Sec.
government as it sees fit. Nothing prevents Congress from decreeing 193 of the LGC withdrew exemptions granted before it was passed and
that even instrumentalities or agencies of the government performing this does not include those granted after; however, the franchise of
governmental functions may be subject to tax. Smart does not expressly provide what kind of taxes Smart is exempt
from. Tax exemptions are construed strictly against the taxpayer so the
A franchise tax is a tax on the privilege of transacting business in the provision must be interpreted to mean that Smart is not exempt from
state and exercising corporate franchises granted by the state. It is local franchise tax; the provision in question is functus officio already
imposed not on the ownership or mere existence but on the exercise by with the EVAT Law which subjects telecommunications companies to
the corporation of a privilege to do business. VAT instead of franchise taxes; (2) Tax exemption and exclusion have
the same nature and effect and there is no essential difference between AMUSEMENT TAXES
them; (3) the findings of BLGF as a consultative agency is not binding
on the courts; (4) There is no intent on the part of Congress to give a Sec. 140, LGC
blanket tax exemption to all telecommunication entities by its grant to 1. The PROVINCE may levy an amusement taxes on proprietors,
Globe; and (5) the franchise of Smart which constitutes the contract lessees or operators of theaters, cinemas, concert halls,
between it and the government does not provides for such exemption. circuses, boxing stadia and other places of amusement
 RATE: Not more than 30% gross receipts from
Doctrine: The withdrawal of tax exemptions or incentives provided in
admission fees
the LGC can only affect those franchises granted prior to its effectivity.
2. In case of theaters or cinemas
a. Tax shall first be deducted and withheld by proprietors,
[National] franchise tax on telecommunications companies has become
lessees, or operators
functus officio because of RA 7716, as amended by the Expanded Value
b. And then paid to the provincial treasurer before gross
Added Tax Law (RA 8241), subjecting them to VAT.
receipts are divided between said operators, lessees or
operators and distributors of the films
The franchise tax that an LGU may impose must comply with Sections
137 and 151 of RA 7160. It must not exceed 50% of 1% of the gross
3. EXEMPTED FROM AMUSEMENT TAX:
annual receipts for the preceding calendar year based on the income on
a. The HOLDING of operas, concerts, dramas, recitals,
receipts realized within the territorial jurisdiction of the LGU.
painting, art exhibitions, flower shows, musical
programs, literary and oratorical presentations
b. EXCEPT:
CITY OF IRIGA v. CAMARINES SUR III ELECTRIC COOPERATIVE (1) Pop, rock or similar concerts
September 5, 2012

Summary: The city of Iriga assessed local taxes on Camarines Sur III PHILIPPINE BASKET ASSOCIATION v. CA
Electric Cooperative, Inc. (CASURECO III), an electric cooperative. The August 8, 2000
latter refused to pay the taxes on the ground that it is exempted from
doing so pursuant to PD 269. The SC ruled that CASURECO is not
Summary: The CIR assessed PBA for deficiency amusement tax. The
exempt from paying the local taxes and that the LGUs have the power to
PBA contested the assessment, but such was denied by the CIR. The CTA
impose and collect such taxes.
and CA both upheld the ruling of the CIR. PBA was contending that,
among other things, the power to levy and collect amusement taxes has
Doctrine: Sec. 193 of the LGC withdrew tax exemptions or incentives
been transferred from the national government to the local
previously enjoyed by "all persons, whether natural or juridical,
government, therefore BIR has no authority to collect. The SC, in
including government-owned or controlled corporations, except local
denying PBA’s petition, declared that the amusement tax on
water districts, cooperatives duly registered under R.A. No. 6938, non-
professional basketball games is in the nature of a national tax.
stock and non-profit hospitals and educational institutions."… The
power of the local government units to impose and collect taxes is
Doctrine: Amusement tax on the gross receipts from the sale of
derived from the Constitution itself which grants them "the power to
admission tickets for professional basketball games is national in
create its own sources of revenues and to levy taxes, fees and charges
nature. Therefore, the BIR has the authority to collect and levy such.
subject to such guidelines and limitation as the Congress may provide."
Class Notes:
- This case is modified by Pelizloy
- What exactly is taxed by Sec. 140? The venue of the activity?
 Sir said it is vague since some items seem to tax the
venue while some items tax the activity
 In any case, argue your stand.

FILM DEVELOPMENT COUNCIL OF THE PHIL. v. SM PRIME


HOLDINGS INC.
April 3, 2013

Summary: RA 9167 was passed creating the Film Development Council.


Quality movies were given “amusement tax rewards.” The City of Cebu
filed a petition for declaratory relief in the RTC of Cebu, assailing the
constitutionality of this law. FDC sued SM Prime Holdings in RTC of
Pasig for the payment of the amusement tax rewards. The Court held
that the Cebu Case is the appropriate vehicle to determine the rights of
the parties.

Doctrine: The presence of City of Cebu as party plaintiff in the Cebu Case
would afford proper relief to respondent in the event the Cebu City RTC
renders judgment sustaining the validity of the said provisions.
Therefore, the defense raised by SM Prime of prior payment in good
faith would be best ventilated in the Cebu Case where the City of Cebu is
a party.
The requisites of litis pendentia are (a) the identity of parties, or at least
such as representing the same interest in both actions; (b) the identity
of rights asserted and relief prayed for, the relief being founded on the
same facts, and (c) the identity of the two cases such that judgment in
one, regardless of which party is successful, would amount to res
judicata in the other.
PELIZLOY REALTY CORPORATION v. PROVINCE OF BENGUET
April 10, 2013 REAL PROPERTY TAXATION
Summary: Pelizloy is protesting the imposition of tax upon it on the
argument that the Municipality of Tuba Benguet is levying upon it a Real Property Tax = LGU Property
percentage tax not provided among its powers, hence ultra vires. Dean - Real property taxes belong solely to LGUs
Leonen clarifies that percentage tax is within the power of the - National government no longer has any share in the real
municipal government to impose however, the subject of such tax is property taxes collected by the local governments
limited to amusement places, and under the statutory interpretation of
ejusdem generis Pelizloy doesn’t qualify under it being a resort. Fundamental Principles (Sec 198)
1. Real property shall be appraised at its current and fair
Doctrine: Based on the principle of ejusdem generis, the resorts are not market value
included in the enumeration of what comprises places of amusement 2. Real property shall be classified for assessment purposes on
which are already defined by black letter law, and is further clarified in the basis of its actual use
jurisprudence which denotes “places of amusement” as where “one 3. Real property shall be assessed on the basis of a uniform
seeks admission to entertain oneself by seeing or viewing shows or classification within each LGU
performances” In this case, the definition of' amusement places' in 4. Appraisal, assessment, levy and collection of real property
Section 131 (c) of the LGC is a clear basis for determining what tax shall not be let to any private person
constitutes the 'other places of amusement' which may properly be 5. Appraisal and assessment shall be equitable
subject to amusement tax impositions by provinces. There is no reason
for going beyond such basis. To do otherwise would be to countenance Appraisal of Real Property
an arbitrary interpretation/application of a tax law and to inflict an - All real property, taxable or exempt, shall be appraised at the
injustice on unassuming taxpayers. current and fair market value prevailing in the locality where
the property is situated (Sec. 201)
Class Notes: This case highlights the parameters of taxing power.  To claim exemption, declarant shall file substantiating
A. The LGC establishes the parameters of the taxing powers of documents to the provincial, city, or municipal assessor
LGUs within 30 days from date of declaration
1. Sec. 130 – fundamental principles - Owners, administrators or their duly authorized
a. Taxation shall be uniform in each LGU representatives are required to file with the local assessor a
b. Taxes, fees, charges, and other impositions shall: sworn statement declaring the current and fair market value
(1) Be equitable and based as far as practicable of their real property (Sec. 202)
on the taxpayer’s ability to pay  Taxable or exempt; Previously declared or undeclared
(2) Be levied and collected only for public  WHEN to file: Once every 3 years during Jan. 1 to June
purposes 30 starting 1992
(3) Not unjust, excessive, oppressive, - If owners or administrators fail to file declaration, local
confiscatory assessors are authorized under Sec. 204 to make the
(4) Not contrary to law, public policy, national declaration and assessment
economic policy, or in restraint of trade
2. Collection of local taxes, fees, charges and other Assessment Roll of Real Properties
impositions shall in no case be let to any private person - Under Sec. 205, provincial or city assessor and the municipal
3. Revenue collected pursuant to provisions of LGC shall assessor of the Metropolitan Manila Area are mandated to
inure solely to the benefit of, and be subject to the prepare and maintain an assessment roll where all real
disposition by the LGU levying the tax, fee, charge or properties located in their respective jurisdiction are listed,
other imposition whether exempt or not
a. Unless otherwise specifically provided for in the
LGU Preparation of Schedule of Fair Market Values (Sec. 212)
4. Each LGU shall, as far as practicable, evolve a 1. Enacted by Ordinance
progressive system of taxation  A schedule of the FMVs of real properties prepared by
5. Sec, 133 provides the common limitations the provincial, city, and municipal assessors of the
Metro Manila Area shall be enacted by ordinance by the
sanggunian concerned before any general revision of
property assessment is made
 See Ty v. Trampe

2. Posting and Publication Requirement


 Schedule shall be published in a newspaper of general
circulation in the province, city, or municipality
concerned
 If no such newspapers, post in the provincial capitol,
city/municipal hall, and in 2 other conspicuous public
places

General Revision of Assessments and Property (Sec. 219)


- General revisions of assessments and property classifications
can be done within 2 years from LGC effectivity and every
three years thereafter

Effectivity of Assessments (Sec. 221)


- GENERAL RULE: All assessments or reassessments made
after Jan. 1 of any year shall take effect on Jan 1 of the next
year
- EXCEPTIONS: However, certain reassessments take effect at instrumentalities or agencies of the government, which are
the beginning of the quarter following the reassessments. neither corporations nor agencies integrated within the
 These are reassessments due to: departmental framework, but vested by law with special
1. Partial/total destruction of real property functions or jurisdiction, endowed with some if not all
2. Major change in its actual use corporate powers, administering special funds, and enjoying
3. Any great and sudden inflation or deflation of real operational autonomy usually through a charter including,
property values but not limited to, the following:
4. Gross illegality of the assessment (1) the Manila International Airport Authority (MIAA),
5. Any other abnormal cause (2) the Philippine Ports Authority (PPA),
 In these cases, reassessment is made within 90 days (3) the Philippine Deposit Insurance Corporation (PDIC),
from date of occurrence of the circumstance (4) the Metropolitan Waterworks and Sewerage System
(MWSS),
Assessment Appeals (5) the Laguna Lake Development Authority (LLDA),
1. Any owner or person having legal interest in the property (6) the Philippine Fisheries Development Authority
who is not satisfied with the action of the assessor in the (PFDA),
assessment may appeal to the Local Board of Assessment (7) the Bases Conversion and Development Authority
Appeals (BCDA), the Cebu Port Authority (CPA),
 Within 60 days from receipt of the written notice of (8) the Cagayan de Orb Port Authority,
assessment (9) the San Fernando Port Authority,
2. Decisions by the Local Board appealable to Central Board of (10) the Local Water Utilities Administration (LWUA) and
Assessment Appeals (11) the Asian Productivity Organization (APO).

Properties Exempt from Real Property Taxes (Sec. 234) 3. Sec. 3(o) - Government-Owned or -Controlled Corporation
1. Ownership Exemptions: real properties owned by: (GOCC) refers to any agency organized as a stock or nonstock
a. The Republic corporation, vested with functions relating to public needs
b. A province whether governmental or proprietary in nature, and owned
c. A city by the Government of the Republic of the Philippines directly
d. A municipality or through its instrumentalities either wholly or, where
e. A barangay applicable as in the case of stock corporations, to the extent
f. Registered cooperatives of at least a majority of its outstanding capital stock:
Provided, however, That for purposes of this Act, the term
NOTE: Real property owned by the Republic and its “GOCC”- shall include GICP/GCE and GFI as defined herein.
political subdivisions is NOT exempt when the
beneficial use thereof has been granted, for 4. Sec. 3(p) - Nonchartered GOCC refers to a GOCC organized
consideration or otherwise, to a taxable person and operating under Batas Pambansa Bilang 68, or “The
Corporation Code of the Philippines”.
2. Character Exemptions:
a. Charitable institutions
b. Houses and temples of prayer like churches, LOPEZ v. CITY OF MANILA
parsonages, or convents appurtenant thereto, mosques February 19, 1999
c. Non-profit religious cemeteries
Summary: Pursuant to a Memorandum issued by Department of
3. Usage Exemptions: on the basis of ACTUAL, DIRECT, AND Finance, the City Assessor prepared a proposed schedule of fair market
EXCLUSIVE use: values of real properties. This was published in newspapers (Manila
a. Lands, buildings, and improvements which are used for Standard and Balita) and was later on enacted as Manila City Ordinance
religious, charitable, or educational purposes (Revised Schedule of Fair Market Values of Real Properties of the City of
b. Machineries and equipment used by LWDs or GOCCs Manila) which increased tax assessments to 1, 020% was enacted. This
engaged in the supply and distribution of water and/or ordinance was subjected to regular process of ordinance enactment..
generation and transmission of electric power
c. Machinery, equipment used for pollution control and As a result of such ordinance, Lopez’s tax on his land and on the
environmental protection improvements increased. Lopez filed a special proceeding for the
declaration of nullity of a alleging that these are unjust, excessive,
All other exemptions previously granted to natural or juridical persons oppressive, or confiscatory. He also questioned the application of Sec
including GOCCs were withdrawn upon the effectivity of the LGC. 212 and 221 of the LGC. The RTC dismissed the petition for failure to
exhaust administrative remedies.
Special Law cited in the Syllabus – RA 10149 (Act to promote financial
viability and fiscal discipline in GOCCs) The SC affirmed. Sec 212 on the procedure on the preparation of the
1. Sec. 3(m) - Government Financial Institutions (GFIs) refer to schedule of fair market values of real properties was complied with. On
financial institutions or corporations in which the the argument that base on Sec 221 of the LGC, the Ordinance which
government directly or indirectly owns majority of the took effect on Jan 1, 1996 could not be implemented on the same year
capital stock and. which are either: because it should be implemented on the following year, the Court
(1) registered with or directly supervised by the Bangko ruled that it has become moot and academic due to the enactment of
Sentral ng Pilipinas; or Manila Ordinance 7905 which provides for retroactive effect of the
(2) collecting or transacting funds or contributions from reduced tax assessments.
the public and places them in financial
instruments or assets such as deposits, loans, bonds and Doctrine: Mandatory Conduct of General Revision of Real Property
equity including, but not limited to, the Government assessments pursuant to Sec 219 of the LGC:
Service Insurance System and the Social Security (1) The preparation of Schedule of Fair Market Values.
System. (2) The enactment of Ordinances:
(a) levying an annual "ad valorem" tax on real property and
2. Sec. 3(n) - Government Instrumentalities with Corporate an additional tax accruing to the SEF.
Powers (GICP)/Government Corporate Entities (GCE) refer to
(b) fixing the assessment levels to be applied to the market Vasquez approved the findings and recommendations of the
values of real properties; Ombudsman. The SC modified the resolution, suspending Callanta,
(c) providing necessary appropriation to defray expenses Delos Reyes, and Concon for 1 year while the rest were reprimanded.
incident to general revision of real property All of them were also warned.
assessments; and
(d) adopting the Schedule of Fair Market Values prepared The Court found that the City Assessor had no authority to reconsider
by the assessors. real property assessments as based on the Real Property Tax Code, this
power is vested on the Local Board of Assessment Appeals. As regards
The preparation of fair market values as a preliminary step in the injury or prejudice to the City, the Court held that the right to collect the
conduct of general revision was set forth in Section 212 of LGC, to wit: taxes had vested on the city upon the issuance of the notice, and this
(1) The city or municipal assessor shall prepare a schedule of right had fully vested upon failure to make a correct appeal. The
fair market values for the different classes of real property certification to the Secretary of Finance had been dispensed with by the
situated in their respective LGUs for the enactment of an LGC. Finally, the Court found the penalties too harsh considering the
ordinance by the sanggunian concerned. lack of any indicators of corruption or graft based on the black letter
(2) The schedule of fair market values shall be published in a law.
newspaper of general circulation in the province, city or
municipality concerned or the posting in the provincial Doctrine: The power to act on requests for reconsideration on appeals
capitol or other places as required by law. of property owners explicitly vested on the LBAA, NOT the City
Assessor. The City Assessor’s participation in the process ends with the
Class Notes: sending of the notice of new assessments.
- What are the remedies of a taxpayer?
1. Sec. 187 LGC – taxpayer may question the Determination of injury/prejudice requires determining when the
constitutionality or legality of tax ordinance on appeal assessments became due and payable.
within 30 days from effectivity to Justice Sec.
2. Sec. 226 LGC – owner of real property not satisfied with Even if §23 seems to make the Certification a condition sine qua non for
assessment may within 60 days from notice of the effectivity of the revised values, the phrase “the assessments shall
assessment appeal to the BAA become effective and taxes shall accrue and be payable thereunder in
3. Sec. 252 LGC – if taxpayer questions the excessiveness accordance with the provisions of this Code.” points to the rest of the
of the amount of tax, he should pay under protest first RPTC as guiding as to the effectivity of the new assessments. The
and then appeal to the BAA specific provision is §24, which states that “the assessments take effect
on the first day of January of the succeeding year after the revision is
made.”

YAMANE v. BA LEPANTO CONDOMINIUM CORPORATION The LGC has dispensed with obliterated the Certification requirement
February 13, 2005 in §23 of the Real Property Tax Code, such that Certification of
completion of general revision of property assessments is
Summary: In 1998, BA Lepanto Condominium Corporation (Lepanto) unquestionably not a precondition to the effectivity of the new
received a tax assessment in the amount of P1.6 million from Luz assessments.
Yamane, the City Treasurer of Makati, for business taxes. Lepanto
protested the assessment as it averred that Lepanto, as a corporation, is An assessment fixes and determines the tax liability of a taxpayer. As
not organized for profit; that it merely exists for the maintenance of the soon as it is served, an obligation arises on the part of the taxpayer
condominium. Yamane denied the protest. Lepanto then appealed the concerned to pay the amount assessed and demanded.
denial to the RTC of Makati. RTC Makati affirmed the decision of
Yamane. Lepanto then filed a petition for review under Rule 42 with the With respect to real property taxes, the obligation to pay arises on the
Court of Appeals. The Court of Appeals reversed the RTC. first day of January of the year following the assessment. Corollarily, on
the same date, the right of the local government to collect said taxes
Doctrine: The fees collected from condo-unit owners redound to their also arises. And where the taxpayer fails to question such assessment
benefit and so the Corporation operating the condominium and within the reglementary period provided by law, the local government’s
collecting such cannot be considered as being organized for profit. right becomes absolute upon the expiration of such period with respect
Thus, they are not liable for the tax assessment. The notice of to that taxpayer’s property.
assessment, which stands as the first instance the taxpayer is officially
made aware of the pending tax liability, should be sufficiently
informative to apprise the taxpayer the legal basis of the tax. The TY v. TRAMPE
Condominium Act that a condominium corporation is precluded by December 1, 1995
statute from engaging in corporate activities other than the holding of
the common areas, the administration of the condominium project, and Summary: In 1994, Municipal Assessor of Pasig (now a city) sent a
other acts necessary, incidental or convenient to the accomplishment of notice of assessment respecting certain real properties of Alejandro Ty
such purposes. Neither the maintenance of livelihood, nor the and MVR Picture Tube Inc. located in Pasig, Metro Manila. Ty and MVR
procurement of profit, fall within the scope of permissible corporate filed with the RTC of NCR Petition for Prohibition with prayer for writ
purposes of a condominium corporation under the Condominium Act. of preliminary injunction to declare null and void the new tax
assessments and to enjoin the collection of real estate taxes. RTC judge
Trampe denied the petition. MR was also denied. Ty and MVR filed a
CALLANTA v. OFFICE OF THE OMBUDSMAN petition for review with SC. SC reversed the decision stating that there
January 30, 1998 was no express nor implied repeal, there was no failure to exhaust
administrative remedies. Schedule of Market Values for properties in
Summary: This case originates from administrative and criminal Pasig City prepared by respondent Assessor, as well as the
complaints by the City of Cebu against Callanta, Edera, Pahamtang, corresponding assessments and real estate tax increases based thereon
Lucero, Abellana, Abarri, Salares, Delos Reyes, Concon, and Palmero are NULL and void
(from the Office of the City Assessor), for reassessments they made as
to the valuations for certain properties. As regards the administrative Doctrine: Sec. 9 of P.D. 921 requires that the schedule of values of real
aspect of the case, the Deputy Ombudsman ordered the dismissal of properties in the Metropolitan Manila area shall be prepared jointly by
Callanta while the rest were to be suspended for 3 months. Ombudsman the city assessors in the districts created therein while Sec. 212 of R.A.
7160 states that the schedule shall be prepared "by the provincial, city when the tax assessment is made on the basis of the actual use of the
and municipal assessors of the municipalities within the Metropolitan property.
Manila Area for the different classes of real property situated in their
respective local government units for enactment by ordinance of the In either case, the unpaid realty tax attaches to the property but is
sanggunian concerned. directly chargeable against the taxable person who has actual and
beneficial use and possession of the property regardless of whether or
The 2 laws are not co-extensive and mutually inclusive in their scope not that person is the owner.
and purpose. While R.A. 7160 covers almost all governmental functions
delegated to local government units all over the country, P.D. 921 Furthermore, contractual liability alone cannot be deemed “an interest
embraces only a “progressive revenue raising program that will not in the property” and basis for the enforcement of tax liabilities against it
unduly burden the tax payers in Metropolitan Manila the Metropolitan by the Municipality.
Manila area. There is no implied repeal.
In previous cases, the Court has upheld a contractual liability as being
Class Notes: the basis for tax liabilities against an entity who was the non-owner of
- In harmonizing, the SC said: We can harmonize Sec. 9 and the property. However, in those cases the effectiveness of the tax
Sec. 212: liabilities assumed by the vendees was based on their possession and
1. Assessor in each municipality or city in MM shall use of the property subject to tax.
prepare his/her proposed schedule of values following
Sec. 212 LGC In other words, the contractual assumption of liability was
2. Local Treasury and Assessment District shall then meet supplemented by an interest that the party assuming liability had on
as per Sec. 9, PD 921 the property taxed;
a. We follow the districts in Sec. 1 of PD 921
b. Here, different assessors shall compare their Class Notes
individual assessments, discuss, and then jointly - Basis for liability:
agree and produce a schedule of values for their 1. Ownership
district 2. Legal interest – simply means beneficial ownership
c. Note: they should evolve a progressive revenue
raising program that will not unduly burden the
taxpayers MCIAA v. MARCOS
3. Schedule jointly agreed shall then be published in a September 11, 1996
newspaper of general circulation and submitted to
sanggunian concerned for enactment by ordinance as Summary: MCIAA was created by RA 6958. Under Sec. 14 of said
per Sec. 212, LGC Charter, MCIAA was exempted from the payment of real property taxes.
- Sir said that the SC made an entirely new procedure which Subsequently, the LGC was enacted. Pursuant to the Code, Cebu City OIC
was improper. Treasurer assessed MCIAA real property taxes. MCIAA objected to such
assessment, arguing that (a) it was exempted from paying RPT under
Sec. 14 of its Charter, and (b) as an instrumentality of the National
NAPOCOR v. PROVINCE OF QUEZON Government, LGUs cannot assess taxes against it, pursuant to Sec.133
July 15, 2009 (o), LGC. Cebu City refused to cancel the assessment and claimed that
under Sec. 193 and 234, LGC, MCIAA’s exemption, as a GOCC, has been
Summary: The NPC entered into an Energy Conversion Agreement withdrawn. MCIAA paid under protest and filed a petition for
(ECA) with the private corporation Mirant. declaratory relief with the RTC of Cebu. RTC dismissed its petition and
held that MCIAA was liable to pay RPT. SC affirmed such decision.
Mirant will build and finance a coal-fired thermal power plant on the
lots owned by the NPC in Pagbilao, Quezon for a period of 25 years. The Doctrine: The power to tax is primarily vested in the Congress;
NPC, in turn, will supply the necessary fuel to be converted by Mirant however, in our jurisdiction, it may be exercised by local legislative
into electric power, take the power generated, and use it to supply the bodies, no longer merely by virtue of a valid delegation as before, but
electric power needs of the country. At the end of the 25-year term, pursuant to direct authority conferred by Section 5, Article X of the
Mirant will transfer the power plant to the NPC without compensation. Constitution. Under the latter, the exercise of the power may be subject
Among the obligations undertaken by the NPC under the ECA was the to such guidelines and limitations as the Congress may provide which,
payment of all taxes that the government may impose on Mirant. however, must be consistent with the basic policy of local autonomy.

After the power plant was built,, Municipality of Pagbilao, Quezon Reading together Section 133, 232 and 234 of the LGC, it can be
assessed Mirant’s real property taxes on the power plant and its concluded that:
machineries . - GENERAL RULE: pursuant to Sec. 133, the taxing powers of
LGUs cannot extend to the levy of "taxes, fees, and charges of
The NPC objected to the assessment against Mirant on the claim that, as any kind on the National Government, its agencies and
the entity liable for the tax, it (the NPC) is entitled to the tax exemptions instrumentalities, and local government units"
provided in Section 234, paragraphs (c) and (e) of the LGC.
- EXCEPTION: pursuant to Sec. 232, provinces, cities,
Doctrine: On who has personality to contest an assessment: Section 226 municipalities in the Metropolitan Manila Area may impose
of the LGC lists down the two entities vested with the personality to the real property tax
contest an assessment: the owner and the person with legal interest in
the property. The liability for taxes generally rests on the owner of the - EXCEPTION TO EXCEPTION (RPT-exempt): on "real property
real property at the time the tax accrues. However, a person legally owned by the Republic of the Philippines or any of its
burdened with the obligation to pay for the tax imposed on a property political subdivisions
has legal interest in the property AND the personality to protest a tax
assessment on the property. - EXC to EXC to EXC (RPT-liable): when the beneficial use
thereof has been granted, for consideration or otherwise, to a
In turn, personal liability for realty taxes may rest on the entity with taxable person, as provided in item (a) of the first paragraph
the beneficial use of the real property, such as the tax on property of Sec. 234.
owned by the government but leased to private persons or entities, or
Section 234 of LGC provides for the exemptions from payment of real with the threat that the airport will be sold at a public auction if the
property taxes and withdraws previous exemptions therefrom granted taxes remained unpaid. Hence, this action. MIAA claims to be exempt
to natural and juridical persons, including government owned and from tax under its Charter.
controlled corporations, except as provided therein.
The SC upheld the exemption of MIAA from real property taxes,
Pimentel: These exemptions are based on the ownership, character, and characterizing MIAA as a government instrumentality which is not a
use of the property. (You may skip this part). GOCC, despite its possession of corporate powers. Hence, the SC ruled
1) Ownership Exemptions. Exemptions from real property taxes that a government instrumentality, even though granted with corporate
on the basis of ownership are real properties owned by: (i) powers, is not necessarily a GOCC which is taxable under the LGC.
the Republic, (ii) a province, (iii) a city, (iv) a municipality,
(v) a barangay, and (vi) registered cooperatives. Doctrine: A government instrumentality is exempt from any kind of tax
2) Character Exemptions. Exempted from real property taxes on from the local governments. When the law vests in a government
the basis of their character are: (i) charitable institutions, (ii) instrumentality corporate powers, the instrumentality does not become
houses and temples of prayer like churches, parsonages or a corporation. Unless the instrumentality is organized as a corporation,
convents appurtenant thereto, mosques, and (iii) non profit it remains a government instrumentality exercising not only
or religious cemeteries. governmental but also corporate powers and is exempt from local
3) Usage exemptions. Exempted from real property taxes on the taxation.
basis of the actual, direct and exclusive use to which they are
devoted are: (i) all lands buildings and improvements which The term government "instrumentality" is broader than the term
are actually, directed and exclusively used for religious, "GOCC".
charitable or educational purpose; (ii) all machineries and
equipment actually, directly and exclusively used or by local Government-owned or controlled corporation refers to any agency
water districts or by government-owned or controlled organized as a stock or non-stock corporation, vested with functions
corporations engaged in the supply and distribution of water relating to public needs whether governmental or proprietary in
and/or generation and transmission of electric power; and nature, and owned by the Government directly or through its
(iii) all machinery and equipment used for pollution control instrumentalities either wholly, or, where applicable as in the case of
and environmental protection. stock corporations, to the extent of at least fifty-one (51) percent of its
4) Other Exemptions Withdrawn. All other exemptions capital stock. (Administrative Code)
previously granted to natural or juridical persons including
government-owned or controlled corporations are Class Notes: Why the difference in treatment?
withdrawn upon the effectivity of the Code. - MCIAA and MIAA have substantially the same charter. Now,
we have 2 entities substantially similar but treated
“Republic” : boarder and synonymous with "Government of the differently.
Republic of the Philippines" which the Admin Code defines as the
"corporate governmental entity though which the functions of the
government are exercised through at the Philippines, including, save as
the contrary appears from the context, the various arms through which NAPOCOR v. LANAO DEL SUR
political authority is made effective in the Philippines, whether November 19, 1996
pertaining to the autonomous reason, the provincial, city, municipal or
barangay subdivision or other forms of local government." Summary: NPC was assessed by Lanao del sur of real estate taxes for
NPC’s power plant complex for the period of June 14, 1984-Dec. 31,
"National Government": the entire machinery of the central 1989 allegedly because the tax exempt status of NPC was withdrawn by
government, as distinguished from the different forms of local PD 1931 and E.O. 93 and was never validly restored since the FIRB
Governments. It is composed of the three great departments the resolutions which purportedly restored it was not valid since it was an
executive, the legislative and the judicial. undue delegation of taxing power and was only signed by the Chairman
of the FIRB, not FIRB as a board.
“Agency" of the Government refers to "any of the various units of the
Government, including a department, bureau, office instrumentality, or Summary: Section 2 of PD 1931 empowered the President and/or the
GOCCs, or a local government or a distinct unit therein Minister of Finance to restore tax exemptions to deserving entities. In
order to reinstate the NPC’s tax exemptions, Hon. De Roda, Jr., in his
“Instrumentality" refers to "any agency of the National Government, not concurrent capacities as Acting Minister of Finance and as Acting
integrated within the department framework, vested with special Chairman of FIRB, signed FIRB Resolution No. 10-85 which was made
functions or jurisdiction by law, endowed with some if not all corporate effective as of June 11, 1984, the promulgation date of PD 1931, until
powers, administering special funds, and enjoying operational June 30, 1985. As to FIRB Resolution No. 1-86, Hon.
autonomy; usually through a charter. This term includes regulatory Virata fully restored the tax exemption as of July 1, 1985, to continue
agencies, chartered institutions and government-owned and controlled for an indefinite period. He also signed the same in his dual capacities
corporations". as Minister of Finance and as Chairman of the FIRB. The Albay case had
been superseded by Maceda vs. Macaraig, Jr. where the SC en banc
Class Notes: If Congress wants to tax an instrumentality, it can, expressly ruled that FIRB Resolution Nos. 10-85 and 1-86 are valid. The
SC however states that this ruling is solely with respect to the
purported realty tax liabilities of NPC for the period from June 14, 1984
to December 31, 1989. The SC shall not, in this Decision, rule upon the
MIAA v. CITY OF PASAY effect, if any of Republic Act No. 7160 upon NPC’s tax-exempt status.
April 2, 2009 The exemption claimed by NPC is partly based on PD 464 which, though
repealed by the Local Government Code in its paragraph (c), Section
Summary: MIAA operates the NAIA Complex in Pasay and Parañaque 534, Title Four of Book IV, was still good law during the period the
pursuant to its Charter (EO 903). Under its charter, MIAA is exempt exemption was being claimed in the instant case.
from tax. However, upon the effectivity of the LGC, the City of Pasay is of
the belief that the LGC withdrew the blanket tax exemption enjoyed by Class Notes: Good law? NO, exemption has been withdrawn.
certain entities and that it can now impose real property taxes on MIAA.
Hence, in 2002, it assessed real property tax delinquencies against
MIAA amounting to more than P1 billion and issued warrants of levy
LRTA v. CENTRAL BOARD OF ASSESSMENT APPEALS There are two substantial distinctions between cooperatives under PD
October 12, 2000 269 and cooperatives under 6938.
- As regards capital contribution by members. The definition of
Summary: LRTA assails the assessment of the Manila City Assessor for a cooperative was stated in RA 6938. The importance of
real property tax on its carriageways and terminals, arguing that they capital contributions by members of a cooperative under RA
are not taxable since (1) they are attached to government-owned 6938 was emphasized during Senate deliberations as one of
national roads, they form part of such roads, and (2) they are devoted the key factors which distinguished electric cooperatives
to public use. The Assessor, all the way to the SC, denied its claim. The under PD 269. PD 269 does not require cooperatives to make
SC ruled that: (1) the subject improvements are not public property and equitable contributions to capital. To qualify as a member of
do not form part of the public roads, (2) they are only accessible to the an electric cooperative in PD 269, only the payment of
paying public, not to the general public, and (3) even assuming that membership fee of 5 pesos is required. Under RA 6938, the
they are government-owned, under the RPT Code, properties whose articles of incorporation must be accompanied with the
beneficial use is assigned to taxable persons are taxable. In this case, EO bonds of accountable officers and sworn statement of the
603, LRTA’s charter, does not grant is exemption from real estate tax. treasurer showing that tat least 25% of authorized share
capital has been subscribed and at least 25% of the total
Summary: LRTA and MTO function as service-oriented business entities, subscription has been paid and the paid-up capital share
which provide valuable transportation facilities to the paying public. In shall not be less than 2000 pesos.
the absence of any express grant of exemption in their favor, they are
subject to the payment of real property taxes. - As regards the extent of government control. The principle
adhered to by RA 6938 is the principle of subsidiarity,
Also, under the Real Property Tax Code, real property is classified for wherein the government may only engage in development
assessment purposes on the basis of actual use, which is defined as “the activities where cooperatives do not possess the capability
purpose for which the property is principally or predominantly utilized nor the resources to do so and only upon the request of such
by the person in possession of the property.” cooperatives. In contrast, the extent of government control
over electric cooperatives under PD 269, is largely a function
of the role of the NEA.
PHILRECA v. DILG SECRETARY & DOF SECRETARY
June 10, 2003 The transitory provisions of R.A. No. 6938 are indicative of the
recognition by Congress of the fundamental distinctions between
Summary: In order to finance electrification projects, the Philippine electric cooperatives organized under P.D No. 269 and cooperatives
Government, acting through the National Economic Council and the under the new Cooperative Code. Article 128 of the Cooperative Code
National Electrification Administration (NEA), entered into 6 loan provides that all cooperatives registered under previous laws shall be
agreements with the US Government. The beneficiaries of these loan deemed registered with the CDA upon submission of certain
agreements were electric cooperatives who are members of petitioner requirements within one year. However, cooperatives created under
Philippine Rural Electric Cooperatives Association (PHILRECA). The P.D. No. 269, as amended, are given three years within which to qualify
loan agreements contain similarly worded provisions on the tax and register with the CDA, after which, provisions of P.D. No. 1645
application of the loan and any property or commodity acquired which expand the powers of the NEA over electric cooperatives, would
through the proceeds of the loan [See footnote 1]. With the passage of no longer apply.
the LGC, PHILRECA contends that their tax exemptions have been
invalidly withdrawn. PHILRECA filed a petition seeking to declare the
unconstitutionality of Sections 193 and 234 of the LGC, for violating the
equal protection clause and the non-impairment clause. The SC upheld PROVINCIAL ASSESSOR OF MARINDUQUE v. CA
the validity of the aforementioned Sections of the LGC. April 30, 2009

Doctrine: It is clear from the tenor of Sec. 234 of the LGC that the real Summary: Marcopper Mining Corporation received a Notice of
property owned by duly registered cooperatives under RA 6938 are Assessment from the Provincial Assessor of the Province of Marinduque
tax-exempt and that with respect to real property taxes, the LGC for real property taxes due on its real properties, including its Siltation
withdraws exemptions enjoyed by all other entities upon the LGC’s Dam and Decant System. Marcopper paid the taxes but appealed the
effectivity. Likewise, Sec. 193 of the LGC effectively withdraws assessment before the Local Board of Assessment Appeals on the
exemptions from local taxation enjoyed by various entities upon the ground that the subject property is exempt from real property taxation
effectivity of the LGC except for a) local water districts; b) cooperatives because the dam is intended primarily for the prevention of pollution
duly registered under RA 6938; and c) non-stock, non-profit hospitals (Sec. 234 (e) of RA 7160). The LBAA denied the appeal ruling that the
and educational institutions. the limited and restrictive nature of the property is taxable and cited Benguet Corporation v Central Board of
tax exemption privileges under the Local Government Code is Assessment Appeals where the Court ruled that a tailings dam is a
consistent with the State policy to ensure autonomy of local permanent improvement not exempt from real property taxation. Upon
governments and the objective of the Local Government Code to grant appeal to the CBAA, the ruling of the LBAA was affirmed alleging that
genuine and meaningful autonomy to enable local government units to the subject property was neither a machinery nor an equipment but a
attain their fullest development as self-reliant communities and make permanent improvement and therefore no tax exempt under Sec. 234
them effective partners in the attainment of national goals. The obvious (e) of RA 7160. The CA reversed the ruling of the CBAA and held that
intention of the law is to broaden the tax base of local government units the dam falls within the meaning of machinery. Upon appeal to the SC,
to assure them of substantial sources of revenue. the decision of the CA was reversed and the rulings of the CBAA and
LBAA were reinstated. The SC ruled that Marcopper was not able to
The Constitutional mandate that every local government unit shall sufficiently prove that the property was actually used to prevent
enjoy local autonomy does not mean that the exercise of power by local pollution. Likewise, the documents that Marcopper submitted did not
governments is beyond regulation by Congress. Thus, while each show that the dam falls within the meaning of machinery or equipment.
government unit is granted the power to create its own sources of
revenue, Congress, in light of its broad power to tax, has the discretion Doctrine: The burden is upon the taxpayer to prove, by clear and
to determine the extent of the taxing powers of local government units convincing evidence, that his claim for exemption has legal and factual
consistent with the policy of local autonomy. Section 193 of the Local basis.
Government Code is indicative of the legislative intent to vest broad
taxing powers upon local government units and to limit exemptions In order to be exempt under Sec. 234(e), RA 7160, it must be proved
from local taxation to entities specifically provided therein. that the property falls within the definition of machinery, which
embraces machines, equipment, mechanical contrivances, instruments, PHILIPPINE FISHERIES DEVELOPMENT AUTHORITY v. CA
appliances or apparatus which may or may not be attached, July 30, 2007
permanently or temporarily, to the real property.
Summary: Philippine Fisheries Authority was created by President
It includes the physical facilities for production, the installations and Marcos. It was given the administration of the Iloilo Fishing Port
appurtenant service facilities, those which are mobile, self-powered or Complex. In its administration it leased portion of the said port to
self-propelled and those not permanently attached to the real property private businesses. Later, the city of Iloilo assessed the Authority with
which are actually, directly, and exclusively used to meet the needs of real property taxes and in case the same fail to pay the taxes, IFPC will
the particular industry, business or activity and which by their very be sold at public auction to satisfy the claim of the local government.
nature and purpose are designed for, or necessary to its manufacturing, The SC ruled that, generally, government instrumentality are exempt
mining, logging, commercial, industrial or agricultural purposes. from real property taxes. However, the Authority falls under the
However, after the enactment of RA 7942, a pollution control device exception provided in Sec. 234 of the LGC. Hence, the Authority is liable
under RA 7160 is now defined to include “infrastructure” or to pay real property taxes with respect to those portions it leased with
“improvement” and not just machinery or equipment. private entities. The SC further ruled that despite holding the Authority
liable, IFPC cannot be levied without Congressional fiat, it being a “port”
The exemption granted under Sec. 23 (e) of RA 7160 is based on usage. and is a public domain.
The term usage means direct, immediate and actual application of the
property itself to the exempting purpose. Sec. 199 of RA 7160 defines Doctrine: Instrumentalities of the national government are exempt from
actual use as the purpose for which the property is principally or local taxes pursuant to Section 133(o) of the Local Government Code.
predominantly utilized by the person in possession thereof. It refers to This exemption, however, admits of an exception with respect to real
concrete, as distinguished from mere potential, use. A claim for tax property taxes. Applying Section 234(a) of the Local Government Code,
exemption under Sec. 234 (e) must be supported by evidence that the the Court ruled that when an instrumentality of the national
property sought to be exempt is actually, directly, and exclusively used government grants to a taxable person the beneficial use of a real
for pollution control and environmental protection. property owned by the Republic, said instrumentality becomes liable to
pay real property tax. (but only with respect to those portions leased to
private parties)
LUNG CENTER OF THE PHILIPPINES v. QUEZON CITY
June 29, 2004
GSIS v. CITY TREASURER
Summary: The Lung Center of the Philippines is a non-stock, non-profit December 23, 2009
entity established by virtue of PD 1823. It is the registered owner of a
parcel of land in Quezon Avenue. Erected in the middle of the lot is the Summary: GSIS owned 2 properties in Manila, one of which is leased to
hospital, Lung Center. A big space at the ground floor of the building is a private entity. The City of Manila assessed both properties real
being leased to private parties for canteen and small store spaces and to property tax for the period 1992-2002 and threatened to seize and sell
Medical or professional practitioners for their private clinics and they both if the deficiency is not paid. RTC: Assessment valid.
charge their patients for their professional services. Almost half of the
entire area on the left side of the building is vacant and idle. A big SC: GSIS is not liable. PD 1146 which established the GSIS exempted it
portion on the right of the building is being leased for commercial from all taxes. RA 7160 removed this tax exemption. However, RA 8291
purposes to a private enterprise. which took effect in 1997 restored this full tax exemption, with all
assessments as of the passage of said law considered as paid. As to the
The land and the hospital building of Lung Center were assessed for property leased to a private entity, realty tax may be assessed but it is
realty taxes by Quezon City. Lung Center invokes its exemption from the private entity which is liable, not GSIS.
real property taxes. The Supreme Court held that only for those real
properties actually, directly, and exclusively used for charitable Doctrine: GSIS is exempt from all taxes, assessments, fees, charges or
purposes. Those areas used and leased to private entities are taxable. duties of all kinds. GSIS was exempted under its charter (PD 1146). This
exemption was withdrawn by LGC 1991 but was returned by virtue of
Doctrine: (When is one a charitable institution) The test whether an RA 8291.
enterprise is charitable or not is whether it exists to carry out a purpose
reorganized in law as charitable or whether it is maintained for gain, A repeal of this exemption in RA 8291 by a subsequent law would not
profit, or private advantage. As a general principle, a charitable suffice to affect the full exemption benefits granted the GSIS, unless the
institution does not lose its character as such and its exemption from following are met: (1) The repealing clause must expressly, specifically,
taxes simply because it derives income from paying patients, whether and categorically revoke or repeal Sec. 39; and (2) a provision is
out-patient, or confined in the hospital, or receives subsidies from the enacted to substitute or replace the exemption referred to herein as an
government so long as the money received is devoted or used essential factor to maintain or protect the solvency of the fund.
altogether to the charitable object which it is intended to achieve.
Beneficial Use Doctrine Applicable
(On exemption from real property taxes) Thus, to be exempted from real The foregoing notwithstanding, the leased Katigbak property shall be
property taxes under the 1987 Constitution and the LGC, claimant must taxable pursuant to the "beneficial use" principle under Sec. 234(a) of
prove: (1) That it is a charitable institution, and (2) That its real the LGC: SEC. 234. Exemptions from Real Property Tax. – The following
properties are actually, directly, and exclusively used for charitable are exempted from payment of the real property tax: (a) Real property
purposes. Exclusively means synonymous with solely. It cannot be owned by the Republic of the Philippines or any of its political
substituted for “dominant use” or “principal use.” What is meant by subdivisions except when the beneficial use thereof has been granted,
actual, direct, and exclusive use of the property for charitable purposes for consideration or otherwise, to a taxable person.
is the direct and immediate and actual application of the property itself
for charity and not the use of the income. This exemption, must be read in relation with Sec. 133(o) LGC:
SEC. 133. Common Limitations on the Taxing Powers of Local
THUS, portions of the land leased to private entities as well as those Government Units. – Unless otherwise provided herein, the exercise of
parts of the hospital leased to private individuals are not exempt from the taxing powers of provinces, cities, municipalities, and barangays
such taxes. Those portions of the land occupied by the hospital and shall not extend to the levy of the following:
portions of the hospital used for its patients, paying or non-paying, are xxxx
exempt from real property taxes. (o) Taxes, fees or charges of any kinds on the National Government, its
agencies and instrumentalities, and local government units.
Read together, the provisions allow the Republic to grant the beneficial Doctrine: A GOCC must be organized as a stock or non-stock
use of its property to an agency or instrumentality of the national corporation; and instrumentality need not be organized as a
government. Such grant does not necessarily result in the loss of the tax corporation but it is vested with some or all corporate powers. To be a
exemption. The tax exemption that the property of the Republic or its GOCC, an agency or instrumentality must be a a stock or non-stock
instrumentalities carry ceases only if, as stated in Sec. 234(a) of the LGC corporation.
1991, "beneficial use thereof has been granted, for a consideration or
otherwise, to a taxable person." GSIS, as a government instrumentality, Based on Sec. 16, Art. XII, of the 1987 Constitution, two conditions must
is not a taxable juridical person under Sec. 133(o) LGC. GSIS, however, be met for Congress to create a GOCC: 1. The GOCC must be established
lost that status as to the Katigbak property when it contracted its for the common good, and 2. The GOCC must meet the test of economic
beneficial use to MHC, a taxable person. viability.

The test of economic viability does not apply to government entities


NATIONAL HOUSING AUTHORITY v. ILOILO vested with corporate powers and performing essential public services.
August 20, 2008 Instrumentalities perform essential public services for the common
good and need not be economically viable since the government may
Summary: Iloilo auctioned off a lot owned by NHA because NHA was not even subsidize their entire operations.
paying realty taxes. NGA filed a complaint to annul the auction sale.
Iloilo filed a MTD contending that s267 of LGC dictates that a court only Under Sec. 234 of the LGC, real property owned by the Republic is
acquires jurisdiction if the complainant pays a deposit equivalent to the exempt from real property tax unless the beneficial use thereof has
amount of real property that was sold in the auction. NHA answered been granted, for consideration or otherwise, to a taxable person.
that it was not liable under s267 because the provision requires a
deposit only from taxpayers and by virtue of certain statutes, it was a Reclaimed lands are reserved land for public use. They are properties of
tax-exempt entity. RTC granted the motion to dismiss. CA affirmed. SC public dominion. The ownership of such lands remain with the State
reversed, sided with NHA. unless they are withdrawn by law or presidential proclamation for
public use. Properties of public dominion as not subject to execution or
Doctrine: The deposit precondition is an ingenious legal device to foreclosure sale.
guarantee the satisfaction of the tax delinquency, with the local
government unit keeping the payment on the bid price no matter the
final outcome of the suit to nullify the tax sale. Thus, the requirement is
not applicable if the plaintiff is the government or any of its agencies as
it is presumed to be solvent, and more so where the tax exempt status
of such plaintiff as basis of the suit is acknowledged. In this case, NHA is
indisputably a tax-exempt entity whose exemption covers real property
taxes and so its property should not even be subjected to any
delinquency sale. Perforce, the bond mandated in Section 267, whose
purpose it is to ensure the collection of the tax delinquency should not
be required of NHA before it can bring suit assailing the validity of the
auction sale.

REPUBLIC v. PHILIPPINE RECLAMATION AUTHORITY


July 18, 2012

Summary: The Philippine Reclaramation Authority (PRA) reclaimed


lands in Paranaque. The Paranaque City Treasurer levied the lands
because PRA was delinquent in paying for the real property taxes on
them. PRA filed a TRO but this was denied and so Paranaque proceeded
to auction off the lands. Certificates of Sale were issued pursuant to the
auction sale.

PRA filed this petition for certiorari, arguing that it is exempt from
paying real property taxes because it is an instrumentality of the
national government and thus exempt under Sec. 234(A) and 133(O) of
the LGC. PRA also prayed that the certificates of sale be cancelled
because the lands sold are government property outside of public
domain. Paranaque argued that PRA is not an instrumentality of the
government but a GOCC based on Sec. 3 of its charter, PD 1084. It said
that it is a stock corporation with capital stock divided into no par value
shares, and that is exemption from taxes granted under its charter was
withdrawn by Sec. 193 of the LGC.

The RTC sided with Paranaque. The SC reversed and ruled that the PRA
is an instrumentality of the government, defined in Sec. 2(1) of the
Introductory Provisions of the Administrative Code. PRA is not a GOCC
as it is neither a stock or non-stock corporation, and it fails to pass the
“test of economic viability” under Sec. 16, Art. XII of the 1987
Constitution. The Court cited MIAA v. CA that said that the basic
principle behind Sec. 133(O) of the LGC is that local government cannot
tax the national government. The SC also declared the certificates of
sale as void because the lands levied are of public dominion.
VIOLATIONS OF TAX ORDINANCES
Penalties for Violation of Tax Ordinances (Sec. 516)
- The sanggunian is authorized to prescribe fines or other
penalties for violation of tax ordinances
- LIMITATIONS:
1. Fine shall not be less than P1,000 nor more than P5,000
2. Imprisonment shall not be less than 1 month nor more
than 6 months
- For Sangguniang Barangay, may prescribe:
a. Fine of not less than P100 nor more than P1,000
- The penalties shall be imposed at the discretion of the court

Omission of Property from Assessment or Tax Rolls by Officers and


Other Acts (Sec. 517)
A. Assessor’s omissions penalized
1. Certain omissions by assessors or officers charged with
assessing real property are punished:
a. Willful failure to assess any real property
b. Intentional omission of any real property from
inclusion in the assessment or tax roll
c. Willful or negligent underassessment
d. Intentional violation or failure to perform any
duty imposed upon him relating to assessment of a
taxable real property
2. Penalties:
a. Fines not less than P1,000 nor more than P5,000
b. Imprisonment of not less than 1 month nor more
than 6 months
c. Or both, at discretion of court

B. Tax Collectors
1. Realty tax collectors who willfully or negligently fail to
collect the tax due on real property and institute
necessary proceedings to collect the tax shall face the
same penalties as mentioned above

C. Other Officers
1. Officers who willfully fail to discharge their duties
relative to the administration of real property tax
provisions of the LGC or to assist the assessor or
treasurer in such administration are punishable
2. Penalties
a. Same penalties
b. Except that insofar as fines are concerned, the
minimum imposable fine is P500

Government Agents Delaying Assessment of Real Property and


Assessment Appeals (Sec. 518)
1. Acts punishable: Government officials who intentionally and
deliberately delay:
a. Assessment of real property or
b. Filing of an appeal against its assessment
2. Penalties:
a. Fine of not less than P500 and not more than P5,000
b. Imprisonment of not less than 1 month but not more
than 6 months
c. Or both, at discretion of the court

Failure to Dispose Delinquent Real Property at Public Auction


1. Penalized under Sec. 519 are willfully or negligently:
a. Delaying the disposal of delinquent real property by the
treasurer, or
b. Obstructing the prompt disposal of delinquent real
property by any local official or employee
2. Penalties:
a. Fine ranging from P1,000 to P5,000
b. Imprisonment not less than 1 month but not more than
6 months
c. Or both, at discretion of court
SHARES OF LGUs IN THE PROCEEDS OF NATIONAL TAXES
ALVAREZ v. GUINGONA
INTERNAL REVENUE ALLOTMENT January 31, 1996

Summary: RA 7720 was passed converting the municipality of Santiago


IRA (Internal Revenue Allotment - The share of the LGU in the national into an independent component city. The petitioners are assailing the
internal revenue taxes (Sec. 284) constitutionality of the RA on the ground that the income of Santiago,
- Share of LGUs: after deducting the IRA, does not meet the 20M minimum income
1. 30% for 1992 requirement for conversion. The SC ruled that the IRA should be
2. 35% for 1993 included in the computation of the income.
3. 40% for 1994 and onwards
- Basis of tax sharing – the tax collection of the third fiscal year Doctrine: The IRAs are items of income because they form part of the
preceding the current fiscal year gross accretion of the funds of the local government unit. The IRAs
 E.g. For fiscal year 1993, the IRA is based on the regularly and automatically accrue to the local treasury without need of
national internal revenue tax collections of 1990 any further action on the part of the local government unit. They thus
constitute income which the local government can invariably rely upon
Reduction of LGU Share as the source of much needed funds.
1. WHO may reduce? The President
2. WHAT ground? In the event of an unmanageable public IRAs are a regular, recurring item of income; nil is there a basis, too, to
sector deficit on the part of the national government classify the same as a special fund or transfer, since IRAs have a
3. REQUISITES: technical definition and meaning all its own as used in the Local
a. It be recommended by the Secretaries of Finance, Government Code that unequivocally makes it distinct from special
Interior and Local Government, and Budget funds or transfers referred to when the Code speaks of “funding
b. President consult with the Senate President, Speaker of support from the national government, its instrumentalities and
the house, presidents of the Leagues (Province, City, government-owned-or-controlled corporations.”
Municipality, Barangay, SK)
c. Reduction must not go below 30% of the internal Class Notes:
revenue collections of the 3rd fiscal year preceding the - Congress cannot provide for less than 40% as IRA in its GAA. A
current fiscal year GAA is not a proper amendatory law of the LGC
- Note also that it is not the appropriation that is automatic!
Criteria for Sharing LGU Allotments (Sec. 285)  It is the release
- Share of LGUs in the IRA shall be allocated as follows:
1. 23% for Provinces
2. 23% for Cities PIMENTEL v. AGUIRRE
3. 34% for Municipalities July 19, 2000
4. 20% for Barangays
Summary: The president enacted AO 372 where sec. 1 requires all
- Each share of each province, city, and municipality shall be government units to reduce their expenditure by at least 25% , while
determined on the basis of the following formuala: sec. 4 withholds 10% of the LGUs IRA pending evaluation of the
1. Population – 50% emerging fiscal situation. The petitioner assails the constitutionality of
2. Land Area – 25% the said provision as being violative of the fiscal autonomy of the LGUs.
3. Equal Sharing – 25% The SC ruled that sec. 1 is valid because it is merely advisory and not
mandatory, while Sec. 4 is invalid for it encroaches upon the fiscal
- As for the share of each barangay: autonomy of the LGUs.
 Barangays with at least 100 inhabitants, its share shall
not be less than P80,000 per annum Doctrine: A basic feature of local fiscal autonomy is
 Balance is allocated on the basis of the following the automatic release of the shares of LGUs in the national internal
formula (on the 3rd year of LGC effectivity and revenue. This is mandated by no less than the Constitution. The Local
thereafter): Government Code specifies further that the release shall be made
a. Population – 60% directly to the LGU concerned within five (5) days after every quarter of
b. Equal Sharing – 40% the year and "shall not be subject to any lien or holdback that may be
imposed by the national government for whatever purpose." As a rule, the
Automatic Release of Shares (Sec. 286) term "shall" is a word of command that must be given a compulsory
- Releases of shares of the LGU of the internal revenue taxes are meaning. The provision is, therefore, imperative.
required to be done AUTOMATICALLY
 When? Within the first 5 days of every quarter
 The shares shall NOT be subject to any lien and may THE PROVINCE OF BATANGAS v. ROMULO
NOT be withheld for ANY reason May 27, 2004
- Automatic release shall not be subject to any conditions
Summary: Herminaldo Mandanas, Governor of Batangas, petitioned to
Local Development Projects (Sec. 287) declare unconstitutional and void certain provisos contained in the
- At least 20% of its annual IRA must be appropriated by an General Appropriations Acts (GAAs) of 1999, 2000, and 2001, insofar as
LGU for development projects they uniformly earmarked for each corresponding year the amount of
P5billion for the Internal Revenue Allotment (IRA) for the Local
Government Service Equalization Fund (LGSEF) & imposed conditions
for the release thereof.

Doctrine: Pursuant to the precept of Local Autonomy, the Constitution


and the LGC both mandate the automatic release of the LGUs’ share in
the national revenue taxes. Therefore, such releases cannot be subject
to conditions.
The only possible exception to the mandatory automatic release of the
LGUs’ IRA is if the national internal revenue collections for the current
fiscal year is less than 40 percent of the collections of the preceding
third fiscal year, in which case what should be automatically released
shall be a proportionate amount of the collections for the current fiscal
year. The adjustment may even be made on a quarterly basis
depending on the actual collections of national internal revenue taxes
for the quarter of the current fiscal year.

ACORD v. ZAMORA
June 8, 2005

Summary: The General Appropriation Act for the year 2000 set apart a
portion of the IRA as “unprogrammed funds.” It will only be released
only when the original revenue targets submitted by the President to
Congress can be realized based on a quarterly assessment to be
conducted by certain committees. Petitioners assailed the above
provision for being contrary to Article X, Section 6 of the Constitution
(for subjecting the release of the IRA to a condition vs. automatic
release of the IRA as mandated by the mentioned Constitutional
provision.) The Court invalidated the provision.

Doctrine: Article X, Section 6 of the Constitution – the same provision


relied upon in Pimentel – enjoins both the legislative and executive
branches of government.

While the Constitution gave the executive branch the duty to


automatically release the just share of local governments in the national
taxes, it also enjoins the legislature not to pass laws that might prevent
the executive from performing this duty. The executive cannot
disregard Constitutional provisions just because it has the backing of a
statute. This is tantamount to saying that the Constitution can be
amended by a mere statute.
Special Law cited in the Syllabus: RA 7076 (An Act Creating a People’s
SHARE OF LGUs IN NATIONAL WEALTH Small-Scale Mining Program and for other purposes)
- Sec. 19 - The revenue to be derived by the Government from
Share In the Proceeds from the Development and Utilization of National the operation of the mining program herein established shall
Wealth (Sec. 289) be subject to the sharing provided in the Local Government
- LGUs shall have an equitable share in the proceeds derived Code.
from the utilization and development of national wealth
within their respective areas
 “National Wealth” means the natural resources of the
nation (land, water, forests, fishes, minerals, etc.) which CREDIT FINANCING
are being utilized and developed anywhere in the
country Under Title 4 of Book II of the LGC, LGUs are granted the power to
- LGUs may share the proceeds thereof with the inhabitants in create indebtedness from government financing institutions or
the area by way of direct benefits domestic private banks to fund local infrastructure or other
socioeconomic development projects and to stabilize local finances
Share of LGUs in the National Wealth (Sec. 290)
- Internal revenue collections from the national wealth through General Policy (Sec. 296)
mining, forestry, and fishery activities in the form of taxes, 1. Any LGU may create indebtedness and avail of credit
charges, interests, fines, or shares of the central government facilities to finance local infrastructure and other
in any co-production, joint venture or production sharing socioeconomic development projects in accordance with the
agreements over such activities are to be shared with the approved local development plan and public investment
LGUs having jurisdiction over the areas devoted to the program
utilization and development of national wealth 2. Any LGU may avail of credit lines from government or
- NOTE: This is on TOP of the IRA private banks and lending institutions for the purpose of
stabilizing local finances
- Shares of LGUs:
1. 40% of the gross collections derived by the national Browse through Sec. 295 – 303. They are technical provisions but just
government from utilization and development of the remember that LGUs are expressly given the power to incur indebtedness
national wealth by private persons or entities for the purposes mentioned in Sec. 296.
2. If the person or entity is a public agency, Sec. 291
applies (whichever will produce a higher share for the
LGU)
a. 1% gross sales or receipts of the preceding LOCAL FISCAL ADMINISTRATION
calendar year, or
b. 40% of mining taxes, royalties, forestry, and Browse through Sec. 304 – 354, but take note of Sec. 304 on the
fishery charges and such other taxes, fees, charges Fundamental Principles
the government agency or GOCC would have paid
if it were not exempt Title 5, Book II on Local Fiscal Administration governs the conduct and
management of financial affairs, transactions, and operations of
Allocation of Shares (Sec. 292) - Among themselves, the share of the provinces, cities, municipalities, and barangays.
LGUs are distributed as follows:
A. Where natural resources utilized or developed are located in Fundamental Principles on the Financial Affairs of an LGU (Sec. 305)
a province: (a) No money shall be paid out of the local treasury except in
1. 20% goes to the province pursuance of an appropriations ordinance or law;
2. 45% to the component city or municipality
3. 35% to the barangay (b) Local government funds and monies shall be spent solely for
public purposes;
B. Where natural resources are located in ICC or HUC
1. 65% to the city (c) Local revenue is generated only from sources expressly
2. 35% to the barangay authorized by law or ordinance, and collection thereof shall
at all times be acknowledged properly;
C. If natural resources are located in 2 or more provinces,
component cities, municipalities, or barangays, shares (d) All monies officially received by a local government officer in
determined by the following criteria: any capacity or on any occasion shall be accounted for as
1. 70% population local funds, unless otherwise provided by law;
2. 30% land area
(e) Trust funds in the local treasury shall not be paid out except
D. If natural resources are located in 2 or more cities: in fulfillment of the purpose for which the trust was created
1. 70% population or the funds received;
2. 30% land area
(f) Every officer of the local government unit whose duties
LGU Shares remitted directly (Sec. 293) – Within 5 days from the end of permit or require the possession or custody of local funds
every quarter, government agencies or GOCCs concerned are required shall be properly bonded, and such officer shall be
to remit the share of LGUs to the local treasurer accountable and responsible for said funds and for the
safekeeping thereof in conformity with the provisions of law;
LGU Shares for Development, Livelihood and Energy Projects (Sec. 294)
- Share of LGUs in the proceeds from utilization and (g) Local governments shall formulate sound financial plans, and
development of national wealth are intended for: local budgets shall be based on functions, activities, and
1. Local development projects projects, in terms of expected results;
2. Local livelihood projects
(h) Local budget plans and goals shall, as far as practicable, be
harmonized with national development plans, goals, and
strategies in order to optimize the utilization of resources of their salaries is what sets the employees and officials of
and to avoid duplication in the use of fiscal and physical the Sangguniang Panlalawigan apart from the other employees and
resources; officials of the province. Accordingly, the appointing power of the Vice-
Governor is limited to those employees of the Sangguniang
(i) Local budgets shall operationalize approved local Panlalawigan, as well as those of the Office of the Vice-Governor, whose
development plans; salaries are paid out of the funds appropriated for the Sangguniang
Panlalawigan. As a corollary, if the salary of an employee or official is
(j) Local government units shall ensure that their respective charged against the provincial funds, even if this employee reports to
budgets incorporate the requirements of their component the Vice-Governor or is assigned to his office, the Governor retains the
units and provide for equitable allocation of resources authority to appoint the said employee pursuant to Section 465(b)(v) of
among these component units; Rep. Act No. 7160.

(k) National planning shall be based on local planning to ensure


that the needs and aspirations of the people as articulated by QUISUMBING v. GARCIA
the local government units in their respective local December 8, 2008
development plans are considered in the formulation of
budgets of national line agencies or offices; Summary: COA conducted a financial audit on the Province of Cebu and
found out that several contracts were not supported with a
(l) Fiscal responsibility shall be shared by all those exercising Sangguniang Panlalawigan resolution authorizing the Provincial
authority over the financial affairs, transactions, and Governor to enter into a contract, as required under Section 22 of R.A.
operations of the local government units; and No. 7160. Gov Garcia sought reconsideration of this findings. However,
without waiting for its resolution, filed an action for declaratory relief
(m) The local government unit shall endeavor to have a balanced with the RTC. Gov claimed that the no prior authorization is required
budget in each fiscal year of operation. because the expenditures incurred are already authorized by the
appropriation ordinances of the previous year which are deemed re-
enacted. RTC ruled that no prior authorization is required. SC ruled that
the case must be remanded because the action for declaratory relief is
Special Law cited in the Syllabus – RA 8185 (An Act Amending Sec. not proper and must be dismissed due to the breach committed prior to
324(d) of LGC) the filing of the action. The trial court should have determined the
character of the questioned contracts, i.e., whether they were mere
Five percent (5%) of the estimated revenue from regular sources shall disbursements pursuant to the ordinances supposedly passed by the
be set aside as annual lump sum appropriations for relief, sanggunian, or new contracts which requires authorization of the
rehabilitation, reconstruction and other works or services in provincial board.
connection with calamities which may occur during the budget year.
Doctrine: Disbursement, as used in Sec. 346, should be understood to
Provided, however, That such fund shall be used only in the area, or a pertain to payments for statutory and contractual obligations which the
portion thereof, of the local government unit or other areas affected by sanggunian has already authorized thru ordinances enacting the annual
a disaster or calamity, as determined and declared by the local budget and are therefore already subsisting obligations of the local
sanggunian concerned. government unit.

"Calamity shall be defined as a state of extreme distress or misfortune, Contracts, as used in Sec. 22(c) on the other hand, are those which bind
produced by some adverse circumstance or event or any great the local government unit to new obligations, with their corresponding
misfortune or cause or loss or misery caused by natural forces. terms and conditions, for which the local chief executive needs prior
authority from the sanggunian
"In case of fire or conflagration, the calamity fund shall be utilized only
for relief operations. The question of whether a sanggunian authorization separate from the
appropriation ordinance is required should be resolved depending on
"The local development council shall more monitor the use and the particular circumstances of the case. Resort to the appropriation
disbursement of the calamity fund." ordinance is necessary in order to determine if there is a provision
therein which specifically covers the expense to be incurred or the
contract to be entered into.
ATIENZA v. VILLAROSA
May 10, 2005 Should the appropriation ordinance, for instance, already contain in
sufficient detail the project and cost of a capital outlay such that all that
Summary: The issue started when the Governor invalidated the signing the local chief executive needs to do after undergoing the requisite
authority of the Vice Governor over the affairs of the Sangguniang public bidding is to execute the contract, no further authorization is
Panlalawigan. The Court rules that due to decentralization the powers required, the appropriation ordinance already being sufficient. On the
reside with the Vice Governor and his signing authority is evidenced by other hand, should the appropriation ordinance describe the projects in
Section 466 of the Local Government Code. generic terms such as "infrastructure projects," "inter-municipal
waterworks, drainage and sewerage, flood control, and irrigation
Doctrine: The Governor has the authority to appoint officials and systems projects," "reclamation projects" or "roads and bridges," there
employees whose salaries are paid out of the provincial funds, this does is an obvious need for a covering contract for every specific project that
not extend to the officials and employees of the Sangguniang in turn requires approval by the sanggunian.
Panlalawigan because such authority is lodged with the Vice-
Governor. In the same manner, the authority to appoint casual and job Specific sanggunian approval may also be required for the purchase of
order employees of the Sangguniang Panlalawigan belongs to the Vice- goods and services which are neither specified in the appropriation
Governor. ordinance nor encompassed within the regular personal services and
maintenance operating expenses.
The authority of the Vice-Governor to appoint the officials and
employees of the Sangguniang Panlalawigan is anchored on the fact
that the salaries of these employees are derived from the appropriation
specifically for the said local legislative body. Indeed, the budget source
SISON v. PEOPLE
March 9, 2010 PROPERTY AND SUPPLY MANAGEMENT
Summary: Rolando E. Sison was the municipal mayor of Calintaan, IN THE LGUs
Occidental Mindoro. A post-audit investigation revealed that during his
incumbency, no public bidding was conducted for the purchase of a See Sec. 355-383. These provisions govern the procurement, care,
Toyota Land Cruiser, 119 bags of Fortune cement, an electric generator utilization, custody and disposal of supplies by LGUs and other aspects of
set, certain construction materials, two Desert Dueler tires, and a supply management at the local levels.
computer and its accessories. Sison was found guilty of violation of
Section 3(e) of RA 3019. Sison appealed to the SC, alleging for and his
guilt not proven beyond reasonable doubt. SC ruled that he failed to GSIS v. PROVINCE OF TARLAC
follow requirements of the LGC on government acquisitions, and December 1, 2003
affirmed his conviction.
Summary: While Gov. Cojuangco was in office, Tarlac Province donated
Doctrine: RA 7160 explicitly provides that, as a rule, "acquisitions of a lot to GSIS, for the latter to build an office building. A deed of donation
supplies by local government units shall be through competitive and MOA were executed therefor. When Gov. Yap was elected, he
bidding." The only exceptions are: sought to re-evaluate the donation and the MOA, under the belief that
(1) personal canvass of responsible merchants; the donation was unfair to Tarlac Province. The Provincial
(2) emergency purchase; Administrator demanded from GSIS the value of the lot donated. GSIS
(3) negotiated purchase; refused to pay. Hence, Tarlac Province filed a complaint for declaration
(4) direct purchase from manufacturers or exclusive distributors of nullity of the donation and the MOA before the RTC. RTC found the
(5) purchase from other government entities. donation valid, and dismissed the complaint. CA reversed, finding that
there was no prior valuation of the property. SC reinstated RTC, ruling
For personal canvass,: Sec. 367. “Procurement through Personal that such valuation is not a condition sine qua non for the validity of a
Canvass.—Upon approval by the Committee on Awards, procurement of donation from an LGU to a government instrumentality.
supplies may be affected after personal canvass of at least three (3)
responsible suppliers in the locality by a committee of three (3) Doctrine: There is nothing in the LGC which expressly states that the
composed of the local general services officer or the municipal or lack of an appraised valuation renders the transfer void. An appraised
barangay treasurer, as the case may be, the local accountant, and the valuation is merely a formal and procedural requisite. The lack of such
head of office or department for whose use the supplies are being valuation cannot overturn substantive and vested rights.
procured. The award shall be decided by the Committee on Awards. A transfer of property by an LGU to an instrumentality of government
Purchases under this Section shall not exceed the amounts specified without first securing an appraised valuation from the local committee
hereunder for all items in any one (1) month for each local government on awards is NOT one of the void contracts enumerated in Art. 1409.
unit: Municipalities: First Class— (P150,000.00), Second and Third Neither does Sec. 381, LGC expressly prohibit or declare void such
Class— (P40,000.00), Fourth Class and Below— (P20,000.00)” transfers.

Sec. 364. “The Committee on Awards.—There shall be in every


province, city or municipality a Committee on Awards to decide the
winning bids and questions of awards on procurement and disposal of
property.

The Committee on Awards shall be composed of the local chief


executive as chairman, the local treasurer, the local accountant, the
local budget officer, the local general services officer, and the head of
office or department for whose use the supplies are being procured, as
members. In case a head of office or department would sit in a dual
capacity a member of the sanggunian elected from among its members
shall sit as a member. The Committee on Awards at the barangay level
shall be the sangguniang barangay. No national official shall sit as
member of the Committee on Awards.”

The requirement that the canvass and awarding of supplies be made by


a collegial body assures the general public that despotic, irregular or
unlawful transactions do not occur. It also guarantees that no personal
preference is given to any supplier and that the government is given the
best possible price for its procurements.
THE LOCAL GOVERNMENT UNITS
4. Barangay Secretary
THE BARANGAY  Mandatory appointive official (Sec. 394)
 Appointed by the Punong Barangay subject to
See Sec. 384-439 Sanggunian concurrence
Special Laws cited in the Syllabus
5. Barangay Treasurer
1. RA 8441 (An Act Increasing the Cash Gift to P5,000;
 Also a mandatory appointive official (Sec. 395)
amending RA 6686)
a. Sec. 4 – All officials and employees of LGUs may receive
6. Other officials
the same benefits of this act
a. Lupong Tagapamayapa
(1) Chargeable against their local funds
b. Community Brigades (e.g. Tanod Brigade and Disaster
2. RA 9178 (An Act to promote the establishment of Barangay
Brigade)
Micro Business Enterprises) – See compilation
3. RA 9285 (An Act to Institutionalize the use of Alternative
The Barangay Assembly (Sec. 397 and 398)
Dispute Resolutions System in the Phhilippines)
- Composed of actual residents of a barangay for at least 6
a. Sec. 53 – Applicability of the Kataranguang
months who are citizens of the Republic, at least 15 years of
Pambarangay; RA 9285 does NOT repeal, amend, or
age and are registered in the list of barangay assembly
modify the jurisdiction of the Katarungang
members
Pambarangay under the LGC
- Functions:
4. Anti-VAWC10: The Barangay Protection Order issued by
1. Initiate legislative process; recommend to Sanggunian
the Punong Barangay or, in his unavailability, by any
the adoption of measures for the welfare of the
available Barangay Kagawad, merely orders the perpetrator
barangay and the city/municipality concerned
to desist from (a) causing physical harm to the woman or her
2. Decide on adoption of initiative as a legal process
child; and (2) threatening to cause the woman or her child
3. Hear and pass upon the semestral report of the
physical harm.
Sanggunian concerning its activities and finances
a. This function is purely executive in nature pursuant to
his or her duty under the Local Government Code to
“enforce all laws and ordinances,” and to “maintain
public order in the barangay.”
b. Citing jurisprudence, the Court explained that the mere PEOPLE v. RECTO
fact that an officer is required by law to inquire into the October 27, 2001
existence of certain facts and to apply the law thereto in
order to determine what his official conduct shall be Summary: RTC found Julio Recto guilty beyond reasonable doubt of (1)
and the fact that these acts may affect private rights do two counts of the complex crime of qualified direct assault with
not constitute an exercise of judicial powers.”11 frustrated homicide (Melchor Recto and Brgy Captain Orbe), (2) the
complex crime of qualified direct assault with murder (Macalipay), and
(3) homicide (Emiliano Santos). SC modified the conviction as to
Barangay – The barangay discharges 3 seminal functions:
Melchor Recto stating that there was no direct assault because he was
1. As a basic political unit
not engaged in the performance of duty.
2. As a primary planning and implementing unit
3. As a forum
Doctrine: Direct assault, a crime against public order, may be committed
Chief Officials and Offices (Sec. 387) in two ways:
1. Punong Barangay 1.) by any person or persons who, without a public uprising,
shall employ force or intimidation for the attainment of any
 Functions are enumerated in Sec. 389
of the purposes enumerated in defining the crimes of
 Appointment of barangay officials by the punong
barangay is subject to confirmation by the majority of rebellion and seditiontantamount to rebellion/sedition
all members of the Sangguniang Barangay 2.) by any person or persons who, without a public uprising,
 Among his more important powers are: shall attack, employ force, or seriously intimidate or resist
(a) Enforcement of laws relative to pollution control any person in authority or any of his agents, while engaged in
and protection of the environment the performance of official duties, or on occasion of such
(b) Administration of barangay justice or the performance more common form of assault
Katarungang Pambarangay
(c) Holding of annual palarong barangay 2nd is aggravated when (a) the assault is committed with a weapon, or
(b) when the offender is a public officer or employee, or (c) when the
2. Sangguniang Barangay – 7 members offender lays a hand upon a person in authority
 Powers and functions are outlined in Sec. 391 and 392
An agent of a person in authority is “any person who, by direct
3. Sangguniang Kabataan chairman provision of law or by election or by appointment by competent
authority, is charged with the maintenance of public order and the
protection and security of life and property, such as barrio councilman,
barrio policeman and barangay leader, and any person who comes to
the aid of persons in authority. In this case, Melchor Recto, barangay
tanod of Ambulong, Magdiwang, Romblon was clearly an agent of a
person in authority. However, he was not engaged in the performance
10 of his official duties” at the time he was shot. Neither was he attacked
Sir sent this via email. “on the occasion of such performance. Melchor Recto was a barangay
11
Garcia v. Drilon, G.R. No. 179264, June 25, 2013. See also Tua v. chief tanod but at the crime scene he was a mere bystander.
Mangrobang, G.R. No. 170701, January 22, 2014.
DAVID v. COMELEC THE KATARUNGANG PAMBARANGAY
April 8, 1997
See Sec. 399-422, and 515. The LGC is now the governing law on
Summary: Barangay chairman David is contending that his term of Katarungang Pambarangay. PD 1508 was expressly repealed
office is 5 years, under Sec. 2 of RA 6653, as reiterated in RA 6679. And
so he sought prohibition to stop the elections on his third year of office. Special Laws cited in the Syllabus
The Solicitor General agreed with David, arguing that RA 6679 was not 1. SC Admin Circ. No. 1493 – these are guidelines on the
repealed by RA 7160. COMELEC disagreed, citing Sec. 43 of RA 7160 Katarungang Pambarangay conciliation procedure (See
which reduced the term of office of all local elective officials to 3 years. compilation of Special Laws for full text)
Amicus curiae, former Senator Aquilino Q. Pimentel, Jr. agreed with 2. RA 9285 (An Act to Institutionalize the use of Alternative
COMELEC. The SC agreed with COMELEC and Pimentel, and ruled that Dispute Resolution system in the Philippines
RA 7160 repealed the previous laws and reduced the term of office of a. Sec. 35 – Construction disputes which fall within the
baranggay officials to 3 years. original and exclusive jurisdiction of the Construction
Industry Arbitration Commission shall include:
Doctrine: RA 7160 is a set of laws that specifically applies to local (1) Those between or among parties to, or who are
government units. It specifically and definitively provides in Sec. 43-c otherwise bound by, an arbitration agreement,
that “the term of office of barangay officials… shall be for three directly or by reference whether such parties are
years.” It is a special provision, and RA 7160 is a special law insofar as project owner, contractor, subcontractor, quantity
it governs the term of office of barangay officials. In its repealing surveyor, bondsman or issuer of an insurance
clause, RA 7160 states that “all general and special laws x x x which are policy in a construction project.
inconsistent with any of the provisions of this Code are hereby repealed b. The Commission shall continue to exercise original and
or modified accordingly.” exclusive jurisdiction over construction disputes
although the arbitration is "commercial"

ALQUIZOLA v. OCOL The Lupong Tagapamayapa (Sec. 399)


August 27, 1999 - Composition:
1. Punong Barangay as chair
Summary: Alquizola, the punong barangay of Brangay Tubod, Iligan 2. 10 to 20 actual residents or workers in the barangay as
City, terminated the services of Ocol et al, who were composed of the members to be appointed by the Punong Barangay
baragay treasurer, barangay secretary, and other appointive barangay - Objective: To bring controversies among barangay residents
oficials. In turn, he appointed new barangay treasurer and as barangay to an amicable conclusion
secretary. He submitted both appointments to the Sangguniang  This is without prejudice to the settling of disputes
Barangay for approval. The Sanggunian rejected the appointments. among members of the indigenous cultural
communities through their traditional cultural
Doctrine: On how the power of the punong barangay to replace or structures
appoint includes the power to remove:
“Section 389. Chief Executive: Powers, Duties, and Functions. Pangkat ng Tagapagkasundo (Sec. 404)
– x x x- - A Pangkat is constituted for each dispute brought before the
For efficient, effective and economical governance, the purpose of Lupon
which is the general welfare of the barangay and its inhabitants - Composition:
pursuant to Section 16 of this Code, the punong barangay shall:  Three members who are chosen by the parties to a
“x x x x x x x x x controversy from a list of members of the Lupon
“(5) Upon approval by a majority of all the members of the sangguniang  If parties fail to agree on the composition, they shall be
barangay, appoint or replace the barangay treasurer, the barangay determined by the drawing of lots by the Lupon
secretary, and other appointive barangay officials;” Chairman
The term “replace” embraces not only the appointment of the
replacement but ALSO the prior removal of, or the vacation by, the Subject Matter of Amicable Settlement (Sec. 408)
official currently occupying the appointive position concerned. - The Lupon shall have authority to bring together the parties of
a controversy together for purposes of settling their
Thus, the power of appointment is to be exercised CONJOINTLY by the differences
Punong Barangay and a majority of all the members of the sangguniang  PROVIDED they are residents of the same
barangay. Without such conjoint action, neither an appointment nor a city/municipality
replacement is effectual. - GENERAL RULE: All disputes are submitted to the Lupon
- EXCEPTIONS:
Aside from what may be implicit in Section 389, there is no other 1. Where one party is the government, or any subdivision
provision in the LGC that treats of the power of the Punong Barangay to or instrumentality thereof;
remove the barangay secretary, the barangay treasurer, or any other 2. Where one party is a public officer or employee, and
appointive barangay official from office. dispute relates to the performance of his official
functions
Where the tenure of the office is not fixed by law, it is a sound and 3. Offenses punishable by imprisonment exceeding 1 year
useful rule to consider that the power to remove is deemed implied in or a fine exceeding P5,000
the power to appoint. 4. Offenses where there is no private offended party
5. Where the dispute involves real properties located in
On how the power of the punong barangay to appoint is subject to the different cities or municipalities
approval of the Sangguniang Barangay:  UNLESS parties agree to submit to amicable
The power of appointment of is to be exercised conjointly by the settlement by an appropriate lupon
punong barangay an a majority of all the members of the sangguniang 6. Disputes involving parties who actually reside in
barangay. barangays of different cities or municipalities
 EXCEPT when such barangay units adjoin each
other and parties thereto agree to submit their
differences to amicable settlement by an
appropriate Lupon
7. Such other classes of disputes which the President may UY v. CONTRERAS
determine in the interest of justice or upon September 26, 1994
recommendation of DOJ Sec.
Summary: Felicidad Uy leases from Susana Atayde a portion of a
Venue (Sec. 409) building for her beauty parlor. After the expiration of the lease contract,
an argument arose between them when Uy sought to remove her
PARTIES LUPON properties from the bilding. It turned into a physical fight prompting
Between actual residents of the the lessor Atayde to file a complaint with the barangay. However, she
Lupon of said barangay
same barangay did not attend the conciliation meetings and instead sought the filing of
Lupon of the barangay where the the information with the MTC against Uy. Uy filed a Motion to Dismiss
Between actual residents of
respondent or any of the raising the complainant’s non-compliance with the Katarungang
different barangays but within
respondents actually resides at Pambarangay Law. On the other hand, the complainant argued that the
same city/municipality
the option of complainant offense was about to prescribe and that before the offense prescribes,
Lupon of barangay were the real she had to file the criminal cases with the court. The lower court Judge
Involving real property or any Contreras denied the Motion to Dismiss. The issue before the SC was
property or the larger portion
interest therein the propriety of dismissing the action.
thereof is located
Between parties arising at the
Lupon of the barangay were the The SC dismissed the action for non-compliance with the Revised
workplace or at institutions of
workplace or institution is located Katarungang Pambarangay Law under the LGC.
learning

Objections to venue must be raised before the punong barangay. Failure Doctrine: Conciliation process at the Barangay level is a condition
to raise such is waiver. precedent for the filing of a complaint in Court. Non-compliance with
that condition precedent could make a complaint vulnerable to
Conciliation (Sec. 412) dismissal on the ground of lack of cause of action or prematurity. The
- An effort to effect an amicable settlement of disputes before condition is analogous to exhaustion of administrative remedies, or the
the lupon or the pangkat is a condition precedent for lack of earnest efforts to compromise suits between family members,
complaints, petitions, actions, or proceedings to be filed with a lacking which the case can be dismissed.
court of law
- EXCEPTIONS: Direct recourse to courts may be resorted to in However, such non-compliance is not a jurisdictional matter which
the following instances: would deprive a court of its jurisdiction. Where the fact of non-
1. Where accused is under detention compliance with such procedure has been seasonably raised as an issue
2. Where a person has otherwise been deprived of before the court first taking cognizance of the complaint, dismissal of
personal liberty calling for habeas corpus the action is proper.
3. Where actions are couples with provisional remedies
4. Where the action may otherwise be barred by statute of The precise technical effect of failure to comply with this requirement is
limitations much the same effect produced by non-exhaustion of administrative
remedies. The complaint becomes afflicted with the vice of pre-
NOTE: Prior referral to Lupon is NOT Jurisdictional maturity. The controversy is not ripe for judicial determination. The
- Therefore, it may be waived if not seasonably raised complaint becomes vulnerable to a motion to dismiss.

Effect of Amicable Settlement (Sec. 416) The prescriptive period becomes automatically suspended for a
- An amicable settlement entered into by the parties before the maximum period of sixty days when the complaints were filed with the
lupon or pangkat and an arbitration award issued by the latter lupon.
have the force and effect of a courts judgment
 AFTER 10 days
 UNLESS the settlement: ZAMORA v. HEIRS OF IZQUIERDO
1. Has been repudiated, or November 18, 2004
2. A petition to nullify the award has been filed
before the proper court Summary: The Zamoras, lessees, did not pay the increased rental
imposed by the lessor, Izquierdos. The Zamoras applied for a water
installation but was not given by an owner’s consent by the attorney of
MORATA v. GO the Izquierdos. Zamoras had several confrontations in the barangay
October 27, 1983 with the Izquierdos but no conciliation was made. The Izquierdos
obtained a Certification to File Action. The Izquierdos filed a complaint
Summary: Sps. Go filed a complaint for recovery of SOM plus damages for unlawful detainer against the Zamoras. The Zamoras filed a motion
against Sps. Morata before the Cebu CFI. Sps. Morata filed a MTD for the to dismiss arguing that the Punong Barangay, as Lupon Chairman, did
Sps. Go’s failure to allege prior availment of barangay conciliation not constitute the Pangkat ng Tagapagkasundo before whom mediation
process as required by PD 1508. CFI Judge Tomol denied the MTD, proceedings should have been conducted as required by the LGC.
saying that the conciliation process is mandatory only for cases
cognizable by inferior courts (MTCs, MeTCs). Upon appeal to the SC, the Doctrine: Section 412(a) of R.A. No. 7160 requires the parties to
Court disagreed with Judge Tomol and restrained Judge Tomol from undergo a conciliation process before the Lupon Chairman or
further proceeding with the case [until the barangay conciliation the Pangkat as a precondition to filing a complaint in court. In this case,
process is conducted]. the Punong Barangay, as Chairman of the Lupong Tagapamayapa,
conducted conciliation proceedings. There were confrontations before
Doctrine: The conciliation process at the barangay level, prescribed by the barangay chairman on 9 different dates wherein not only the issue
P.D. 1508 as a pre-condition for filing a complaint in court, is of water installation was discussed but also the terms of the lease and
compulsory not only for cases falling under the exclusive competence of the proposed execution of a written contract relative thereto. While it is
the metropolitan and municipal trial courts, but for actions cognizable true that the Sertifikasyon is entitled ‘Ukol Sa Hindi Pagbibigay Ng
by the regional trial courts as well. Pahintulot Sa Pagpapakabit Ng Tubig’, this title must not prevail over
the actual issues discussed in the proceedings.
In Diu vs. Court of Appeals, the SC held that “while the Pangkat was not Dipologo City, against Daria Lagas Galleros, a resident of Misamis
constituted, however, the parties met nine (9) times at the Office of Occidental. The case was decided by Judge Hernandez, who dismissed
the Barangay Chairman for conciliation wherein not only the issue of the suit for failure of the parties to refer the issue to the Lupong
water installation was discussed but also petitioners’ violation of the Tagapamayapa before filing the same in court. An administrative
lease contract. It is thus manifest that there was substantial compliance complaint was thereafter filed against Judge Hernandez for abuse of
with the law which does not require strict adherence thereto.” authority and ignorance of law.

The Office of the Court Administrator recommends the dismissal of the


BOLEYLEY v. VILLANUEVA case because there is no allegation of bad faith or knowingly rendering
September 14, 1999 an unjust judgment. The SC did not adopt the recommendations of the
OCA. The SC found Judge Hernandez guilty of gross ignorance of the law
Summary: The RTC granted Surla’s motion to dismiss Boleyley’s and was ordered to pay a fine of P2,000 with a warning that repetition
complaint for a sum of money on the ground that it was not previously of the same act will be dealt with more severely. The SC found that
referred to the barangay lupon or pangkat for conciliation. The SC, there was no need to submit the dispute to the Lupong Tagapamayapa
however, reversed, finding that, as implied from the allegations in the because the parties do not reside in the same city or municipality.
complaint, which governs in determining venue, Surla and Boleyley did
not reside in the same city or municipality. Doctrine: In Tavora v Veloso, the Court ruled that where parties do not
reside in the same city or municipality or in adjoining barangays, there
Doctrine: Jurisdiction of the court over the subject matter of the action is no requirement for them to submit their dispute involving real
is determined by the allegations of the complaint, irrespective of property to the Lupong Tagapamayapa.
whether or not the plaintiff is entitled to recover upon all or some of the
claims asserted therein. The jurisdiction of the court cannot be made to
depend upon the defenses set up in the answer or upon the motion to IDOLOR v. CA
dismiss, for otherwise, the question of jurisdiction would almost February 7, 2001
entirely depend upon the defendant.
Summary: Teresita Idolor obtained a loan secured by a Real Estate
Mortgage from Gumersindo De Guzman. When Idolor failed to pay, De
MENDOVA v. AFABLE Guzman filed a complaint before the Office of the Brgy. Capt. of Brgy.
December 4, 2002 Ramon Magsaysay, QC which resulted in a Kasunduang Pag-aayos. In
the Kasunduan, Idolor acknowledged the outstanding loan obligation
Summary: Complainant Abraham Mendova alleged that Robert Palada and was given a 90-day grace period to settle the account. However,
committed the crime of slight physical injuries on February 15, 1998. Idolor failed on this undertaking which prompted Gumersindo to file a
On February 18, 1998, Mendova filed his complaint with the Office of motion for execution before the Office of the Brgy. Capt. which issued a
the Brgy. Chairperson. The parties failed to reach an amicable certification to file action. Gumersindo then filed an extrajudicial
settlement. Hence, Mendova filed with the MCTC a complaint for slight foreclosure of the real estate mortgage.
physical injuries. Judge Afable dismissed the complaint on the ground of
prescription. In his Decision, Judge Afable stated that a light offense The property was sold in a public auction to Gumersindo. Idolor then
(such as slight physical injuries) prescribes in two months. From the filed with the RTC a complaint for annulment of Sheriff’s Certificate of
date of the commission of the alleged offense, more than two months Sale with prayer for TRO and writ of preliminary injunction. RTC issued
have elapsed. Mendova filed with the Office of the Court Administrator the writ of injunction. On appeal, CA reversed the RTC and annulled the
an administrative complaint against Judge Afable for ignorance of the preliminary injunction. SC affirmed the CA.
law. The OCA found him guilty. The Supreme Court ruled that the
administrative complaint was premature. Mendova should have availed Doctrine: There is no doctrinal pronouncement on Katarungang
of a judicial remedy, which is a Motion for Reconsideration. Pambarangay but this is an illustrative case on an amicable settlement by
the Office of Barangay Capt. which can be a subject of a motion for
Doctrine: Pursuant to the provisions of Section 410(c) of The Local execution.
Government Code of 1991, the filing of the complaint with the office of
the Brgy. Chairperson interrupted the prescriptive period and started (On Injunction) It is always a ground for denying injunction that the
to run again upon receipt by the complainant of the Certification to File party seeking it has insufficient title or interest to sustain it, and no
Action or Certificate of Repudiation issued by the Pangkat Secretary. claim to the ultimate relief sought.

The law provides ample judicial remedies against errors or (On Novation) Where the parties to the new obligation expressly
irregularities committed by a Trial Court in the exercise of its recognize the continuing existence and validity of the old one, where, in
jurisdiction. The ordinary remedies against errors or irregularities other words, the parties expressly negated the lapsing of the old
which may be regarded as normal in nature (i.e., error in appreciation obligation, there can be no novation.
or admission of evidence, or in construction or application of
procedural or substantive law or legal principle) include a motion for
reconsideration (or after rendition of a judgment or final order, a AQUINO v. AURE
motion for new trial), and appeal. February 18, 2008

The extraordinary remedies against error or irregularities which may be Summary: Aure filed a complaint for ejectment with the MeTC against
deemed extraordinary in character (i.e., whimsical, capricious, despotic Aquino. Sps. Aure alleged that after they give the money to Aquino,
exercise of power or neglect of duty, etc.) are inter alia the special civil however, the latter refused to vacate the subject property thereafter.
actions of certiorari, prohibition or mandamus, or a motion for MeTC dismissed the case on the ground that the parties belonging
inhibition, a petition for change of venue, as the case may be. within the same barangay failed to settle the case amicably at the
barangay level. RTC affirmed MeTC judgement. CA reversed. SC ruled
that non-compliance with the baranagay conciliation proceedings does
VERCIDE v. JUDGE HERNANDEZ not remove the courts of its jurisdiction. Here, Aquino did not raise lack
April 6, 2000 of barangay conciliation proceeding in his answer. He cannot later use
such defense to ask for the dismissal of the case.
Summary: A suit for recovery of possession of a piece of land located in
Misamis Occidental was filed by Vercide and his wife, residents of
Doctrine: The precise technical effect of failure to comply with the that no conciliation had been reached by the parties, the case should be
requirement of Section 412 of the Local Government Code on barangay dismissed.
conciliation is the same produced by non-exhaustion of administrative
remedies -- the complaint becomes afflicted with the vice of pre- (Admin circular 14-93) All disputes are subject to Barangay
maturity; and the controversy alleged is not ripe for judicial conciliation and prior recourse thereto is a pre-condition before filing a
determination. The complaint becomes vulnerable to a motion to complaint in court or any government offices, except in the following
dismiss. Nevertheless, the conciliation process is not a jurisdictional disputes:
requirement, so that non-compliance therewith cannot affect the (1) Where one party is the government, or any subdivision or
jurisdiction which the court has otherwise acquired over the subject instrumentality thereof;
matter or over the person of the defendant. (2) Where one party is a public officer or employee and the
dispute relates to the performance of his official functions;
(3) Where the dispute involves real properties located in
PANG-ET v. MANACNES-DAO-AS different cities and municipalities, unless the parties thereto
March 2, 2007 agree to submit their difference to amicable settlement by an
appropriate Lupon;
Summary: An action for recovery of real property was filed in the MCTC (4) Any complaint by or against corporations, partnerships or
by Pang-et against an Igorot couple. During pre-trial, the parties to the juridical entities, since only individuals shall be parties to
case agreed to refer the matter for arbitration in the Lupon. The parties Barangay conciliation proceedings either as complainants or
appeared before the Lupon, but to no avail as the Igorot couple refused respondents;
to enter into any agreement for arbitration and insisted in pursuing the (5) Disputes involving parties who actually reside in barangays
case in court, so the Lupon issued a certificate to file action (the of different cities or municipalities, except where such
daughter of the couple signed the agreement for arbitration on their barangay units adjoin each other and the parties thereto
behalf, but this was disregarded by the Lupon as it should be the couple agree to submit their differences to amicable settlement by
who would personally sign any agreement, and the couple clearly an appropriate Lupon;
manifested their intention to pursue the case in court). However, the (6) Offenses for which the law prescribes a maximum penalty of
MCTC ordered the Lupon to issue an Arbitration Award since an imprisonment exceeding one [1] year or a fine of over five
agreement for arbitration was entered into by the parties (the one thousand pesos;
signed by the daughter only). (7) Offenses where there is no private offended party;
(8) Disputes where urgent legal action is necessary to prevent
The Lupon made an arbitration award and Pang-et filed with the MCTC injustice from being committed or further continued,
an action for enforcement of said award. MCTC: Petition dismissed, specifically the following:
couple did not personally sign the arbitration agreement. RTC: MCTC (a) Criminal cases where accused is under police custody
reversed. CA: RTC reversed, MCTC reinstated. SC: Petition denied, MCTC or detention
and CA ruling upheld. (b) Petitions for habeas corpus by a person illegally
deprived of his rightful custody over another or a
Doctrine: The object of the Katarungang Pambarangay Law is the person illegally deprived of or on acting in his behalf;
amicable settlement of disputes through conciliation proceedings (c) Actions coupled with provisional remedies such as
voluntarily and freely entered into by the parties. The disputing parties preliminary injunction, attachment, delivery of personal
are not compelled to settle their controversy during the barangay property and support during the pendency of the
proceedings before the Lupon or the Pangkat. They are free to find action; and
recourse in the courts in the event that no compromise is reached. (d) Actions which may be barred by the Statute of
Limitations.
What is compulsory under the Katarungang Pambarangay Law is that (9) Any class of disputes which the President may determine in
there be a confrontation between the parties before the Lupon the interest of justice or upon the recommendation of the
Chairman or the Pangkat and that a certification be issued that no Secretary of Justice;
conciliation or settlement has been reached, as attested to by the Lupon (10) Where the dispute arises from the Comprehensive Agrarian
or Pangkat Chairman, before a case falling within the authority of the Reform Law (CARL)
Lupon may be instituted in court or any other government office. In (11) Labor disputes or controversies arising from employer-
other words, the only necessary pre-condition before any case falling employee relations
within the authority of the Lupon or the Pangkat may be filed before a (12) Actions to annul judgment upon a compromise which may be
court is that there has been personal confrontation between the parties filed directly in court
but despite earnest efforts to conciliate, there was a failure to amicably
settle the dispute.

Class Notes: Can a change of name be the subject of conciliation? No, it is


a special proceeding and it is not adversarial.

AGBAYANI v. CA
June 25, 2012

Summary: Agbayani filed a complaint against Genabe for grave oral


defamation. the office of the city prosecutor found probable cause. but
DOJ dismissed the complaint because it did not comply with the
mandatory barangay conciliation as required by sections 408 and 409D
of LGC. SC affirmed DOJ.

Doctrine: The compulsory process of arbitration is a pre-condition for


the filing of the complaint in court. Where the complaint (a) did not
state that it is one of excepted cases, or (b) it did not allege prior
availment of said conciliation process, or (c) did not have a certification
THE SANGGUNIANG KABATAAN the context of the CBA. The SC ruled that SK elections are considered as
regular elections.
Special Laws cited in the Syllabus
1. RA 7808 – act resetting the SK elections to the 1st Monday of Doctrine: SK elections are regular elections. The fact that only those
May 1996, and every 3 years thereafter between 15 and 21 take part in the election for members of the SK does
2. RA 8044 – act creating the National Youth Commission not make such election any less a regular local election.
a. Sec. 10 defines the functions of the Commission
3. RA 9164 – act providing for a synchronized barangay and SK
elections MIGUEL v. CA
4. RA 9340 – act amending RA 9164, resetting the barangay and February 23, 1994
SK elections
Summary: Roberto A. Miguel was elected President of KPB-QC, whch
Composition of the SK was created under the old LGC. Afterwards, the new LGC took effect,
1. SK Chairman abolishing the Pambansang Katipunan ng mga Barangay and replacing
2. 7 members it with the Ligas. Pending elections to be held under the LGC of 1991,
3. Appointive officials: Miguel continued to serve as president of Liga-QC. However, in spite of
a. SK Secretary Miguel’s objections, an ad hoc committee elected a new set of officers,
b. SK Treasurer including a new president, under the old constitution and by-laws of the
KPB-QC. The SC, in ruling that the election was invalid, said that,
Age Limit according to the RRI of the LGC of 1991, a uniform constitution and by-
- Age limit for membership in the SK is 18 years, the minimum laws must first be adopted prior to the holding of any elections.
is 15 years (lowered from 21 to 18 by RA 9164)
 Note also that to qualify for membership, they must be Doctrine: Article 210 (f) (1) of Rule XXIX of the RRI: " [p]ending election
duly registered in the list of the SK or in the official list of the presidents of the municipal, city, provincial, and metropolitan
of barangay residents in the custody of the barangay chapters of the Liga, the incumbent presidents of the association of the
secretary barangay councils in the municipality, city, province and Metropolitan
- But an SK official who had been elected when he was below 18 Manila shall continue to act as presidents of the corresponding liga
but had, since the election, passed 18, he will still be allowed chapters"
to serve the full term of the office to which he had been
elected Article 210 (d) (3) of Rule XXIX: "incumbent presidents of the
municipal, city, and provincial chapters of the liga shall continue to
Katipunan ng Kabataan (Sec. 424) – the barangay youth organization serve as ex officio members of the sanggunian concerned until the
- Composition expiration of their term of office, unless sooner removed for cause."
 Citizens actually residing in the barangay for at least 6
months The election of the first set of officers of the national and local chapters
of the Liga cannot be held unless a constitution and by-laws for the Liga
 Who are 15 but less than 18 years of age on the day of
is first adopted and ratified by the barangay national assembly (Rule
election
XXIX, Art. 210 [g] [3]). The said constitution and by-laws was to be
 Duly registered in the list of the SK or in the official
drafted by the committee created under Article 210 (f) (2) of the RRI.
barangay list
- For the meaning of “less than 18 years of age” see Garvida v.
Sales
MARQUEZ v. COMELEC
August 25, 1999
Powers and Functions of the SK
- They are outline in Sec. 426
- The most important function of the SK is to initiate programs Summary: Marquez and Santos both ran for SK chairman. Marquez won
to enhance the social, political, economic, cultural, intellectual, and was proclaimed as winner. Santos filed an election protest before
moral, spiritual, and physical development of its members the MTC on the ground that Marquez was already overage. Marquez
questioned the jurisdiction of the MTC. The Court upheld the
Qualifications of SK Officials (Sec. 428) (as amended by RA 9164) jurisdiction of the MTC since the case was filed after the election and
1. Citizens of the Republic proclamation of Marquez.
2. At least 15 years of age but not more than 21 on the day of
their election Doctrine: So any contest relating to the election of members of the
3. Qualified voters of the Katipunan ng Kabataan Sangguniang Kabataan (including the chairman) – whether pertaining
4. Residents of the barangay for at least 1 year immediately to their eligibility or the manner of their election – is cognizable by
prior to the election MTCs, MCTCs, and MeTCs. Section 6 of COMELEC Resolution No. 2824
5. Able to read and write Filipino, English, or local dialect (“cases involving the eligibility or qualification of candidates shall be
6. Innocent of any crime involving moral turpitude decided by the city/municipal Election officer whose decision shall be
final”) applies only to proceedings before the election.
Term of Office – SK Chair and members shall hold office for a period of 3
years, unless sooner removed for cause as provided by law, Before proclamation, cases concerning eligibility of SK officers and
permanently incapacitated, die, or resign from office (Sec. 429) members are cognizable by the Election Officer or EO as he is called in
Section 6. But after the election and proclamation, the same cases
become quo warranto cases cognizable by MTCs, MCTCs, and MeTCs.
ASSOCIATED LABOR UNIONS v. LETRONDO
October 18, 1994
GARVIDA v. SALES
April 18, 1997
Summary: Associated labor union and its employer AMS Farming
entered into a collective bargaining agreement which provides, among
other, that regular elections are paid holidays. The President of the Phil Summary: Lynette Garvida’s proclamation as winner in the SK elections
declared as a ‘special day’ the election day for Sangguniang Kabataan. was put on hold on the ground that she was already 21 years and 10
Employees filed claims for holiday pay for the said date which was months old, exceeding the age limit for membership in the Katipunan
denied by the company on the ground that it is not a regular election in ng Barangay as laid down in Section 3[b] of COMELEC Resolution 2824.
SC, despite stating that COMELEC didn’t follow procedure to acquire did not have jurisdiction. Their MR was denied by COMELEC, and the SC
proper jurisdiction declared Garvida ineligible for being overage. agreed with COMELEC.

Doctrine: If the candidate is over the maximum age limit on the day of Doctrine: The grant by the Constitution to the COMELEC of the power to
the election, he is ineligible. The fact that the candidate was elected will investigate and prosecute election offenses is intended to enable the
not make the age requirement directory, nor will it validate his election. COMELEC to assure the people of “free, orderly, honest, peaceful and
The will of the people as expressed through the ballot cannot cure the credible elections.” This is an adjunct to its constitutional duty to
vice of ineligibility. enforce and administer all election laws. Failure by the COMELEC to
exercise this power could result in the frustration of the true will of the
A closer look at LGC will reveal that there is a distinction between max people and make an idle ceremony of the sacred right and duty of every
age of a Member and max age of an elective SK official qualified citizen to vote.
1. Sec. 424 sets a maximum age of 21; no further provision as to
when The finding of probable cause in the prosecution of election offenses
2. Sec. 428 provides that one must be at most 21 on the day of rests in the COMELEC’s sound discretion. The COMELEC exercises the
his election constitutional authority to investigate and, where appropriate,
prosecute cases for violation of election laws, including acts or
THUS, the member may be more than 21 on election day or on the day omissions constituting election frauds, offenses and malpractices.
he registers as member of the Katipunan, BUT he must not be more Generally, the Court will not interfere with such finding of the
than 21 on the day of election COMELEC absent a clear showing of grave abuse of discretion.

The phrase “not more than 21 years of age” means not over 21 years, It is only in the exercise of its adjudicatory or quasi judicial powers that
not beyond 21 years. Meaning 21 365-day cycles. It does not mean 21 the COMELEC is mandated to hear and decide cases first by division and
years and one or some days. It is not equivalent to less than 22 years then, upon motion for reconsideration, by the COMELEC en banc.

So, General rule is that en elective official of the SK must not be more Class Notes
than 21 years on the day of election. Exception: when he reaches the - If in the exercise of quasi-judicial powers, the case must first
age of 21 during incumbency since Sec. 423(b) allows him to serve the be heard and decided by a Division before review by the En
remainder of his term Banc
- But in the exercise of COMELEC’s administrative function, one
HOWEVER, Sec. 3(b) of Com Res 2824 is ultra vires insofar as it sets the may go directly to En Banc
age limit of a voter in the SK elections at exactly 21 on the day of
election. This qualification is not provided in Sec. 424 of the LGC . Since
Sec. 424 does not provide that the max age of a member of the SAMBARANI v. COMELEC
Katipunan is determined on the day of election September 15, 2004

Summary: Due to a failure of elections in 11 barangays in Lanao del Sur,


ZARATE v. COMELEC the COMELEC issued Resolution No. 5479 setting special elections on
November 19, 1999 13 August 2002 in the affected barangays in Lanao del Sur
including the 5 barangays. But subsequently Acting Election Officer
Summary: The Board of Canvassers proclaimed Julian Lallave, Jr as SK Esmael Maulay issued a certification that there were no special
Chariman of Brgy Incan, Malasiqui, Pangasinan. Llave garnered 46 votes elections held on the said date. COMELEC issued the Resolution
as against Zarate who garnered 45 votes. Zarate filed an election directing DILG to proceed with the appointment of Barangay Captains
protest with the MTC which set aside the proclamation of Lallave ruling and Barangay Kagawads as well as SK Chairmen and SK Kagawads in
that the ballots containing “JL” are stray votes and should not be accordance with the LGC since to hold another special election in these
counted in favor of Lallave. Lallave appeal to the Comelec. Comelec en barangays as prayed for by petitioners is untenable. SC reversed the
banc set aside the MTC decision, declared Lallave as the SK Chairman COMELEC resolution.
and ruled that the ballots containing “JL” sufficiently identified Lallave
and must be counted in his favor. SC set aside Comelec resolution and Doctrine: In fixing the date for special elections the COMELEC should
ruled that pursuant to Section 3, Subdivision C of Article IX of the see to it that:
Constitution, Comelec en banc does not have the authority to hear and (1) It should not be later than thirty (30) days after the cessation
decide election cases at the first instance. The case must be referred to a of the cause of the postponement or suspension of the
Comelec Division election or the failure to elect; and,
(2) It should be reasonably close to the date of the election not
Doctrine: Under Section 3, Subdivision C of Article IX of the held, suspended or which resulted in the failure to elect.
Constitution, election cases shall be heard and decided in division,
provided that motions for reconsideration of decisions shall be decided The first involves a question of fact. The second must be determined in
by the Commission en banc. The Commission, sitting en banc, does not the light of the peculiar circumstances of a case. Thus, the holding of
have the authority to hear and decide the same at the first instance. elections within the next few months from the cessation of the cause of
the postponement, suspension or failure to elect may still be considered
“reasonably close to the date of the election not held.”
BAYTAN v. COMELEC
February 4, 2003 The prohibition on conducting special elections after thirty days from
the cessation of the cause of the failure of elections is not absolute. It is
Summary: Reynato Baytan, Reynaldo Baytan and Adrian Baytan directory, not mandatory, and the COMELEC possesses residual power
registered in Barangay 18. They subsequently registered again, this to conduct special elections even beyond the deadline prescribed by
time in Barangay 28. They sent a letter to COMELEC asking for advice law. The COMELEC erroneously perceived that the deadline in Section
on how to cancel their previous registration. The COMELEC en banc 6 is absolute. The COMELEC has broad power or authority to fix other
recommended the filing of criminal cases against them for double dates for special elections. The COMELEC may fix other dates for the
registration. They argued that they were in good faith and that their conduct of special elections when the same cannot be reasonably held
letter to COMELEC should be treated as substantial compliance in within the period prescribed by law.
cancelling their previous registration, and that the COMELEC en banc
c. Initiates and maximizes the generation of resources and
THE MUNICIPALITY revenues and applies them to the priorities defined in
Sec. 18 LGC
See Sec. 440-447, and 444(b)(3) d. Ensures delivery of basic services under Sec. 17
e. Exercises such powers and performs such other duties
Role of the Municipality – The municipality, consisting of a group of as may be prescribed by law or ordinance
barangays, serves primarily as a general purpose government for the
coordination and delivery of basic, regular, and direct services and 2. He has the power to appoint all officials and employees who
effective governance of the inhabitants within its territorial jurisdiction are paid out of municipal funds, whether wholly or partially
(Sec. 440) a. UNLESS their appointment is lodged with other officials
(1) Like in cases of the officials and employees of the
Officials of the Municipal Government (Sec. 443) sanggunian who are appointed by the Vice Mayor
A. Mandatory Municipal Elective Officials (2) Also, the treasurer and assistant treasurer are
1. Mayor appointed by Finance Secretary
2. Vice Mayor b. Subject to sanggunian concurrence
3. Regular members of the Sangguniang Bayan
4. President of the Municipal Chapter of the Liga ng mga 3. The mayor may also cause to be charged administratively or
Barangay judicially not only municipal officials or employees, but also
5. President of the Pambayang Pederasyon ng mga SK national officials or employees who violate any law in the
6. Representatives of the following sectors: performance of their official duties within the municipality
a. Women
b. Labor (agriculture or industry) 4. Also remember that he is the deputized representative of the
c. Urban poor, indigenous cultural communities, of NAPOLCOM, so the mayor:
the disable, and other sectors as determined by a. Formulates the law and order plan of the municipality
the sanggunian concerned and, when approved, implements it
b. Exercises general and operational control and
B. Additional Members of the Sanggunian supervision over local police forces in the municipality
1. President of the Liga ng mga Barangay of a city or
province 5. Mayor’s salary grade: SG 27
2. President of the Pedersayon ng mga Sangguniang
Barangay of a city of province The Sangguniang Bayan (Sec. 447)
3. The 3 sectoral representatives 1. Powers and Functions – categories:
a. Those needed for efficient and effective municipal
C. Mandatory Appointive Officials12 governance such as the power to approve and pass
1. Secretary of the Sanggunian13 ordinances and resolutions
2. Treasurer14 b. Those needed to generate and maximize the use of
3. Assessor resources and revenues for the development of the
4. Accountant municipality
5. Budget Officer c. Those needed to tax or regulate business activities
6. Planning and Development Coordinator including the grant of franchises in the municipality
7. Engineering/building Official d. Those needed to regulate the use of land, buildings, and
8. Health officer structures within the municipality
9. Civil Registrar e. Those needed to ensure efficient and effective delivery
of basic services (Sec. 17 LGC)
D. Other Appointive Officials (not mandatory)
1. An administrator 2. The Sangguniang Bayan, like the Sangguniang Panlungsod,
2. A legal officer has the power to review all ordinances approved by the
3. Agriculturist Sangguniang Barangay and the EOs issued by the Punong
4. Environment and natural resources officer Barangay to determine whether or not they are within the
5. Social welfare and development officer scope of their powers
6. Architect
7. Information officer 3. It also has the power to create offices and positions for the
municipal government
The Sanggunian checks the major appointments and may create other
offices necessary to carry out the functions of the municipality. It may 4. By a majority vote of all members, the Sanggunian may
also create other offices necessary to carry out the purposes of the authorize the mayor to negotiate and contract loans and
municipal government. other forms of indebtedness

The Mayor (Sec. 443) 5. It also has the power to grant tax exemptions, incentives, or
1. Powers – in general: reliefs to entities engaged in COMMUNITY GROWTH
a. Exercises general supervision and control of all INDUSTRIES with the concurrence of at least 2/3 of all its
municipal programs, projects, services, and activities members
b. Enforces all laws and ordinances; implements all
municipal programs, projects, services, and activities 6. In connection with the power to grant licenses lodged with it,
the Sanggunian may now regulate business, occupations,
professions, or callings that DO NOT REQUIRE
12 GOVERNMENT EXAMINATIONS within its jurisdiction
Appointed by the mayor subject to confirmation or approval by the
majority vote of all members of the sanggunian concerned 7. Salary range of members: SG 25
13
The Secretary is appointed by the Vice-Mayor
14
Treasurer and Assistant Treasurer are appointed by the Finance
Secretary from a list of 3 qualified persons submitted by the mayor
OLIVAREZ v. SANDIGANBAYAN reclassify land within jurisdiction of the municipality subject to
October 4, 1995 pertinent provision of the LGC, and (c) enact integrated zoning
ordinances in consonance with the approved comprehensive land use
Summary: Baclaran Credit Cooperative, Inc. (BCCI) charged, in the plan, subject to existing laws, rules and regulations.
Ombudsman, Parañaque Mayor Dr. Pablo R. Olivarez with violation of
the Sec 3(e) of RA 3019 or Anti-Graft and Corrupt Practices for The constitutional guaranty of non-impairment of contracts is limited
unreasonably refusing to issue a mayor's permit. by the exercise of the police power of the State, in the interest of public
health, safety, morals and general welfare. Contractual restrictions on
The Ombudsman thereafter filed an Information for Violation of Sec. the use of property could not prevail over the reasonable exercise of
3(e) of R.A. 3019 in the Sandiganbayan. police power through zoning regulations.

Olivarez argues that there was no probable cause for the filing of an
information against him. He argues that no permit could be issued in RAMOS v. CA
favor of BCCI because the latter never filed an application therefor with October 30, 1981
the proper office, that is, the Business Permit and Licensing Office.
Summary: The municipality of Hagonoy, Bulacan, through a private law
The SC held that there was probable cause for the filing of an firm filed a case for the recovery of a fishpond against the lessees. The
information against Olivarez. provincial fiscal of Bulacan and the municipal attorney of Hagonoy
entered their appearance as counsel for the municipality with the
Doctrine: On how a Municipal Mayor cannot deny absence of knowledge manifestation that its private counsel would be under their control and
that he has the power to pass upon application for permits: supervision. Notwithstanding this appearance, the defendants lessees
It is not denied that on November 13, 1992, BCCI, through its general and sublessees moved to disqualify the law firm from serving as
manager, wrote Olivarez requesting for a permit to operate, but this counsel of the municipality.
was rejected outright by him on the theory that the application should
be made with the proper municipal official. The SC disqualified the law firm from appearing in the case.

The indifference shown by Olivarez to BCCI's application taints his Doctrine: A private law firm should not be allowed to act as counsel for
actuations with dubiety. As the mayor of the municipality, the officials the municipality in collaboration with the fiscal and the municipal
in the Business Permit and Licensing Office were definitely under his attorney.
authority and he was not without recourse to take appropriate action
on the letter-application of BCCI although the same was not strictly in The fact that the municipal attorney and the fiscal are supposed to
accordance with normal procedure. collaborate with a private law firm does not legalize the latter’s
representation of a municipality. While a private prosecutor is allowed
There was nothing to prevent him from referring said letter-application in criminal cases, an analogous arrangement is not allowed in civil cases
to the licensing department, but which paradoxically he refused to do. wherein the plaintiff is a municipality.

Whether Olivareswas impelled by any material interest or ulterior Section 1683 of the Revised Administrative Code, as complemented by
motive may be beyond us for the moment since this is a matter of Section 3 of the Local Autonomy Law, is clear in providing that only the
evidence, but the environmental facts and circumstances are sufficient provincial fiscal and the municipal attorney can represent a
to create a belief in the mind of a reasonable man that this would not be municipality in its lawsuits. That provision is mandatory.
completely improbable, absent countervailing clarification.
Class Notes: Rules on Representation
Lastly, it may not be amiss to add that Olivarez, as a municipal mayor, is 1. Civil cases – Legal Officer
expressly authorized and has the power to issue permits and licenses  Office of the Legal Officer is mandatory for province and
for the holding of activities for any charitable or welfare purpose, city
pursuant to Section 444 (b) (3) (iv and v) of the Local Government Code  Optional for municipality
of 1991 (Republic Act No. 7160). Hence, he cannot really feign total lack  Based on case law, the prosecutor represents
of authority to act on the letter-application of BCCI. 2. Criminal cases
a. Prosecutor
b. Private prosecutor allowed
UNITED BF HOMEOWNERS’ ASSOCIATION v. 3. Administrative cases – Legal Officer
PARANAQUE CITY MAYOR
February 7, 2007
MUNICIPALITY OF PILILLA v. CA
Summary: The Sangguniang Bayan of Pque enacted Municipal June 28, 1994
Ordinance 97-08 which reclassified El Grande and Aguirre Avenues in
BF Homes Pque from residential to commercial areas. Because of this, Summary: The Municipality of Pililla won a case against PPC. The
UBFHAI together with other homeowners associations filed a petition decision became final. Atty. Mendiola, a private lawyer, filed a petition
for prohibition with the CA, assailing the constitutionality of said for such judgment’s execution on behalf of the Municipality of Pililla.
Ordinance. They claimed that such ordinance was unconstitutional When the case reached the CA, PPC questioned Atty. Mendiola’s
because it amounts to impairment of the contracts that BF Homes Pque authority to represent the Municipality of Pililla. The SC held that Atty.
had with lot buyers, which says that the property shall be used “for Mendiola cannot represent the Municipality.
residential purposes only”. The Municipal City Mayor and other public
respondents argue that he Ordinance was a valid exercise of police Doctrine: The matter of representation of a municipality by a private
power and that such can supersede the contractual obligations entered attorney has been settled in Ramos vs. CA where the SC ruled that
into by BF Homes and homeowners. The CA dismissed the petition and private attorneys cannot represent a province or municipality in
held that the Ordinance was a valid exercise of police power of the lawsuits. The exception is that the provincial fiscal was disqualified to
Municipality. SC agreed with the CA and said that the enactment was handle the municipality’s case and such fact must appear on record. The
within the powers of the Sangguniang Bayan. fiscal’s refusal to represent the municipality is not a legal justification
for employing the services of private counsel. Instead of engaging the
Doctrine: Under Sec. 447, LGC, the Sangguniang Bayan has the power to services of a special attorney, the municipal council should request the
(a) adopt a comprehensive land use plan for the municipality, (b) Secretary of Justice to appoint an acting provincial fiscal in place of the
provincial fiscal who has declined to handle and prosecute its case in When the municipality is an adverse party in a case involving the
court, pursuant to Section 1679 of the Revised Administrative Code. provincial government or another municipality or city within the
The legality of his representation can be questioned at any stage of the province. [Sec. 481 of the LGC]
proceedings.
When may local government officials, sued in their official capacity,
secure the services of private counsel?
RAMOS v. CA The key to resolving this issue lies on the nature of the action and the
March 3, 1997 relief that is sought. Where rigid adherence to the law on
representation of local officials in court actions could deprive a party of
Summary: Atty. Romanillos appeared a collaborating counsel for the his right to redress for a valid grievance, the hiring of private counsel is
Municipality of Baliuag. Ramos, et al., the adverse party in the case, proper.
questioned his authority to appear for the municipality. A joint motion
was filed for Atty. Romanillos to withdraw and for Provincial Atty. A provincial governor sued in his official capacity may hire a private
Regalado to adopt the proceedings participated in by Atty. Romanillos. lawyer when the complaint contains other allegations and a prayer for
The RTC, as affirmed by the CA, granted the joint motion, finding that moral damages which must be satisfied by the local official concerned
(1) Ramos, et al., were stopped from raising the issue since they did so in his private capacity.
only when a formal offer of evidence has been made, (2) Atty.
Romanillos, was merely a collaborating counsel, and (3) in any case, the Where the allegations are aimed at questioning certain acts that can
adoption by Atty. Regalado cured the defect, if any, of the proceedings. bring the case beyond the confines of official functions and where it is
The SC ruled that (1) Ramos, et al., are not estopped since the issue can alleged that the local official acted in a wanton, oppressive, or
be raised at any stage of the proceedings, (2) the status as collaborating malevolent manner, hiring of private counsel is justified because the
counsel cannot validate the unlawful representation, but (3) the local official’s actions were ultra vires and this may result in the
adoption of Provincial Atty. Regalado renders new trial unnecessary. incurrence of personal liability.

Doctrine: Only the provincial fiscal, provincial attorney, and municipal


attorney may represent a municipality in its lawsuits, except: MANCENIDO v CA
(1) when original jurisdiction over the case is vested in the SC, April 12, 2000
(2) when the municipality is a party adverse to the provincial
government, Summary: Mancenido filed a petition for mandamus and damages with
(3) when the municipality is a party adverse to another the RTC of Camarines Norte against the provincial board of Camarines
municipality in the same province, or when the provincial Norte, school board, provincial governor, provincial treasurer, and
attorney, his wife, or child, is pecuniarily involved in the case. provincial auditor (all of whom were represented by a private counsel)
for the payment of unpaid salary increases for teachers. The RTC
rendered a decision in favor of Mancenido and ordered the payment of
ALINSUG v. RTC the claim. The Provincial Board, et al. filed a notice of appeal, which the
August 23, 1993 RTC granted. It later issued an order recalling the said order granting
the appeal. Provincial Board, et al. filed a petition for mandamus,
Summary: Zonsayda Alinsug was a regular employee of the municipal prohibition, and injunction with the CA. The CA granted the appeal and
government of Escalante, Negros Occidental, when she received a recognized the authority of the private counsel of the Provincial Board.
permanent appointment as Clerk III in the office of the Municipal Mancenido appealed to the SC. The SC ruled in favor of the Provincial
Planning and Development Coordinator of the same municipality. Board and recognized the authority of the private counsel to represent
Subsequently, Alinsug received an order from the new mayor detailing the Provincial Board.
her to the Office of the Mayor. She complied with the said Order.
Subsequently, Alinsug absented herself from work to attend to family Doctrine: In resolving whether a local government official may secure
matters. She did not ask permission from the mayor. The mayor issued the services of private counsel in an action filed against him in his
an Order suspending Alinsug for one month, and the suspension carried official capacity, the nature of the action and the relief sought are to be
with it forfeiture of certain benefits and leave credits during her considered. In the case at bar, the action involved damages. In view of
suspension. Alinsug filed an injunction case against the mayor and the damages sought which, if granted, could result in personal liability,
municipal treasurer. The said local officials, through a private counsel, respondents was properly represented by private counsel.
filed their Answer. Alinsug asked the court to expunge from the records
their Answer, arguing that said local officials should be represented by
either the municipal legal officer or the provincial legal officer or NATIVIDAD v. FELIX
prosecutor. The Supreme Court ruled that the local officials involved February 4, 1994
were properly represented by a private counsel.
Summary: The widow of Severino Aquino, Lourdes, sent a letter to the
Doctrine: In instances where personal liability on the part of local PNP requesting the Tarlac Prov. Prosec. to investigate the Municipal
government officials is sought, they may properly secure the services of Mayor Arnulfo Natividad of Tarlac for the death of Severino. Eventually
private counsel. the Prov. Prosec. found that there was probable cause so he filed an
information for murder before the RTC where Judge Felix was
When is a provincial legal officer disqualified to represent in court a presiding. Natividad moved to quash the warrant of arrest
particular municipality? subsequently issued and to remand the case for preliminary
1. When the original jurisdiction of case involving the investigation arguing that Judge Felix of RTC has no jurisdiction; that it
municipality is vested in the SC; was the Ombudsman and not the Prov Prosec who had jurisdiction to
2. When the municipality is a party adverse to the provincial conduct the preliminary investigation on the basis of Sec. 15(1) of RA
government to some other municipality or city in the same 6770; and that the proper court which has jurisdiction is the
province; Sandiganbayan, not the RTC. The RTC denied the motion, hence
3. When, in a case involving the municipality, the Provincial Natividad’s certiorari case before the SC.
Legal Officer, or his wife/ her husband, or child, is
pecuniarily involved, as heir legatee, creditor or otherwise SC held that Judge Felix of RTC did not committed grave abuse of
discretion in admitting the information filed by the provincial
When is a municipality allowed to hire a private counsel? prosecutor and in directing Natividad’s arrest as the RTC has
jurisdiction over the case.
Doctrine: A perusal of PD 1861, Sec. 1 which amends Sec. 4 (a)(2) shows
that 2 requirements must concur for an offense to fall under the
Sandiganbayan’s jurisdiction:
(1) The offense committed by the public officer must be in
relation to his office and
(2) The penalty prescribed be higher than prision correccional
or imprisonment of 6 years or a fine of P6,000

The SC cannot accept Natividad’s specious argument that the alleged


offense was committed in the performance of the mayor’s responsibility
to maintain peace and order on the pretext that the victim, a robbery
and NPA suspect, was under investigation at the time when he was
allegedly killed ta the police station. By no stretch of imagination or
logic, can we construe that the alleged act falls under any of the
functions of the municipal mayor in Sec. 444 LGC or implied therefrom.

Assuming arguendo that the act satisfied the requirement that the same
must be in performance of official functions, still it cannot be
overlooked that the OMB has only primary jurisdiction over cases
cognizable by the Sandiganbayan, NOT EXCLUSIVE ORIGINAL
jurisdiction.

MONTUERTO v. TY
October 6, 2008

Summary: Montuerto was appointed as a budget officer by the mayor


without obtaining the concurrence of the SB. After 10 years, SB issued a
resolution for the revocation of Montuerto’s appointment. SB also sent
to the CSC 201 file of Montuerto to show that the SB did not concur to
her appointment. Montuerto, on her part, send to SB a joint affidavit of
the majority of the Sanggunian showing that her appointment was
verbally concurred in in an SB Session. SC ruled that the law is clear and
that under Section 443(a) and (d) of the Local Government Code, the
head of a department or office in the municipal government, such as the
Municipal Budget Officer, shall be appointed by the mayor with the
concurrence of the majority of all Sangguniang Bayan members subject
to civil service law, rules and regulations.

Doctrine: Verbal concurrence allegedly given by the Sanggunian is not


the concurrence required under R.A. No. 7160. TheSanggunian, as a
body, acts through a resolution or an ordinance. Absent such resolution
of concurrence, the appointment of petitioner failed to comply with the
mandatory requirement of Section 443(a) and (d) of R.A. No. 7160.
 Coverage: Suits FOR THE RECOVERY OF FUNDS AND
THE CITY PROPERTY on behalf of the city, even without prior
authorization from the Sanggunian
See Sec. 448-458
4. Salary Grade: SG 30
The City, like the municipality, is considered a general-purpose
government for the delivery of basic, regular, and direct services to and The Sangguniang Panlungsod (Sec. 457-458)
effective governance of its inhabitants 1. Composition:
a. City Vice Mayor as presiding officer
NOTE: RA 9009 increased the income requirement for municipalities b. Regular members
wanting to become cities to P100,000,000 for the last 2 consecutive c. President of the City Chapter of the Liga ng mga
years based on 2000 constant prices Barangay
d. President of the City Pederasyon ng mga SK
Classification of Cities (Sec. 451) e. Three sectoral representatives:
1. Highly Urbanized Cities – Cities proclaimed as such by the (1) Women’s
President upon compliance with the income and population (2) Labor
requirements in Sec. 452 (3) Urban poor or the indigenous cultural
a. Population = At least 200,000 communities or the disabled,, or any other sector
b. Annual income = At least P50M based on 1991 Constant
prices15 2. NOTE: ALL Sanggunian members are elected

2. Independent Component Cities – Cities whose charters 3. Powers (Sec. 458)


prohibit their voters from voting for provincial elective a. As the legislative body of the City it has the power to
officials enact ordinances, approve resolutions, and appropriate
funds for the general welfare of the city and its
3. Component Cities – Cities that allow their voters to inhabitants
participate in the election of provincial officials and whose b. Essentially the same powers as those exercised by the
annual income does not qualify them to be highly urbanized Sangguniang Bayan

City Officials (Sec. 454) 4. Salary Grade:


A. City Mandatory Officials a. SG 25 for component cities
1. City Mayor b. SG 27 for highly urbanized cities
2. City Vice Mayor
3. Sangguniang Panlungsod members
4. City Secretary CANET v. DECENA
5. City Treasurer January 20, 2004
6. City Assessor
7. City Accountant Summary: The Sangguniang Bayan of Bula, Camarines Sur issued a
8. City Budget Officer Resolution authorizing Canet to open and operate a cockpit. Canet
9. City Planning and Development Coordinator applied for a mayor’s permit for this purpose. However, the Mayor did
10. City Engineer not grant it because according to her, a Municipal Ordinance is required
11. City Health Officer before she could grant it. RTC: Ruled for Canet. CA/ SC: Overturned the
12. City Civil Registrar RTC, an ordinance is required for the Mayor to be compelled to issue a
13. City Administrator permit.
14. City Legal Officer
15. City General Services Officer Doctrine: As there is no ordinance allowing the operation of a cockpit,
Resolution 049 authorizing Canet to open and maintain a cockpit
B. City Optional Officials – may be appointed by the Mayor cannot be implemented. To compel the Mayor to issue a Mayor’s permit
1. City Architect without the justification of a municipal Ordinance is a violation of Sec.
2. City Information Officer 447 of the LGC (1991) and is an undue encroachment on the Mayor’s
3. City Agriculturalist administrative prerogatives.
4. City Population Officer
5. City Environment and Natural Resources Officer
6. City Cooperatives Officer AGUIRRE v. DE CASTRO
December 17, 1999
The Sanggunian has the power to maintain existing offices although not
enumerated under Sec. 454; create others as may be necessary; or Summary: De Castro, the Chief of the Legal Affairs and Complaint
consolidate the functions of any office for efficiency and economy Services of the Division of City Schools of Manila, was asked by the City
Legal Officer to appear before it for the purpose of conduction of a
The City Mayor (Sec. 455) formal investigation pursuant to a charge of grave misconduct against
1. The chief executive of the city; exercises such powers and her. De Castro filed a MTD alleging that she reported to the Sec of DECS
performs such duties and functions as the LGC and other and not the City Legal Officer. City legal officer denied her motion. CA
pertinent laws provide and SC reversed, it is the sec of DECS that has power to discipline

2. Powers, duties, and functions are essentially the same as Doctrine: SEC. 454. Officials of the City Government. --- (a) There shall
those of the municipal mayor be in each city a mayor, a vice-mayor, sangguniang panlungsod
members, a secretary to the sangguniang panlungsod, a city treasurer, a
3. Mayor’s authority to file suits city assessor, a city accountant, a city budget officer, a city planning and
 Authorized by Sec. 455(b)(3)(viii); same with the development coordinator, a city engineer, a city health officer, a city
Municipal Mayor and Provincial Governor civil registrar, a city administrator, a city legal officer, a city
veterinarian, a city social welfare and development officer, and a city
15 general services officer. (b) In addition thereto, the city mayor may
Take note that this was not amended by RA 9009
appoint a city architect, a city information officer, a city agriculturist, a ACEBEDO OPTICAL COMPANY, INC. v CA
city population officer, a city environment and natural resources officer, March 31, 2000
and a city cooperatives officer. xxx
Summary: When the mayor of Iligan issued business permit to Acebedo
Under Book IV, Chapter V, Section 7(4) of the Administrative Code of optical, it was burdened with several conditions. Samahan ng mga
1987, the power to appoint and discipline first-level employees, which Optometrists sa Pilipinas filed a complaint before the Mayor that
include De Castro, is specifically lodged with the regional director of the Acebedo violated that said conditions. After investigating the matter,
Department of Education, Culture and Sports. the mayor cancelled Acebedo’s permit. Acebedo assails the mayor’s act
and ultimately challenges the validity of the conditions imposed. The SC
The source of the wages is not the only criteria in determining whether ruled in favor of Acebedo.
the payor may be deemed the employer. In fact, the most important
factor is the control test; that is, who has the power to supervise and Doctrine: A business permit is issued primarily to regulate the conduct
direct the work of the employee concerned? of business and the City Mayor cannot, through the issuance of such
permit, regulate the practice of a profession, like that of optometry.
Class Notes: Take note of the CONTROL TEST Such a function is within the exclusive domain of the administrative
agency specifically empowered by law to supervise the profession

GORDON v. VERIDIANO Class Notes:


November 8, 1988 - Distinction must be made between grant of license or permit
to do business and issuance of license to engage in the
Summary: The San Sebastian Drugstore and Olongapo City Drugstores practice of profession
in Olongapo City were both owned by Rosalinda Yambao. They were 1. License to do business – granted by local authorities
issued Mayor’s permits by the mayor and licenses to operate by the a. Authorizes the person (natural or otherwise) to
FDA. In a “test buy” conducted by agents of FDA and the PC, 200 tablets engage in business or some form of commercial
of valium were sold by the San Sebastian Drug Store without activity
prescription. Mayor Gordon issued a letter summarily revoking its 2. License to practice - Issued by the Board or Commission
mayor’s permit, effective the next day, for rampant violation of the tasked to regulate the particular profession
Pharmacy Law and the Dangerous Drugs Act. After hearing on the same a. Grant of authority to a natural person to engage in
report, FDA Administrator Regala directed the closure of the drug store the practice of profession
for three days and payment of a fine of P100 for violation of RA 3720. A
stern warning was given to Yambao. After three days, the FDA lifted the
suspension order. Yambao then asked for the reconsideration of the LIM v. CA
mayor’s revocation. When she did not receive any reply, she filed for August 12, 2002
mandamus and damages with the RTC. On the dame day, she asked
permission from the FDA to allow the two drug stores to exchange Summary: Bistro filed a petition for prohibition and mandamus, with
locations for reasons of “business preference.” prayer for TRO or writ of preliminary injunction against Mayor Lim for
the latter’s acts, specifically the disruption of the operations of his
The FDA granted this request. When Mayor Gordon was informed of nightclubs. The TC ruled in favor of Bistro, as did the CA. The SC, in
this, he sent a letter disapproving the transfer and suspending upholding the decision of the CA, said that, while the mayor has power
indefinitely the mayor’s permit of Olongapo City Drugstore. Mayor to inspect and investigate private commercial establishments for any
Gordon wrote the FDA to reconsider its decision allowing the operation violation of the conditions in their licenses and permits, he has no
of San Sebastian Drugstore. This was denied. After hearing, the RTC power to order a police raid in the guise of inspection.
ruled that the revocation of the mayor’s permit of the San Sebastian
Drugstore is null and void. It enjoined Mayor Gordon from any act Doctrine: The power to issue business licenses and permits necessarily
towards the closure of both drug stores. On certiorari, the SC ruled that include power to suspend, revoke, or even refuse to issue the same.
it is the FDA that has authority to decide on the violation of pharmacy HOWEVER, such power is premised on violation of the conditions of
and drug laws, but it is the mayor who was authority to suspend for these permits and licenses.
violation of its mayor’s permit because of the transfer of sites.
While mayor has power to inspect and investigate private commercial
Doctrine: The power to approve a license includes by implication the establishments for any violation of the conditions in their licenses and
power to revoke it. By extension, the power to revoke is limited by the permits, he has no power to order a police raid in the guise of
authority to grant the license, from which it is derived in the first place. inspection.

FDA has jurisdiction to suspend licenses for violations of national law; Class Notes: Sir does not believe that it should be a blanket prohibition
the local executive has jurisdiction to suspend permits for violations of on raids.
ordinances. The FDA would have the right to disapprove the site of a
drug store only if it would impair the health or other interests of the
customers in contravention of the national laws or policies. However, MATHAY v. FELT FOODS, INC.
the local executive would have reasons to object to the location, even if July 28, 1999
approved by the FDA, where it does not conform to ordinances
intended to promote the comfort and convenience of the city residents. Summary: Mathay issued a temporary business permit in favor of FFI.
This permit was subsequently cancelled after finding that violations of
By revoking the mayor’s permit on the same ground for which the San its terms were committed by FFI. Consequently, a closure order was
Sebastian Drug Store had already been penalized by the FDA, the mayor issued against FFI. Subsequently, the permit expired. FFI filed an action
was in effect reversing the decision of the FDA on a matter that came seeking the annulment of the closure order and for a writ of prelim
under its jurisdiction. mandatory injunction. The SC invalidated the issuance of the writ on
the ground that the expiration of the permit had mooted the cause of
Class Notes: Can the mayor require compliance with ALL national laws action of FFI. There was no more legal right to protect since the permit
and regulations before issuing the permit? had already expired by the time the action was filed by FFI.
- YES, the presence of this clause does not invalidate the permit.
However, the finding of violation of national laws must be
made by the national agency, not the mayor
Doctrine: Upon the expiration of FFI’s business permit, it no longer had Doctrine: Administrative circulars cannot go beyond the law they seek
any legal right to do business which could have been the proper subject to implement. A limit cannot be imposed through an administrative
of a judicial relief. circular when the law it is seeking to implement does not provide for
such (own words)

NEGROS ORIENTAL II ELECTRIC COOP v. SANGGUNIANG The DBM has 90 days from the receipt of the copies of the
PANLUNGSOD OF DUMAGUETE appropriation ordinance of cities to review the same. Otherwise, such
November 5, 1987 ordinance was deemed to have been properly reviewed and deemed to
have taken effect. After such period, the legality of the provisions in the
Summary: The members board of directors were being summoned by said ordinance can no longer be questioned.
the Sangguniang Panglungsod ng Dumaguete. They contested the
validity issued by the subpoena and the court ruled that it was beyond
the powers of the SP to issue it. CITY OF CALOOCAN v. CA
May 3, 2006
Doctrine: The exercise by the legislature of the contempt power is a
matter of self-preservation as that branch of the government vested Summary: The Sangguniang Panlungsod of Caloocan City passed an
with the legislative power, independently of the judicial branch, asserts ordinance authorizing the city mayor to negotiate and enter into a
its authority and punishes contempts thereof. The contempt power of contract of sale of land owned by the city. Then Mayor Asistio and Jose
the legislature is, therefore, sui generis, and local legislative bodies C. Go of Ever Gotesco, executed a Deed of Absolute Sale. COA approved
cannot correctly claim to possess it for the same reasons that the the sale on the condition that the price be increased. The Sanggunian
national legislature does. The power attaches not to the discharge of passed an ordinance directing that an amended deed of absolute sale be
legislative functions per se but to the character of the legislature as one executed. Gotesco consented. (New) Mayor Malonzo received the
of the three independent and coordinate branches of government. The documents but refused to sign the amended deed of sale and accept
same thing cannot be said of local legislative bodies which are creations payments. Thus Gotesco filed Civil Case No. C-18274, seeking the
of law. consignation of the purchase price and tax payments.

Absent a constitutional or legal provision for the exercise of these Meanwhile, TCT NO. 326321 covering the property was issued in
powers, the only possible justification for the issuance of a subpoena Gotesco’s name. Caloocan City, through Mayor Malonzo, filed a petition
and for the punishment of non-members for contumacious behaviour for prohibition with application for preliminary injunction and prayer
would be for said power to be deemed implied in the statutory grant of for the issuance of a TRO, docketed as Civil Case No. C-18308. During
delegated legislative power. But, the contempt power and the subpoena the pendency of this suit, another case, docketed as Civil Case No. C-
power partake of a judicial nature. They cannot be implied in the grant 18337, was filed for annulment of sale and cancellation of title.
of legislative power. Neither can they exist as mere incidents of the
performance of legislative functions. To allow local legislative bodies or Gotesco moved for the dismissal of Civil Case No. 18337, but this was
administrative agencies to exercise these powers without express denied. They brought the issue to CA. CA dismissed the case on the
statutory basis would run afoul of the doctrine of separation of powers. ground of litis pendentia, and because of a defect in the certification
against forum-shopping, for not having been signed by Mayor Malonzo,
Class Notes: Good law? the plaintiff or principal party under oath, but instead by the City Legal
- Yes, the LGC did not grant it the power to issue subpoena or to Officer, who was only the counsel, and not a party to the case. SC agreed
cite in contempt with CA.
 But can they be granted by law of these powers? Yes,
but no law so far. Doctrine: The mayor has the authority to file suits "for the recovery of
- The only provision in LGC which makes reference to contempt funds and property" on behalf of the city, even without the prior
power is found in Sec. 515 – Refusal or Failure of Any Party or authorization from the Sanggunian. Being the proper party to file such
Witness to Appear before the Lupon or Pangkat. suits, the mayor must necessarily be the one to sign the certification
 Refusal or willful failure of any party or witness to against forum-shopping, and not the City Legal Officer, who was merely
appear in compliance with a summons issued may be its counsel and not a party to the case.
punished by the city or municipal court as for indirect
contempt of court upon application filed therewith by
the lupon chairman, the pangkat chairman, or by any of SOCIAL JUSTICE SOCIETY v. ATIENZA
the contending parties. March 7, 2007
 NOTE: Still, it is the court which cites the person in
contempt Summary: The Sanggunian Panlungsod of Manila enacted an Ordinance
which directed owners and operators of businesses in the Pandacan –
Sta. Ana area to cease and desist from operating their businesses. One
DADOLE v. COA of the affected businesses was the “Pandacan Terminals” of Caltex,
December 3, 2002 Petron, and Shell. However, later on, the City and the DOE entered into a
Memorandum of Understanding, under which, the oil companies were
Summary: Sangguniang Panglungsod of Mandaue City enacted an just to scale down their operations, instead of shutting down.
ordinance that granted additional monthly allowance of 1500 for judges Resolutions were enacted to ratify the MOU, and to extend the validity
assigned in their locality. However, DBM issued LBC 55 imposing 1000 of the MOU. SJS filed a petition for mandamus, that City Mayor Atienza
as the maximum limit for such allowances. The City Auditor be compelled to enforce the Ordinance. The SC found for SJS and
subsequently disallowed the allowances for the judges and asked them directed Mayor Atienza to immediately enforce the ordinance.
to reimburse the amount in excess of 1000. SC reversed and ruled that
DBM exceeded its supervisory power in imposing a limit not existing on Doctrine: The LGC imposes upon city mayors the duty to “enforce all
the LGC provision it claimed to interpret. Section 458, par. (a)(1)(xi), of laws and ordinance relative to the governance of the city.” Mayors have
RA 7160, the law that supposedly serves as the legal basis of LBC 55, the ministerial duty to enforce ordinances as long as they have not been
allows the grant of additional allowances to judges “when the finances repealed by the Sanggunian, or annulled by the courts. They cannot
of the city government allow.” The said provision does not authorize refuse to perform their duties of enforcement on the ground of an
setting a definite maximum limit to the additional allowances granted alleged invalidity of the statute imposing the duty of enforcement.
to judges. The budget circular is also void for not being published.
2. Powers are the same as the Sangguniang Bayan and
THE PROVINCE Panlungsod

See Sec. 459-468 3. Salary Grade: SG 27

The Province is a political and corporate unit of government like the


city and municipality, it serves as a mechanism for development CARAM v. COMELEC
processes and effective governance of the component cities, August 30, 1993
municipalities, and barangays within its territorial jurisdiction
Summary: Petition for Mandamus was filed by Caram and Laban ng
The Provincial Officials (Sec. 463) Demokratikong Pilipino for COMELEC to conduct special elections for
position of Governor, Vice-Governor, Members of the Sangguniang
A. Provincial Mandatory Officials Panlalawigan representing the 2nd District of the Province of Iloilo and
1. Governor the Congressman for the 2nd Congressional District of said province.
2. Vice Governor They asked for TRO to prevent BOC of Iloilo from canvassing and
3. Members of the Sangguniang Panlalawigan proclaiming the results of the elections since COMELEC had no
4. Sanggunian Secretary authority to exclude the voters of Guimaras to vote for provincial
5. Provincial treasurer officials. It was also alleged that COMELEC failed to properly
6. Provincial assessor disseminate information regarding the right of Guimaras to vote a for a
7. Provincial accountant Congressman for the 2nd Congressional District of Iloilo with the result
8. Provincial engineer that almost one-third of the registered voters of the sub-province were
9. Provincial budget officer not able to exercise the right to vote. SC dismissed the petition.
10. Provincial administrator
11. Provincial health officer Doctrine: Under Section 462, the new legislative districts created as a
12. Provincial social welfare and development officer result of such conversion shall continue to be represented in Congress
13. Provincial general services officer by the duly-elected representatives of the original districts out of which
14. Provincial agriculturalist said new provinces or districts were created until their own
15. Provincial veterinarian representatives shall have been elected in the next regular
congressional elections and qualified.
B. Provincial Optional Officials – may be appointed by governor
subject to approval by Sanggunian In Griño v. Commission on Elections, COMELEC had no authority to
1. Provincial population officer exclude the voters of the Sub-province of Guimaras from voting for the
2. Provincial natural resources and environmental officer positions of the Governor, Vice-Governor, and Members of the
3. Provincial cooperatives officer Sangguniang Panlalawigan representing the Second District of the
4. Provincial architect Province of Iloilo because under Section 462 of the Local Government
5. Provincial information officer Code, it was only in the case where the voters ratified the conversion of
the Sub-province of Guimaras to a regular province that the President
The Sanggunian has the power to maintain existing offices although not was empowered to appoint the officials of the newly created province.
enumerated under Sec. 463; create others as may be necessary; or
consolidate the functions of any office for efficiency and economy The Court noted that the lawmakers failed to foresee that in the event
the negative vote in the plebiscite prevailed, the Sub-province of
NOTE: The governor is required under Sec. 464 to reside officially in the Guimaras would continue to be a part of the Province of Iloilo and be
provincial capital represented by the officials of the province elected in the May 11, 1992
b. All elective and appointive officials of the province elections. However, considering that the "Yes" votes in the plebiscite
are required to hold office in the provincial capital prevailed, SC ruled that it would serve no useful purpose to undo what
the COMELEC had done and that there would be no legal basis to call
The Provincial Governor (Sec. 465) special elections to give a chance to the voters of the Sub-province of
1. As the chief executive of the province, he exercises such Guimaras to vote for the provincial officials of the Provinces of Iloilo.
powers and performs such duties and functions as the LGC
and other pertinent laws provide
PROVINCE OF NEGROS OCCIDENTAL v. ZAYCO
2. Powers and duties are similar to those exercised by the September 28, 2010
municipal or city mayor with minor exceptions
Summary: The Sangguniang Panlalawigan of the Province of Negros
3. Salary Grade: Minimum of SG 30 Occidental passed a resolution allocating P4,000,000 for the
hospitalization and health care insurance benefits of 1,949 officials and
The Sangguniang Panlalawigan (Sec. 467-468) employees of the province. Thereafter, the Province and Philam Care
1. Composition entered into an Agreement involving a payment of P3,760,000,
a. Vice Governor as presiding officer representing the insurance premiums of the former's officials and
b. Regular members employees.
c. President of Provincial Chapter of the Liga ng mga
Barangay However, the Provincial Auditor suspended the payment of premiums
d. President of the Provincial Pederasyon ng mga SK because of lack of approval from the Office of the President as provided
e. President of the Provincial Federation of the under Administrative Order No. 103 (AO 103). The Provincial Auditor
Sanggunian Members of Municipalities and Component also explained that the premium payment for health care benefits
Cities violated RA 6758 or the Salary Standardization Law.
f. Three sectoral representatives:
(1) Women’s President Joseph E. Estrada directed the Commission on Audit (COA) to
(2) Labor lift the suspension but only in the amount of P100,000.
(3) Urban poor or indigenous cultural communities or
the disabled, or other sector
However, the Provincial Auditor ignored the directive of the President
and instead issued a Notice of Disallowance of the payment of
premiums.

The Province appealed the disallowance to the COA, which affirmed the
Provincial Auditor’s Notice of Disallowance.
The COA ruled that under AO 103, no government entity, including a
local government unit, is exempt from securing prior approval from the
President granting additional benefits to its personnel. This is in
conformity with the policy of standardization of compensation laid
down in RA 6758.

The COA added that Section 468(a)(1)(viii)11 of RA 7160 relied upon


by the Province does not stand on its own but has to be harmonized
with Section 1212 of RA 6758.

The issue is whether the COA erred in affirming the disallowance of


P3,760,000 for premium paid for the hospitalization and health care
insurance benefits granted by the Province of Negros Occidental to its
1,949 officials and employees. The SC ruled that it erred.

DOCTRINE: On how the President's power of general supervision over


LGU's does not include the power to approve grants of additional
compensation
Reading AO 103, it can be seen that the requirement of prior approval
from the President under AO 103 is applicable only to departments,
bureaus, offices and government-owned and controlled corporations
under the Executive branch. In other words, AO 103 must be observed
by government offices under the President’s control as mandated by
Section 17, Article VII of the Constitution.

However, EVEN IF AO 103 required LGU's to acquire prior approval


from the President, it would be violative of Section 4, Article X of the
Constitution which provides that the President shall exercise only
general supervision over local government units.

Since LGUs are subject only to the power of general supervision of the
President, the President’s authority is limited to seeing to it that rules
are followed and laws are faithfully executed. The President may only
point out that rules have not been followed but the President cannot lay
down the rules, neither does he have the discretion to modify or replace
the rules.

Thus, the grant of additional compensation like hospitalization and


health care insurance benefits in the present case does not need the
approval of the President to be valid.

On whether LGU's are bound by a national law, the Salary


Standardization law: Also, while it is true that LGUs are still bound by
RA 6758, the COA did not clearly establish that the medical care
benefits given by the government at the time the Resolution was passed
by the Sangguniang Panlalawigan were SUFFICIENT to cover the needs
of government employees especially those employed by LGUs.

It must be pointed out that the Civil Service Commission, through CSC
MC No. 33, as well as the President, through AO 402, already recognized
the deficiency of the state of health care and medical services
implemented at the time the Resolution was passed.
APPOINTIVE LOCAL OFFICIALS
COMMON TO ALL MUNICIPALITIES, CITIES, PROVINCES
Under this Title of the LGC, Lists 22 officials common to all municipality, Qualifications – They are practically the same for every official:
city, and province: 1. Citizens of the Republic
2. Residents of the LGU concerned
APPOINTING MANDATORY/ 3. Good moral character
OFFICIAL 4. Holders of first grade civil service eligibility or its equivalent
AUTHORITY OPTIONAL
5. College degree holders, preferable with specialized studies
Assessor Local chief Executive Mandatory relevant to the work of the offices concerned and for some
Assistant Assessor Local chief Executive Optional offices, specific disciplines demanded by their work
Accountant Local chief Executive Mandatory 6. Work experience related to the character of the office
a. EXCEPT for the secretary of sanggunian where no work
Budget Officer Local chief Executive Mandatory
experience is required at all
Planning and
development Local chief Executive Mandatory
coordinator RAPISORA v. CSC
Engineer Local chief Executive Mandatory December 17, 1993
Health officer Local chief Executive Mandatory
Summary: Rapisora applied for the position of Prov. Health Officer I of
Civil registrar Local chief Executive Mandatory Benguet. The Sec. of Health designated him as the OIC of said position
Administrator Local chief Executive Mandatory and was later on extended a permanent appointment. He actually
Mandatory for cities assumed the duties of the position on Jan. 2, 1992, the day the LGC 1991
and provinces took effect. Pursuant to the LGC, the CSC returned Rapisora’s
Legal officer Local chief Executive appointment to the SOH. The SOH sought the concurrence of the
Optional for
Benguet Governor for the appointment of Rapisora. The Governor gave
municipalities such concurrence. Despite this, the CSC Field Officer only approved
Agriculturalist Local chief Executive Mandatory Rapisora’s appointment as TEMPORARY. Rapisora appealed such
Mandatory for cities decision to the CSC. However, the CSC entirely disapproved his
Social welfare and appointment for failure to possess the educational requirement for the
and provinces
development Local chief Executive position of PHO I (he did not possess a Certificate in Public
Optional for
officer Health/Hospital Administration). Rapisora appealed to the SC. The SC
municipalities ruled in favour of Rapisora and upheld his permanent appointment. It
Environment and pointed out the fact that the Secretaries of Health appreciated the
natural resources Local chief Executive Optional OTHER qualifications of Rapisora. In effect, his deficiency in educational
officer requirement was offset by his training and experience.
Architect Local chief Executive Optional
Doctrine: CSC is not empowered to determine or change the kind of
Information nature of the appointment for it is an essentially DISCRETIONARY
Local chief Executive Optional
officer power and must be performed by the officer on whom it is vested
Cooperatives according to his best lights, the only condition being that the appointee
Local chief Executive Optional should possess the minimum qualification required by law.
officer
Population officer Local chief Executive Optional
When necessary, education, experience or training may be used
Mandatory for cities interchangeably to OFFSET deficiencies.
and provinces
Veterinarian Local chief Executive
Optional for Class Notes: Is the rule on offsetting of qualifications still good law?
municipalities - At least for appointive officials, yes.
Mandatory for cities - However, this will not apply to Constitutional requirements
General services and provinces
Local chief Executive
officer Optional for DBM v. LEONES
municipalities March 18, 2010
Treasurer Finance Secretary Mandatory
Assistant Summary: A municipal treasurer, Leones, was reassigned to the Office of
Finance Secretary Optional the Provincial Treasurer, pending resolution of administrative cases
Treasurer
filed against her. Upon her reassignment, the representation and
Sanggunian Vice Mayor or Vice transportation allowance (RATA) she received as municipal treasurer
Mandatory
Secretary Governor was stopped prompting her to file a suit for mandamus against DBM to
compel the payment of her RATA. DBM argues that her RATA is
Notes: conditioned upon the actual performance of her duties as municipal
c. All are career officials except the Legal Officer, treasurer.
Information Officer, and Administrator whose
terms of office are coterminous with the local chief The SC ruled that upon reassignment, Leones performed duties and
executive who appointed them responsibilities "comparable" to her previous position. Hence, she is
d. All are appointed by the local chief executive entitled to RATA.
EXCEPT for the treasurer, assistant treasurer,
sanggunian secretary
Doctrine: The GAAs do not apply to a local government official like City Accountant
Leones whose compensation and allowances are funded by local Under Section 474, LGC, the accountant is charged with both accounting
appropriation laws passed by the Sangguniang Bayan . It is the and internal audit services of the LGU. She is tasked to maintain the
municipal ordinances, providing for the annual budget for its operation, internal audit system of the LGU and is presumed to know the COA
that apply to her receipt of RATA. rules on granting cash advance, which were consistently not observed.

Thus, irrespective of the level of the LGU involved, no distinction exists City Administrator
in the functions of local treasurers (whether municipal, provincial, etc) Arias v. SB, recognized the impossibility for a head of office to
except in the technical supervision by the provincial treasurer over personally probe all records in his office and that he has to rely, to a
subordinate treasury offices. Logically, the employees in all local reasonable extent, on the good faith of his subordinates. As such, it was
treasuries perform comparable functions. held that conviction of such head of office for conspiracy under RA 3019
must be supported by other grounds than the mere signature or
approval in the voucher.
LA CARLOTA CITY v. ROJO
April 24, 2012 In Sistoza v. Desierto, it was also held that “reliance in good faith by a
head of office on a subordinate upon whom the primary responsibility
Summary: Rojo, a member of the Sangguniang Panlungsod of La Carlota rests negates an imputation of conspiracy by gross inexcusable
City, resigned from such position during a regular session of such negligence to commit graft and corruption.”
Sangguinian where 7 of the 13 members were present. A day after, then
Vice Mayor Jalondoon appointed Rojo as Sangguniang Panlungsod In this case, Gaviola was head of an office independent from the offices
Secretary. After the 2004 elections, the new Mayor and Vice Mayor of his co-accused, i.e., they are not his subordinates. As such, the
assailed the appointment of Rojo, arguing that Rojo could not have invocation of Arias is “shaky.”
validly resigned since there was no quorum of the Sanggunian because However, since Gaviola, as administrator, is tasked only with the overall
the Vice Mayor should not be counted as a member of the Sanggunian administration of the LGU and given the pre-audit process and the
for purposes of determining the existence of a quorum. certification of his co-accused before the voucher reaches him, the
deficiency in the particulars of payment (e.g., purpose, payee) is not
Doctrine: RA 7160 clearly states that the Sangguniang Panlungsod “ enough to charge him with gross and inexcusable neglect.
shall be composed of the city vice-mayor as presiding officer, the
regular sanggunian members, the president of the city chapter of
the liga ng mga barangay, the president of the
panlungsod na pederasyon ng mga sangguniang kabataan, and
the sectoral representatives, as members.”

As the presiding officer, the vice-mayor can vote only to break a tie. In
effect, the presiding officer votes when it matters the most, that is, to
break a deadlock in the votes. To construe otherwise would create an
anomalous and absurd situation where the presiding officer who votes
to break a tie during a Sanggunian session is not considered a
“member” of the Sanggunian. In stating that there were 14 members of
the Sanggunian, the SC in Zamora clearly included the Vice-Governor, as
presiding officer, as part of the entire membership of
the Sangguniang Panlalawigan which must be taken into account in
computing the quorum.

JACA v. PEOPLE
January 28, 2013

Summary: City Accountant Jaca, Administrator Gaviola, and Treasurer


Cesa were charged with violating Section 3(e), RA 3019 for facilitating
the malversation of P18.5M by Badana. Jaca, et al., were found, through
a surprise audit, to have failed to comply with the COA rules in the grant
of cash advances to Badana. They granted additional cash advances
when there are still unliquidated advances in the custody of Badana,
thereby facilitating the latter’s accumulation of funds. The SB found
them guilty. The SC affirmed, ruling, among others, that their defense of
good faith is unpersuasive in the face of clear violation of the law.

Doctrine: [No relevant doctrinal statement, only that the officials


involved are appointive officials. Maybe read the LGC provisions on the
duties of said appointive officials.]

City Treasurer
He certified the cash advances as necessary and lawful, when, under
Section 89, PD 1445, no additional cash advance is allowed unless the
previous cash advance has been settled.

Section 339, LGC, also mandates that cash advance should be granted in
accordance with COA rules.
LEAGUES OF LGUs AND ELECTIVE OFFICIALS
See Sec. 491 - 510 MIGUEL v. CA
February 23, 1994
The Five Leagues
1. Liga ng mga Barangay Summary: Roberto A. Miguel was elected President of KPB-QC, whch
2. League of Municipalities was created under the old LGC. Afterwards, the new LGC took effect,
3. League of Cities abolishing the Pambansang Katipunan ng mga Barangay and replacing
4. League of Provinces it with the Ligas. Pending elections to be held under the LGC of 1991,
5. League of Metropolitan Political Subdivisions Miguel continued to serve as president of Liga-QC. However, in spite of
a. E.g. the Metro Manila League of Mayors Miguel’s objections, an ad hoc committee elected a new set of officers,
including a new president, under the old constitution and by-laws of the
Primary Purpose of the Leagues – To ventilate, articulate, and KPB-QC. The SC, in ruling that the election was invalid, said that,
crystallize issues affecting local government administration and to according to the RRI of the LGC of 1991, a uniform constitution and by-
secure solutions thereto, laws must first be adopted prior to the holding of any elections.

Officers – Every liga directly elects a president, vice-president and 5 Doctrine: Article 210 (f) (1) of Rule XXIX of the RRI: " [p]ending election
members of the Board of Directors of the presidents of the municipal, city, provincial, and metropolitan
chapters of the Liga, the incumbent presidents of the association of the
Ex-Officio Sanggunian Members – The presidents of the liga at the barangay councils in the municipality, city, province and Metropolitan
municipal, city, and provincial levels, automatically become ex-officio Manila shall continue to act as presidents of the corresponding liga
members of the Sangguniang Bayan, Panlungsod, and Panlalawigan chapters"
respectively
Article 210 (d) (3) of Rule XXIX: "incumbent presidents of the
municipal, city, and provincial chapters of the liga shall continue to
serve as ex officio members of the sanggunian concerned until the
GALAROSA v. VALENCIA expiration of their term of office, unless sooner removed for cause."
November 11, 1993
The election of the first set of officers of the national and local chapters
Summary: Raul Galarosa is the incumbent president of the katipunang of the Liga cannot be held unless a constitution and by-laws for the Liga
bayan or Association of Barangay Councils (ABC) of the municipality of is first adopted and ratified by the barangay national assembly (Rule
Sorsogon, province of Sorsogon, and was appointed as a member of the XXIX, Art. 210 [g] [3]). The said constitution and by-laws was to be
Sangguniang Bayan of Sorsogon. Rodolfo Lasay, an incumbent barangay drafted by the committee created under Article 210 (f) (2) of the RRI.
captain and an aspirant for the position of the president of ABC filed a
petition for declaratory relief with the RTC, seeking to clarify whether VIOLA v. ALUNAN III
or not the President of ABC can continue holding office despite the August 15, 1997
termination of the Terms of Office of the Sangguniang Bayan of
Sorsogon on June 30, 1992. The RTC ruled against Galarosa. The SC Summary: Cesar Viola filed a petition for prohibition challenging the
ruled in favor of Galarosa, allowing the later to continue to serve by validity of Art. III, Sec. 1-2 of the Revised Implementing Rules and
virtue of the hold-over doctrine. Guidelines for the General Elections of the Liga ng mga Barangay
Officers insofar as they provide for the election of first, second, and
Doctrine: Unless holding over be expressly or impliedly prohibited, the third vice presidents and for auditors for the National Liga ng mga
incumbent may continue to hold over until someone else is elected and Barangay and its chapters. SC upheld the validity of the Implementing
qualified to assume the office. This rule is demanded by the most Rules
obvious must requirements of public policy, for without it there must
frequently be cases where, from a failure to elect or a refusal or neglect Doctrine: Contrary to petitioner’s contention, the creation of the
to qualify, the office would be vacant and the public service entirely additional positions is authorized by the LGC which provides as follows:
suspended. Section 494 of the Local Government Code could not have
been intended to allow a gap in the representation of the barangays, §493. Organization. The liga at the municipal, city, provincial,
through the presidents of the ABC, in the sanggunian. metropolitan political subdivision, and national levels directly elect a
president, a vice-president, and five (5) members of the board of
Since the term of office of the punong barangays elected in the 28 March directors. The board shall appoint its secretary and treasurer and create
1989 election and the term of office of the presidents of the ABC have such other positions as it may deem necessary for the management of the
not yet expired and taking into account the special role conferred upon chapter. A secretary-general shall be elected from among the members
and the broader powers and functions invested in the barangays by the of the national liga and shall be charged with the overall operation of
Code as a basic political unit, a primary planning and implementing unit the liga on the national level. The board shall coordinate the activities of
of government policies in the community, and as forum wherein the the chapters of the liga. (emphasis added)
collective views of the people may be expressed and considered and
where disputes may be amicably settled, it would be in harmony with This provision in fact requires ¾ and not merely authorizes ¾ the
sound logic to infer that the Code never intended to deprive the board of directors to “create such other positions as it may deem
barangays of their representation in the sangguniang bayan during the necessary for the management of the chapter” and belies petitioner’s
interregnum when the liga has yet to be formally organized with the claim that said provision limits the officers of a chapter to the president,
election of its officers. vice president, five members of the board of directors, secretary, and
treasurer.

Congress can delegate the power to create positions such as these. The
said provision embodies a fairly intelligible standard: “deemed
necessary for the management of the chapters.” There is no undue
delegation of power by the Congress.
supervision of the President. Hence, also of the DILG Secretary as the
BITO-ONON v. FERNANDEZ alter ego of the President. However, DILG, in this case, went beyond
January 31, 2001 supervision and exercised control in its actions and issuance of
memorandum circulars.
Summary: Bito-onon was proclaimed winner as Exec. VP of the Liga ng
mga Barangay Provincial Chapter of Palawan. Quijano filed post- Doctrine: The President’s power of the general supervision, as exercised
proclamation protest with the Board of Election Supervisors (BES) by the DILG Secretary as his alter ego, extends to the Liga ng mga
which decided against him. Quijano filed a petition for review with the Barangay.
RTC. Bito-Onon filed a motion to dismiss claiming that the RTC had no
jurisdiction to review the Board’s decision in any post-proclamation
protest.

SC ruled that Memorandum Circular No. 97-193 of the DILG insofar as it


authorizes the filing a Petition for Review of the decision of the BES
with the regular courts in a post proclamation electoral protest is of
doubtful constitutionality.

Doctrine: The president’s power of general supervision extends to the


liga ng mga barangay since this is not an LGU. As per DOJ Ruling, the
Liga ng mga Barangay is a government organization, being an
association, federation, or league or union created by law or by
authority of law whose members are other appointed or elected
government officials.

The ligas are primarily governed by the provisions of the Local


Government Code. However, their respective constitution and by-laws
shall govern all other matters affecting the internal organization of the
liga not otherwise provided for in the Local Government Code provided
that the constitution and by-laws shall be suppletory to the provisions
of Book III, Title VI of the Local Government Code and shall always
conform to the provisions of the Constitution and existing laws.

In authorizing the filing of the petition for review of the decision of the
BES with the regular courts, the DILG Secretary in effect amended and
modified the GUIDELINES promulgated by the National Liga Board and
adopted by the LIGA which provides that the decision of the BES shall
be subject to review by the National Liga Board.

The amendment of the GUIDELINES is more than an exercise of the


power of supervision but is an exercise of the power of control, which
the President does not have over the LIGA. Although the DILG is given
the power to prescribe rules, regulations and other issuances, the
Administrative Code limits its authority to merely “monitoring
compliance” by local government units of such issuances. To monitor
means “to watch, observe or check” and is compatible with the power
of supervision of the DILG Secretary over local governments, which is
limited to checking whether the local government unit concerned or the
officers thereof perform their duties as per statutory enactments.

Besides, any doubt as to the power of the DILG Secretary to interfere


with local affairs should be resolved in favor of the greater autonomy of
the local government.

The public respondent judge therefore committed grave abuse of


discretion amounting to lack or excess of jurisdiction in not dismissing
the respondent’s Petition for Review for failure to exhaust all
administrative remedies and for lack of jurisdiction.

NATIONAL LIGA NG MGA BARANGAY v. PAREDES


September 7, 2004

Summary: David, representing Liga ng mga Barangay, filed a case to


assail the decision of Judge Paredes appointing DILG as an interim
caretaker of the Liga. The problem started when Rayos contested the
Presidency of David in the Liga, alleging irregularities in the election.
Because of the chaos in the affairs of the Liga, DILG through its
Secretary deemed it wise to take over the management of the Liga. DILG
asked the court under Judge Paredes to appoint it as an interim
caretaker of the Liga. Judge Paredes’ decision appointing DILG as such
and various acts and memorandum circulars issued by DILG is being
assailed in this case. SC ruled that Liga ng mga barangay is under the
MISCELLANEOUS AND FINAL PROVISIONS

PROVISIONS FOR IMPLEMENTATION TRANSITORY PROVISIONS


See Sec. 511-520 See Sec. 526 - 533
e. These are penal provisions which have been
discussed in the relevant portions of the reviewer. Sec. 526 – Applicability of the LGC to LGUs in the Autonomous Regions
f. See next section
See Sec. 521 – 525
1. Sec. 521 – Mandatory Review Every 5 years Sec. 527 – Prior approval or clearance on regular and recurring
 Congress shall undertake a mandatory review of the transactions shall no longer be required after 6 months from LGC
LGC at least once every 5 years and as often as effectivity
necessary
 Primary objective: To provide for a more responsive Sec. 528 – Decentralization of requisite authority and power
and accountable local government structure g. Within 6 months from LGC effectivity, national
government shall effect the deconcentration of
2. Sec. 522 – Insurance Coverage requisite authority and power to the appropriate
 GSIS to establish and administer an appropriate system regional offices whose major functions are not
for insurance coverage for the following officers: devolved to LGUs
1. Punong barangay
2. Members of the sangguniang barangay Sec. 529 – Generally, LGU tax ordinances or revenue measures continue
3. Barangay secretary in force and effect UNLESS:
4. Barangay treasurer 1. Inconsistent with LGC
5. Members of the barangay tanod 2. Violative of LGC
3. Repealed by sanggunian concerned
3. Sec. 523 – Personnel Retirement and/or Benefits
 Official or employee of the national government or LGU Sec. 530 – Local Water Districts
separated from service as a result of reorganization h. They may separate from LWUA
effected under LGC shall, if entitled under the laws then
in force, receive retirement and other benefits accruing Sec. 531 – Debt Relief for LGUs
thereunder i. Basically, program loans secured by LGUs but
relent to private persons, natural or juridical, are
4. Sec. 524 - Inventory of Infrastructure and Other Community written off but the central government shall
Facilities continue to collect from private sector debtors
 Each LGU shall conduct a periodic inventory of
infrastructure and other community facilities and Sec. 532 – Elections for the SK
undertake maintenance, repair, construction, etc.
Sec. 533 – Formulation of the IRR
5. Sec. 525 – Records and Properties
 All records, equipment, buildings, facilities, and other
properties of any office or body of LGU abolished or
reorganized under LGC shall be transferred to the office
or body to which it is substantially devolved
ABBAS v. COMELEC
THE AUTONOMOUS REGION November 10, 1989

IN MUSLIM MINDANAO Summary: Petitioners Abbas et al. and Mama-o sought to enjoin the
COMELEC from conducting a plebiscite, in implementation of RA 6734 -
the law providing for an Organic Act for the ARMM, and prayed that
See Sec. 526 LGC – LGC shall apply to all LGUs in the autonomous said RA 6734 be declared unconstitutional. They argue that said law
regions until such time as the regional government concerned shall violates the Constitution and is in conflict with the Tripoli Agreement.
have enacted its own local government code. In this case, the SC held that it is unnecessary to determine whether RA
6734 is in conflict with the Tripoli Agreement because it is now the
See relevant Constitutional Provisions 16: Art. X, Sec. 1, 15, 16, 18, 20, 1987 Constitution which provides for the creation of an autonomous
and 21 region. It also held that RA 6734 is NOT UNCONSTITUTIONAL. The
petitioners were not able to overcome the presumption of
Special Laws cited in the Syllabus: constitutionality.
1. RA 6734 (1989) – the first Organic Act of ARMM
2. RA 8746 (1999) – An act providing for the date of the regular Doctrine: Under the Constitution and RA 6734, the creation of the
elections of regional governor, vice-governor, and members autonomous region shall take effect only when approved by a majority
of the regional legislative assembly of the ARMM of the votes cast by the constituent units in a plebiscite, and only those
3. RA 9012 (2000) – An act resetting the regular elections for provinces and cities where a majority vote in favor of the Organic Act
elective officials of the ARMM to the 2nd Monday of Sept. shall be included in the autonomous region. The single plebiscite
2001 contemplated will therefore be determinative of (1) whether there shall
4. RA 9054 (2001) – the new Organic Act of ARMM be an autonomous region in Muslim Mindanao and (2) which provinces
and cities, among those enumerated in RA 6734, shall compromise it.
Class Notes: On Autonomous Regions
- The Region, by itself, is an LGU
 But unlike other LGUs, it has a separate legal system LIMBONA v. MANGELIN
 It can have its own LGC provided that it cannot decrease February 28, 1989
the power of the local government officials and the IRA
under the present LGC Summary: Sultan Limbona was an appointed member of the
- Power of the Autonomous region is limited by the Sangguniang Pampook, Regional Autonomous Government, Region XII,
Constitution and the Organic Act representing Lanao del Sur. He was also elected Speaker of the
 There must be at least 2 provinces to form the region Regional Legislative Assembly or Batasang Pampook of Central
 It requires a plebiscite Mindanao. Limbona was invited for consultations by the Committee of
Muslim Affairs of the House of Representatives, The invitation also
- What if in the upcoming BJE plebiscite, barangays want to be requested him to invite his fellow Assemblymen. In line with the
part of the BJE, apart from their mother province. invitation, he declared a “recess” in the Assembly. Without him, the
1. If we look at Art, X, Sec. 15: the [autonomous regions] … members of the Assembly had 2 sessions where his position as Speaker
consisting of provinces, cities, municipalities, and was declared vacant. He filed a case with the SC questioning this.
geographical areas… Pending resolution of the case, the Sangguniang Pampook removed him
a. Barangays are excluded from the enumeration as member through a Resolution. SC: The SC has jurisdiction. The SC
b. What do we mean by “geographical areas”? Maybe ruled for Limbona. As to his removal as member of the Sangguniang
like islands Pampook, there was a violation of due process. As to his removal as
2. Another thing to consider: The separation of the Speaker of the Regional Assembly, the SC found for him under equitable
barangays to form part of the BJE affects the integrity of grounds.
the Province
a. So can we argue that these barangays may separate Doctrine: Under the 1987 Constitution (Art. X) LGUs enjoy autonomy in
and form part of the BJE? Hard to argue 2 senses. The first sense under Secs. 1 and 2, the second sense under
3. Maybe if the barangay gets promoted to a Municipality, Sec. 15.
then it can form part of the Region as per Art. X, Sec. 15
An autonomous government that enjoys autonomy of the latter
- What if the organic act provides for a parliamentary system of category (Sec. 15.) is subject alone to the decree of the organic act
government in the autonomous region? Valid? creating it and accepted principles on the effects and limits of
 Yes, there is no prohibition nor violation "autonomy." On the other hand, an autonomous government of the
former class is under the supervision of the national government.
- Can the COA be devolved to the Autonomous Region?
 NO, COA is an independent constitutional body If the Sangguniang Pampook of Region XII is autonomous in the latter
 Art. IX, Sec. 3 – No law shall be passed exempting any sense, its acts are beyond the domain of the SC in the same way that
entity of the government or its subsidiary in any guise, internal acts of the Congress of the Philippines are. If it is autonomous
whatsoever, or any investment of public funds, from the in the former category only, SC has jurisdiction.
jurisdiction of the COA
An examination of the PD 1618 which created the autonomous
governments of Mindanao shows that they were never meant to
exercise autonomy in the second sense because PD 1618 (the organic
act)mandates that the President shall have the power of general
supervision and control over Autonomous Regions. Also the
Sangguniang Pampook (the legislative arm) is made to discharge chiefly
administrative services.

Class Notes: The Court made a statement in Limbona to the effect that
16 since autonomous regions have political autonomy, MAYBE the courts
For Constitutional Provisions, please refer to the Midterms Reviewer.
cannot look into internal matters of the autonomous region
There is a section on “Constitutional Basis” found in the first part of that - Sir said that the SC may come to regret this someday
reviewer complete with comments and notes.
PANDI v. CA - The Autonomous Region is allowed to legislate on its own,
April 11, 2002 except that it must be consistent with the Constitution and the
Organic Act; follow these steps:
Summary: Regional director and secretary of Dept of Health of ARMM 1. Check where local government official is set
Macacua issued a memorandum designating Dr. Lampa Pandi, who was 2. If ARMM, check their LGC
then DOH-ARMM Assistant Regional Secretary, as Officer-in-charge of 3. If there is no provision in their LGC, check if consistent
the IPHO-APGH Lanao del Sur. in the same memorandum, Macacua with the Constitution and the Organic Act
detail Dr. Mamasao Sani, who was then provincial health officer of 4. If no provision, assume that it is the same as RA 7160
IPHO-APGH, to DOH-ARMM Regional Office in Cotabato City. Lanao del 5. If silent, make the necessary appointment
Sur Provincial Governor Manhid Mutilan issued office order 7
designating Dr. Amer Saber also as officer-in-charge of IPHO-APGH
Lanao del sur. Saber filed with CA a petition for quo warranto claiming DISOMANGCOP v. SECRETARY OF DEPARTMENT OF PUBLIC
that he is the lawfully designated officer-in-charge of IPHO-APGH Lanao WORKS AND HIGHWAYS
del sur. CA ruled for Saber. SC reversed. November 25, 2004

Doctrine Under the ARMM Local Code, the provincial health officer Summary: RA 6734, the first organic act of ARMM was passed. Pursuant
became for the first an official of the provincial government even to this, EO 426 was enacted, placing the control and supervision of the
though he is appointed by the Regional Governor and draws his salary offices of the DPWH within the ARMM under the Autonomous Regional
from regional funds. The ARMM Local Code vests in the Provincial Government (ARG). DO 119 of the DPWH and RA 8999 created a DPWH
Governor the power to "exercise general supervision and control over Marawi Sub-District Engineering Office and an Engineering District in
all programs, projects, services, and activities of the provincial the First District of the Province of Lanao Del Sur. These essentially the
government." Upon the effectivity of the ARMM Local Code, the power same jurisdiction as the First Engineering District of the DPWH-ARMM
of supervision and control over the provincial health officer passed in Lanao Del Sur. Petitioners Disomangcop and Dimalotang, as OIC and
from the Regional Secretary to the Provincial Governor. From then on District Engineer/ Engineer II of the First District of the DPWH-ARMM
the Provincial Governor began to exercise the administrative authority in Lanao Del Sur, sought the revocation of DO 119, non-implementation
to designate an Officer-in-Charge in the provincial health office pending of RA 8999 and to compel the Secretary of DBM to release all funds for
the appointment of a permanent provincial health officer. public works intended for Marawi City and the First District of Lanao
del Sur to the DPWH-ARMM First District in Lanao Del Sur Only. The
Class Notes: Court held that (1) DO 119 violates the provisions of EO 426 but was
- There was a discussion of 5 periods: not issued in grade abuse of discretion and there is also no fault in
1. First – Prior to the Organic Act of 1989 failing to have public consultations. (2) The constitutionality of RA 8999
2. Second – After enactment of 1989 Organic Act but before is not the lis mota of this case. It never became operative and it was also
adoption of LGC 1991 superseded or repealed by a subsequent enactment, RA 9054. (3) The
3. Third – After enactment of LGC 1991 but before adoption petitioners having positions in the First Engineering District of DPWH-
of the ARMM Local Code ARMM have standing to file this case.
4. Fourth – After adoption of ARMM Local Code but before
enactment of 2001 Organic Act Doctrine: Cordillera Board Coalition v. COA: the creation of autonomous
5. Fifth – After enactment of 2001 Organic Act regions in Muslim Mindanao and the Cordilleras, which is peculiar to
- Thus, as the law now stands: (Fifth Period) the 1987 Constitution, contemplates the grant of political autonomy
 RA 9054 took effect on 2001, a total substitution of the and not just administrative autonomy to these regions.
Organic Act 1989
 The Organic Act 2001 adopted, as a minimum, the The aim of the Constitution is to extend to the autonomous peoples the
devolution under the LGC right to self-determination—a right to choose their own path of
 So the LGUs within the ARMM are given the same development; the right to determine the political, cultural and economic
devolved powers, functions, and tax-sharing content of their development path within the framework of the
entitlements enjoyed by other LGUs sovereignty and territorial integrity of the Philippine Republic. In
 Thus, the powers and functions of a Provincial treading their chosen path of development, the Muslims in Mindanao
Governor under the LGC are now enjoyed, as a are to be given freedom and independence with minimum interference
minimum, by the ARMM Governor from the National Government. This necessarily includes the freedom to
1. Meaning, the Provincial Governor appoints decide on, build, supervise and maintain the public works and
the PHO if the latter’s salary comes from infrastructure projects within the autonomous region.
provincial funds
2. If PHO salary comes from regional funds,
then the ARMM LGC applies
a. In which case the Regional Governor is
the appointing power but he must
appoint only from among 3 nominees of
the Provincial Governor
b. Provincial Governor exercises
supervision and control over the PHO
because ARMM LGC classified him as a
provincial government official
 Same state of law as after effectivity of ARMM LGC
but before Organic Act 2001
1. Only difference is that Regional Assembly
cannot amend the ARMM LGC to reduce
power of Provincial Governor because his
devolved power, emanating from LGC, is now
part of OA 2001
- Lesson: Validity of appointment depends on what legal regime
was operative at the time of appointment
province of Ifugao is to be included in the Cordillera Autonomous
THE CORDILLERA Region. It is the first issue which the Court answers in the instant case.

ADMINISTRATIVE REGION
CORDILLERA BOARD COALITION v. COA
Special Laws cited in the Syllabus January 29, 1990
1. EO 220 – The EO creating the CAR, appropriating funds
therefor and for other purposes Summary: Cordillera Broad Coalition and Lilia Yaranon, et al., assail EO
2. RA 6766 (1989) –Organic Act of the Cordillera Autonomous 220, which created CAR, as unconstitutional as it pre-empts the creation
Region of an autonomous region pursuant to the 1987 Constitution. The SC
3. RA 8438 (1997)– An act to establish the Cordillera ruled that what is created under EO 220 is not the autonomous region
Autonomous Region contemplated in the constitution. It was merely created to coordinate
planning and implementation of programs and services in the areas
NOTE: Plebiscite for the Cordillera Autonomous Region failed twice. covered, preparing them for autonomy.

Doctrine: The constitutional guarantee of local autonomy in the


ORDILLO v. COMELEC Constitution refers to the administrative autonomy of local government
December 4, 1990 units or, cast in more technical language, the decentralization of
government authority. On the other hand, the creation of autonomous
Summary: Pursuant to Republic Act No. 6766 entitled “An Act Providing regions contemplates the grant of political autonomy and not just
for an Organic Act for the Cordillera Autonomous Region”, the people of administrative autonomy. Thus, the provision in the Constitution for an
the provinces of Benguet, Mountain Province, Ifugao, Abra and Kalinga- autonomous regional government with a basic structure consisting of
Apayao and the city of Baguio cast their votes in a plebiscite. Results of an executive department and a legislative assembly and special courts
plebiscite: approved by majority of votes in Ifugao, rejected by the rest with personal, family and property law jurisdiction in each of the
of the provinces and city. COMELEC issued Resolution No. 2259 stating autonomous regions.
that the Organic Act for the Region has been approved and/or ratified
by majority of votes cast only in the province of Ifugao. Secretary of
Justice also issued a memorandum for the President reiterating BADUA v. CORDILLERA BODONG ADMINISTRATION
COMELEC resolution. Congress enacted Republic Act No. 6861 setting February 14, 1991
elections in CAR of Ifugao on first Monday of March 1991. Even before
COMELEC resolution, Executive Secretary issued February 5, 1990 a Summary: The tribal court of Maeng Tribunal decided that the land
memorandum granting authority to wind up the affairs of the Cordillera which is subject of a disputed between the sps. Badua and Quema
Executive Board and Cordillera Regional Assembly created under belongs to the latter and ordered that the Baduas vacate the land. When
Executive Order No. 220. President issued Administrative Order No. the Baduas failed to obey, they received a warning order from the
160 declaring among others that the Cordillera Executive Board and Cordillera People’s Liberation Army which made them fearful of their
Cordillera Regional Assembly and all offices under Executive Order No. lives and file a case to SC to prevent the execution of the tribal court
220 were abolished in view of the ratification of Organic Act. Arguing decision and also to question the said court’s legal personality. The SC
that there can be no valid CAR in only one province as the Constitution ruled that the tribal court is just like the conciliation panels in
and RA 6766 require that the said Region be composed of more than barangays whose decisions can be set aside by the regular courts.
one constituent unit, the petitioners prayed that the said memoranda
and Administrative Order be declared null and void, and that EO 220 Doctrine: [As a consequence of the rejection of the people of creating
constituting the Cordillera Executive Board and the Cordillera Regional Cordillera Autonomuse Region] The Maeng Tribunal Court was not
Assembly be still in force and effect until another organic law for the constituted into an indigenous or special court under RA 6766, hence, it
Autonomous Region shall have been enacted and the same is duly is an ordinary tribal court existing under the customs and traditions of
ratified by the by the voters in the constituent units. The SC declared an indigenous cultural community. Such tribal courts are not a part of
that the sole province of Ifugao CANNOT validly constitute the the Philippine judicial system which consists of the Supreme Court and
Cordillera Autonomous Region. the lower courts which have been established by law (Sec. 1, Art. VIII,
1987 Constitution)

Doctrine: The keywords (in Art. X, Sec. 15 of the 1987 Constitution) —


provinces, cities, municipalities and geographical areas connote that ATITIW v. ZAMORA
"region" is to be made up of more than one constituent unit. The term September 30, 2005
"region" used in its ordinary sense means two or more provinces. This
is supported by the fact that the thirteen (13) regions into which the Summary: Atitiw et al., filed a petition for prohibition, mandamus and
Philippines is divided for administrative purposes are groupings of declaratory relief the executive secretary, secretary of the DBM , and
contiguous provinces. (Integrated Reorganization Plan (1972), which the Republic of the Philippines. They sought to have par. 1 of the special
was made as part of the law of the land by P.D. No. 1; P.D. No. 742) provisions of the FAA of 2000 declared unconstitutional, as it provided
Ifugao is a province by itself. To become part of a region, it must join for the use of the funds appropriated to the CAR for the winding up of
other provinces, cities, municipalities, and geographical areas. Aside its operations. The SC upheld the constitutionality of the assailed
from the 1987 Constitution, a reading of the provisions of Republic Act provision.
No. 6766 strengthens the position that the Region cannot be constituted
from only one province. Doctrine: The CAR is not the autonomous region contemplated by the
Constitution.
The Abbas case established the rule to follow on which provinces and
cities shall comprise the autonomous region in Muslim Mindanao which The creation of autonomous regions does not signify the establishment
is, consequently, the same rule to follow with regard to the autonomous of a sovereignty distinct from that of the republic, as it can be installed
region in the Cordillera. However, there is nothing in the Abbas decision only “within the framework of the Constitution and the national
which deals with the issue on whether an autonomous region, in either sovereignty as well as territorial integrity of the Republic of the
Muslim Mindanao or Cordillera could exist despite the fact that only Philippines. “
one province or one city is to constitute it. The issue in this case is
whether the sole province of Ifugao can validly and legally constitute
the Cordillera Autonomous Region. The issue is not whether the
THE METROPOLITAN MANILA Doctrine: It is thus beyond doubt that the MMDA is not a local
government unit or a public corporation endowed with legislative
DEVELOPMENT AUTHORITY power. It is not even a "special metropolitan political subdivision" as
contemplated in Section 11, Article X of the Constitution. The creation
Special Laws cited in the Syllabus of a "special metropolitan political subdivision" requires the approval
1. RA 7924 – An act creating the MMDA by a majority of the votes cast in a plebiscite in the political units
2. See also the following Constitutional provisions directly affected." R. A. No. 7924 was not submitted to the inhabitants of
a. Art. X, Sec. 11 – Congress may, by law, create special Metro Manila in a plebiscite. The Chairman of the MMDA is not an
metropolitan political subdivisions, subject to a official elected by the people, but appointed by the President with the
plebiscite rank and privileges of a cabinet member. In fact, part of his function is
(1) Component cities and municipalities shall retain to perform such other duties as may be assigned to him by the
their basic autonomy and shall be entitled to their President,whereas in local government units, the President merely
own local executive and legislative assemblies exercises supervisory authority. This emphasizes the administrative
(2) Jurisdiction of the metropolitan authority that will character of the MMDA.
thereby be created shall be limited to basic
services requiring coordination Clearly then, the MMC under P.D. No. 824 is not the same entity as the
MMDA under R.A. No. 7924. Unlike the MMC, the MMDA has no power
b. Art. XVIII, Sec. 8 - Until otherwise provided by the to enact ordinances for the welfare of the community. It is the local
Congress, the President may constitute the government units, acting through their respective legislative councils,
Metropolitan Manila Authority to be composed of the that possess legislative power and police power. In the case at bar, the
heads of all local government units comprising the Sangguniang Panlungsod of Makati City did not pass any ordinance or
Metropolitan Manila area. resolution ordering the opening of Neptune Street, hence, its proposed
opening by petitioner MMDA is illegal .

MMDA v. GARIN
April 15, 2005 MMDA v. TRACKWORKS RAIL TRANSIT ADVERTISING
October 25, 2005
Summary: Garin’s license was confiscated by the MMDA. He questioned
the authority of the MMDA to confiscate drivers’ licenses. The Court Summary: MRTC entered into a contract for advertising services with
ruled that while a driver’s license may be confiscated pursuant to police Trackworks. This gave Trackworks the exclusive right to undertake
power, the MMDA is not vested with police power. It cannot confiscate advertising and promotional activities within and along the exterior
licenses absent any law authorizing it to do so. and interior of the MRT3 structure. Thereafter, Trackworks installed
commercial billboards, banners, signages and other forms of
Doctrine: Metropolitan or Metro Manila is a body composed of several advertisement in the different parts of MRT structure. MMDA requested
local government units. With the passage of RA 7924 in 1995, Trackworks to dismantle the billboards in conformity with MMDA
Metropolitan Manila was declared as a "special development and Regulation No. 96-009, prohibiting the posting, installation, and display
administrative region" and the administration of "metro-wide" basic of any kind of form of billboards, signs, posts, streamers, in any part of
services affecting the region placed under "a development authority" the road, sidewalk, center island, posts, trees, parks and open space.
referred to as the MMDA. It is NOT an LGU. The power delegated to the Trackworks refused to comply, invoking its advertising contract. MMDA
MMDA is that given to the Metro Manila Council to promulgate then started dismantling the billboards and streamers of Trackworks.
administrative rules and regulations in the implementation of the Trackworks filed with the trial court a petition for injunction with
MMDA’s functions. There is no grant of authority to enact ordinances prayer for the issuance of a TRO and preliminary injunction. TC
and regulations for the general welfare of the inhabitants of the granted. MMDA did not file an MR but instead filed for certiorari in the
metropolis. CA. The CA denied certiorari. The SC affirmed the TC and CA decisions.
Trackworks was able to establish a right to be protected by a writ of
Where there is a traffic law or regulation validly enacted by the preliminary injunction.
legislature or those agencies to whom legislative powers have been
delegated (the City of Manila in this case), the MMDA is duty-bound to Doctrine: A contract gives rise to a right that can be protected by a writ
confiscate and suspend or revoke drivers’ licenses in the exercise of its of injunction. While the power or authority of a government agency
mandate of transport and traffic management, as well as the being questioned, a writ of injunction may be issued to protect the
administration and implementation of all traffic enforcement private party’s rights.
operations, traffic engineering services and traffic education programs.

FRANCISCO v. FERNANDO
MMDA v. BEL AIR VILLAGE ASSOCIATION November 16, 2006
March 27, 2000
Summary: Ernesto Francisco wants to enjoin Bayani F. Fernando,
Summary: MMDA sent a notice to BAVA that it shall be taking down its Chairman of the Metropolitan Manila Development Authority (MMDA)
wall and opening Neptune Street to the public, BAVA filed for a Writ of and the MMDA from further implementing its “wet flag scheme” (“Flag
Preliminary Injunction in the RTC, which was granted and upheld in the Scheme”).
CA. In the SC the same is upheld because the court discusses that police
power may only be exercised by government units where the National Francisco also wishes that Fernando and the MMDA “respect and
Assembly has devolved its powers to like towards Local Governments, uphold the x x x rights of pedestrians to due process x x x and equal
through the General Welfare Clause Sect 16 of the Local Government protection of the laws x x x.”
Code. The MMDA on the other hand being an administrative agency
have no such powers because while the predecessor of the MMDA, Francisco argues that the MMDA's Flag Scheme is illegal since:
which is the MMC, had powers to issue Ordinances, and the MMDA was (1) it has no legal basis because the MMDA’s governing body, the
purposely made into a governmental agency whose head has the rank Metro Manila Council, did not authorize it;
of a cabinet member, its function is administrative in nature and cannot (2) it violates the Due Process Clause because it is a summary
issue ordinances, only rules and regulations. punishment for jaywalking;
(3) disregards the Constitutional protection against cruel,
degrading, and inhuman punishment; and The MMDA performs planning, monitoring, and coordination functions,
(4) violates “pedestrian rights” as it exposes pedestrians to having regulatory and supervisory authority over metro-wide service-
various potential hazards. delivery without diminution of LGU autonomy on purely local matters.

On the other hand, Fernando and the MMDA contend that the Flag There is also no specific grant of authority/police power to the MMDA
Scheme is a valid preventive measure against jaywalking. (citing MMDA v. Bel-Air Village Association, Inc., and MMDA v. Garin),
so the President cannot delegate it as the project’s implementing
DOCTRINE: The enactment by cities and municipalities within the agency. Consequently, the MMDA cannot order the “elimination” of the
MMDA's jurisdiction of anti-jaywalking ordinances or traffic terminals.
management codes with provisions for pedestrian regulation suffices to
confer upon the MMDA authority to implement schemes to enforce the
same. MMDA v. CONCERNED RESIDENTS OF MANILA BAY
December 18, 2008
On the Flag Scheme’s alleged lack of legal basis, the Supreme Court
notes that all the cities and municipalities within the MMDA’s Summary: On January 29, 1999, Concerned Residents of Manila Bay
jurisdiction, EXCEPT Valenzuela City, have each enacted anti-jaywalking filed a complaint before the RTC in Imus, Cavite several government
ordinances or traffic management codes with provisions for pedestrian agencies MMDA, DENR, DECS,DOH, DOA, DPWH, DBM, PPG, PNP
regulation. Maritime Group, DILG, for the cleanup, rehabilitation, and protection of
the Manila Bay. RTC ordered petitioners to Clen Up and Rehabilitate
Such fact serves as sufficient basis for Fernando and the MMDA’s Manila Bay. CA affirmed the RTC decision. Government agencies
implementation of schemes to enforce anti-jaywalking ordinances and contended that the the cleaning of the Manila Bay is not a ministerial
similar regulations. The MMDA is an administrative agency tasked with act which can be compelled by mandamus and taht pertinent provisions
the implementation of rules and regulations enacted by proper of the Environment Code (PD 1152) relate only to the cleaning of
authorities. The absence of an anti-jaywalking ordinance in Valenzuela specific pollution incidents and do not cover cleaning in general. SC
City does not detract from this conclusion ABSENT any proof that disagreed and upheld the CA decision.
Fernando and the MMDA implemented the Flag Scheme in that city.
Doctrine: Obligation to perform their duties as defined by law and how
WON the Flag Scheme is a reasonable enforcement of anti-jaywalking they are to carry out such duties are 2 different concepts. While
ordinances and similar enactments. The Supreme Court can only implementation of MMDA’s mandated tasks may entail decision-
determine the reasonableness of the enforcement by the MMDA of anti- making, the enforcement of the law or the very act of doing what the
jaywalking ordinances and similar enactments if there are concrete law exacts to be done is ministerial in nature and may be compelled by
facts alleged and prove which demonstrate how they are not mandamus. When RA 9275 (Clean Water Act) took effect, its Section 16
reasonable. The Supreme Court is not a trier of facts. on the subject, Cleanup Operations, amended Section 20 of the
Environment Code (PD 1152). Section 17 mandating upgrading of
Furthermore, the petition proffers mere SURMISES and SPECULATIONS water quality however, continues to be operational. This section,
on the potential hazards of the Flag Scheme. The Court cannot commands concerned government agencies, when appropriate, “to take
determine the reasonableness of the Flag Scheme based on mere such measures as may be necessary to meet the prescribed water
surmises and speculations. quality standards.” In fine, the underlying duty to upgrade the quality of
water is not conditional on the occurrence of any pollution incident

MMDA v. VIRON TRANSPORTATION CO. Class Notes: Lessons from the cases:
August 15, 2007 - MMDA is NOT an LGU
- There are certain programs that are implemented in certain
Summary: As early as 1969, traffic was a problem in Metro Manila. On cities, to the exclusion of the other member cities
February 10, 2003, then-President Gloria Macapagal-Arroyo issued an - MMDA has no legislative power nor police power
Executive Order (179) for the establishment of the Greater Manila Mass
Transport system. It designated the MMDA as the implementing agency
for a mass transport terminal facilities project, which was
recommended by the MMDA as a means to alleviate Metro Manila’s END OF SYLLABUS
traffic caused by buses and the inefficient connectivity of transportation
systems. The MMDA was to prepare the master plan, coordinate land
use, supervise and manage construction, execute the necessary
contracts and agreements, manage the funds, enlist the assistance of
other government instrumentalities, and assign and hire personnel. The
project also entailed the closure of existing provincial bus terminals.
The Metro Manila Council supported the project, citing bus terminals as
a contributor to traffic. Provincial bus operators (Viron Transport Co.,
Inc. and Mencorp Transportation System, Inc.) sought declaratory
relief against the project. The trial court initially found for the MMDA
but reversed itself. The SC upheld the trial court’s reversal, holding that
the MMDA cannot be the implementing agency for the project as its
charter does not include police or legislative powers. Assuming that it
has police powers, the proposed project fails the test for the valid
exercise of police power.

DOCTRINE: The MMDA charter provides specific functions, and there is


no grant of either police or legislative power. The MMDA performs
planning, monitoring, and coordination functions, (§2) so it has
regulatory and supervisory authority over the delivery of metro-wide
services, including transport and traffic management (§3).
END OF LOCAL GOVERNMENT COURSE

You might also like